Ευκλειδης Β 50

Page 1


ΕΚΔΟΣΕΙΣ ΠΑΤΑΚΗ www.patakis.qr

Νεκτάριος Πρωτοπαπάς

Γ. Βιδάλης- Β. Γκιμίσης

Α' Ενιαίου Λυκείου

Γενικής Π α ιδείας , Β' Ενιαίου Λυκείου

Φυσική

Άλγεβρα

Μ. Γεωργάκης, Αθ. Μακρίδης κ.ά. Ευκλείδεια Γεωμετρία Γενικής Παιδείας, Β' Ενιαίου Λυκείου

ΜΕθΟΔΟΛΟΓΙΑ

Φυσικής β'

ενιο ov

λvκειοu

Κ. Α. Κυριακόπουλος

Αντώνης Σαρρηγιάννης

Θετικής Κατεύθυνσης, Β' Ενιαίου Λυκείου

θετικής και Τεχνολογικής Κατεύθυνσης,

Μαθηματικά

Μεθοδολογία Φυσικής Β' Ενιαίου Λυκείου

I.

Φυσική

Γενικής Παιδείας, Β' Ενιαίου Λυκείου

χημε},g

μαθηματικά

Κεφαλλωνίτης, Β. Χρηστίδης

Ματθαίος Τσιλπφίδης

e· Ενιαίου Λuκείοu

" 10)ο'0( i!M(fΣ-4θE:CI'WI.fO':\IAU;OM•ΘfP'.IOιΔ.Y'Iλ*H ••' •• � • "• •ο •.:> •" � • •" • • '"'"

Νεκτάριος Πρωτοπαπάς

Χημεία

θετικής Κατεύθυνσης,

Β' Ενιαίου Λυκείου

r· Ενιαίου Λυκείου

Μαθηματικό

Γενικής Παιδείας, Γ Ενιαίου Λυκείου

ιουοική .....,. ____'""1•--............. "1...__�·· ......

: . . · · · · · · · · · · · · ·· -

Τριαντάφυλλος Μελισσαρόπουλος Φυσική θετικής και Τεχνολογικής Κατεύθυνσης, Γ' Ενιαίου Λυκείου

Τάσος Βαρδόπουλος

Μεθοδολογία Χημείας . θετικής Κατεύθυνσης, Γ' Ενιαίου Λυκείου

NEkTAI'IOt nPoron.uι.ι.ι

Νεκτάριος Πρωτοπαπάς Κριτήρια αξιολόγησης

στη Φυσική Γενικής Παιδείας, Γ Ενιαίου Λυκείου

Ελευθέριος Πρωτοπαπάς Μαθηματικό θετικής και Τεχνολογικής Κατεύθυνσης, Γ' Ενιαίου Λυκείου


ΕΛΛΗΝΙΚΗ ΜΑΘΗΜΑΤΙΚΗ ETAIPEIA Τειίχος

Υηι�ύ()υνοι Έκδοσψ;

Σuvιψy<:'ιιι�<;:

Φελλούρης Ανάργυρος, Σμαριανάκης Μιχάλης, Σταμέλος Γιάννης, Μαλλιάκας Κώστας, Μπουρνής Α, Καραγιάννης Ιωάν­ νης, Κουνάδης Φώτης, Σεiτης Βασίλης, Καραβόλιας Μιχάλης, Σuντuχάκης Χαρά­ λαμπος, Λιγνός Ηλίας

Οκτώβριος

Νοtμβριος

Δεκtμβριος

e·mail: info@hms.gr www.hms.gr

2003

Ευρώ:

2,50

ΠΕΡΙΕΧΟΜΕΝΑ

Μπαραλής Γεώργιος.

Συνιακιικr1 0ιιι1δα:

ΜΑΘΗΜΑJlKO ΠΕΡΙΟΔΙΚΟ ΠΑ ΤΟ ΛYKEIO

Ευσταθίού Ευάγγελος,

Ανδρουλακόκης'Νίκος Βακαλόπουλος Κώστας Βισκαδουρόκης Βασίλης Βλόχου Αγγελική Γιούσης Θεόδωρος Γράψαj; Κωνσταντίνος Δούν<ψης Αντώνης Ευθιηίογλου Πέτρος Θεοδωρέλος Χρίστος Καλίκας Σταμάτης Καρακάσογλου Αλέξανδρος Καρακατσάνης Βασίλης Καρκάνης Βασίλης Καισούλης Γιώργος Κερασαρίδης Γιάννης Κrίπουρός Χρήστος Κ 6νψας Νίκος Κυριακόπουλος Αντώνης Κυριακόπουλος θανάσης Κυpιακοπούλου-Κυβερνήτου Χρυστ. Λ�αρίδης Χρ�στος Λουρίδας Σωτηρης Μαλαφέκ� Θανάσης Μαλιδόκη Ελένη Μεταξάς Νίκος Μυλωνάς ΔrιJ.nΊτρης Μώκος Χρήστος Ντζιώρας Ηλίας Παπ<ψάν Μαριλένα Παππά ς Γιώργος Ρήγας Θεόδωρος Σ<Ufελλάpης Βασίλης ΣαϊUJΕυα Σταθόπουλος Γιώργος Στάϊκος Κωνσταντίνος Στάϊκος Παναγιώτης Στρατής Ιωάννης Ταπεινός Νικόλαος Τασσόπουλος Γιώργος Τριάντος Γεώργιος Τσικαλουδάκης Γιώργος Τσιούμας Θανάσης Χαραλαμπίδης Γιώργος Χαραλαμποπούλου Λίνα ΧαραλάJπιους Θάνος Χριστόπουλος Παναγιώτης

50

2

5

ΕΥΔΗΜΟΣ Ο ΡΟΔΙΟΣ

ΤΟ ΒΗΜΑ ΤΟΥ ΕΥΚΛΕΙΔΗ

18 Ο Ευκλείδη� προιείvει... Ευκλείδη και... Διόφαvιο 21 ΗΟΜΟ MATHEMAτiCUS

24 Μαιlημαηκοί Διαγωνισμοί - Μαιlημαιtκέ� Ολυμπιάδε�

ΜαΟημαιικ(ί για ιην Α'

Τάξι1 ιοu Λuκcίοu

36 Τριώνυμο Β' Οαιlμού

40 Σχέσει� μειαξύ ιωv συvιελεσιώv και ιωv ριζών εξίσωση� 2ou Οαιlμού 43 Παράλληλε� ευιlείε� - λιlροισμα γωνιών ιριγώvωv

48 Δρασιηριόιηια για ιο 5ο κεφάλαιο (παραλληλόγραμμο)

Μα()ι]μαι ικ{ι yια ιηv Β'

Τάξη ιοu Λuκι�ίοu

49 Πολυώνυμα

54 Ο λόγο� ιωv εμΟαδώv δύο ιριγώvωv

58 Ευιlείε�

Μαllημοιικ(ι για ιφ' I" ΤΜ,rι ιοu Λuκι:ίοu 64 Σιαησηκή. Ερμηνεία και καιασκευή διαγραμμάιωv 71 Ασκήσει� σια όρια

ΣΥΝΔΡΟΜΕ� 2,50 ευρώ Ετήσια συνδρομή 10,00 ευρώ (Σχολ) Συvδροιτητές: 10,00+-2,00 ευρώ (Ί'αχ.) Οιι,·Ιη'ιcψΗί: 18,00 ευρώ Ί'1ιχ. Ειιιι•ιγι�c; τ. ι 'ι•cιφι·ίcι ι\/Ιιj "" :;4, . .. . ... . :�0044

Τεύχος:

11

74 Διαφορικό� Λογισμό� Σι οι χ'' ιοllι·.ο ίιι

Ν. Αλεξόπουλος

·

Σι·. ί.ιliοιι ο ίηοιι

Ακαδηιήας 43, τηλ 210-3606826

Εκτύπωση ΙΝΤι-:ι•ιιι•ι·:l: Α.ι-:., Ιεράοδός81- 83 Υπεuθ. Τυπογραφείου. Π Τρureριώnις-Τηλ:3474654


θΥΑΗΜΟC

Σ · ··

··'

ΡΟΑΙΟC

Μιχάλης Σμα ριανάκης

το τελευ�αίο μισό το� 4°υ π.Χ. αιώνα έζη �ε , και συνεγραψε τα εργα του ο γνωστος στην αρχαιότητα ως Εύδημος ο Ρόδιος, κάποια από τα βιβλία του οποίου θεωρούνται οι πιο σημαντικές πηγές για τα Ελληνικά Μαθηματι­ κά της προ-Ευκλείδειας περιόδου. Ο καθηγητής της Ιστορίας των Μαθηματικών Γιάννης Χριστια­ νίδης στο βιβλίο του «Θέματα από την Ιστορία των Μαθηματικών)) αναφέρει τα παρακάτω: «Οι γνώσεις μας για τα μαθηματικά που ανα­ πτύχθηκαν τον 6° και 5° π.Χ. αιώνα είναι τελείως αποσπασματικές. Τα σωζόμενα κείμενα της εποχής είναι ελάχιστα και τις περισσότερες πληροφορίες που έχουμε τις οφείλουμε σε συγγραφείς που έζη­ σαν μέχρι και 1 000 χρόνια αργότερα! Συγκεκριμέ­ να σε ότι αφορά τον 5° αι., διασώζεται ένα από­ σπασμα του μαθητή του Αριστοτέλη Ευδήμου του Ροδίου, που αναφέρεται στον τετραγωνισμό των μηνίσκων από τον Ιπποκράτη τον Χίο, ενώ για τον 4° αι. υπάρχει άλλο ένα απόσπασμα του Ευδήμου για τον διπλασιασμό του κύβου από τον Αρχύτα τον Ταραντίνο· για την ίδια περίοδο επίσης δεν πρέπει να παραλείψουμε να μνημονεύσουμε τα μαθηματικά χωρία που περιέχονται στα γραπτά του Πλάτωνα και του Αριστοτέλψ). Οι συγγραφείς που εμφανίστηκαν μέχρι και 1 000 χρόνια αργότερα από την εποχή του Ευδήμου και στους οποίους αναφέρεται το πιο πάνω από­ σπασμα, είναι κυρίως ο Σιμπλίκιος (σχολιαστής του Αριστοτέλη, ήκμασε περί το 520 μ.Χ.) και ο Πρόκλος (νεοπλατωνικός φιλόσοφος, έζησε το 4 1 0-485 μ. Χ.). Αναφορές στον Εύδημο το Ρόδιο έχουμε από τους Πορφύριο, Διογένη Λαέρτιο, Αλέξανδρο τον --·

ο

Αφροδισιέα, Βοήθιο, ο δε Γεμίνος έγραψε Ιστορία μαθηματικών βασισμένη στο έργο του Εύδημου. Αποσπάσματα έργων του που σώθηκαν σε άλ­ λους συγγραφείς εκδόθηκαν από το Mullach (Fragmenta Pfilosopforum Graccorum 1 88 1 ). Από τον Πρόκλο γνωρίζουμε ότι ο Εύδημος έ­ γραψε έναν αυθεντικό μαθηματικό έργο με τίτλο «Περί γωνίαρ), το οποίο χάθηκε. Ίσως υπήρξαν και άλλα τέτοια έργα του Ευδή­ μου, αφού όπως είναι γνωστό από τη διαδικασία ε­ πανειλημμένων aντιγραφών αρχαίων κειμένων α­ πό τους μοναχούς και καλλιγράφους του μεσαίωνα και του Βυζαντίου, «επιβίωσαν)) και διασώθηκαν μόνο εκείνα τα έργα τα οποία δεν θεωρήθηκαν κα­ τά τη γνώμη τους παρωχημένα. Για παράδειγμα τα έργα τα σχετικά με τις κωνικές τομές του Αρισταί­ ου του Πρεσβύτερου «Στερεοί τόπου) και του Ευ­ κλείδη «Κωνικά)) δεν διασώθηκαν, παρά μόνον οι τίτλοι, αφού περιέπεσαν σε αχρηστία μετά την εμ­ φάνιση των Κωνικών του Απολλώνιου που γρά­ φτηκαν 1 50 χρόνια αργότερα. Έτσι η άποψη ότι ο Εύδημος δεν ήταν μόνο ο πρώτος σημαντικός ιστpρικός των μαθηματικών (όπως αναφέρουν σε άρθρο τους με τη βιογραφία του Εύδημου οι καθηγητές του πανεπιστημίου St Andrews της Σκοτίας, J.J. O'Connor και E.F. Rob­ ertson) αλλά και ενεργός μαθηματικός, δεν μπορεί να θεωρηθεί αβάσιμη και επομένως είναι ανοιχτή στην έρευνα. Φυσικά όπως και οι περισσότεροι σύγχρονοί του διανοούμενοι δεν ήταν μαθηματικός με την έννοια που δίνεται σήμερα στον όρο. Στη χρονο­ λογική σύνοψη που παραθέτει ο B.L. Van der Waerden στο κλασσικό σύγγραμμα Ιστορίας μα­ θηματικών «Η αφύπνιση της επιστήμηρ), κατα-

ΕΥΚΛΕΙΔΗΣ Β ' λζ' τ.2/2


ΕΥ ΔΗΜΟΣ Ο ΡΟΔΙΟΣ

τάσσει τον Εύδημο ανάμεσα στους πέντε σημαντι­ κότερους φιλοσόφους και ιστορικούς της περιόδου από τον Επαμεινώνδα 370 π.Χ. έως τον Μέγα Α­ λέξανδρο 333 π.Χ. (Σωκράτης - Πλάτων - Ηρα­ κλείδης ο εκ,Πόντου - Αριστοτέλης-Εύδημος). Ήταν μαθητής του Αριστοτέλη, εξ' ου και Πε­ ριπατητικός φιλόσοφος και μάλιστα μαζί με τον Θεόφραστο τον Λέσβιο, ήταν υποψήφιος διάδοχός του στη σχολή. Σύμφωνα με πληροφορίες του Αύλου Γελλίου ο Αριστοτέλης για πολύ καιρό δίσταζε να αποφασί­ σει ποιον θα ορίσει διάδοχο. Τελικά όpισε τον Θε­ όφραστο, λέγοντας ότι: ο μεν Ρόδιος οίνος και αδρός, αλλ' ο Λέσβιος γλυκύτερος. Μετά απ' αυτό έφυγε από την Αθήνα και ίδρυ­ σε δική του σχολή, πιθανόν στην πατρίδα του την Ρόδο. Γνωρίζουμε ότι γεννήθηκε στην Ρόδο και ότι είχε αδελφό τον Βόηθο. Δεν γνωρίζουμε τίποτα για την καταγωγή του.

Ως προς το συγγραφικό του έργο, είναι γνωστά τρία βιβλία της Ιστορίας Μαθηματικών. 1 ) Ιστορία της αριθμητικής (τουλάχιστον δύο βιβλία) αναφερόμενη από τον Πορφύριο, που πραγματεύεται την Πυθαγόρεια θεωρία των αριθ­ μών και την αλληλεξάρτηση τους με τη μουσική. 2) Ιστορία της Γεωμετρίας (τουλάχιστον δύο βιβλία). Θεωρείται η σημαντικότερη εργασία του. Ο J.L. Heiberg ισχυρίστηκε ότι ο Πάππος και ο Ευτόκιος, έγραφαν με ανοιχτή μπροστά τους την Ιστορία της Γεωμετρίας του Εύδημου. Χρησιμο­ ποιήθηκε από τον Πρόκλο στα υπομνήματά του στον Ευκλείδη. Χάρις σ' αυτό κυρίως το βιβλίο έ­ γιναν γνωστά τα μαθηματικά επιτεύγματα του Θα­ λή. Στο ίδιο βιβλίο Γεωμετρίας του Εύδημου ανα­ φέρονται και τα σχετικά με τον Ιπποκράτη τον Χίο όπως και για τον Αρχύτα τον Ταραντίνο που ανα­ φέραμε στην αρχή του άρθρου μας. Ο Χριστιανίδης στο βιβλίο που προαναφέραμε (σ. 60) γράφει:

=�

ΓΕΝΙΚΑ ΘΕΜΑΤΑ ΜΑΘΗΜΑτΙΚΩΝ

ΓΕΝΙΚΑ θΕΜΑΤΑ

ΜΑΘΗΜΑ1ΙΚΩΝ

Συνδρομές Μελών

r Λ\ΊαιΟΥ θffoott-τrxNOriCιrd\111 ΚΑΠΥii"\'ΝlΗΣ

Γ' ΛΥΚΕΙΟΥ ΘΗΙΚΗΣ • τΕΧΝΟΛΟΓΙΚΗΣ ΚΑτΕΥΘΥΝΣΗΣ

Συγγραφική ομάδα Μαθηματικών: Β.ΒΛΑΧΟΣ·Π.ΚΟγΤΣΟγΚΟΣ·

Π.ΞΗΡΟΚΩΣΤΑΣ-Χ.ΠΛΑΤΗΣ •

320 θέματα από όλη την ύλη όλα aπαντη­

μ έ ν α . Συν δ υ α σ τ ι κ ά θ έ μ α τ α. Τα λ ε π τ ά

σημεία των Μαθηματικών. Για τους μαθη­

Λόγω της Ολυμπιακής Χρο­

τές που στοχεύουν στο 30 και 40 θέμα. Για τους καθηγητές που

διδάσκουν στο σχολείο, στο φροντιστήριο, στο ιδιαίτερο.

νιάς και τις αυξημένες δραστη­ ριότητες της

Ελληνικής Μα­

θηματικής Εταιρείας

παρακα­

"Ενα βιβλίο που ήδη αγαπήσατε.

ΦΡΟΝτΙΗΗΡΙΑΚΗ ΦΥΣΙΚΗ ΓΕΝΙΚΗΣ ΠΑΙΔΕΙΑΣ

λούμε τα μέλη της όπως τακτο­

Συγγραφική ομάδα Φυσικών:

ποιήσουν τις οικονομικές οφει­

Κ. ΑΝΔΡΟγτΣΑΚΗΣ

Ελ­

ληνική Μαθηματική Εταιρεία να αντεπεξέλθει στις μεγάλες απαιτήσεις των καιρών. '

ΛΥΚ ΕΙΟΥ

Β. ΒΛΑΧΟΣ- Δ. ΦΙΛΙΠΠΑΚΟΠΟΥΛΟΣ ­

λές τους, για να μπορεί η

Β'

"Ολη η θεωρία, ερωτήσεις aπαντημένες

ΗΛΟΟΡΙΣΜΟΣ

Jι ·ι •·

��r!Aka

fLJΚH

Β' AYΚf.tOV

και χωρίς απάντηση, πολλές ασκήσεις λυμέ­ νες αναλυτικά. Τα λεπτά σημεία της φυσικής ανά παράγραφο (να προσέχετε ότι:). Δοκιμασμένο στην πράξη: Μαθητής που θα διαβάσει τη θεωρία, τις

παρατηρήσεις και τις λυμένες ασκήσεις γράφει πάνω από 15. Μια διαφορετική φιλοσοφία στο σχολικό βοήθημα.

ΕΚΔΟΣΕΙΣ: ΒΑΣΙΛΗΣ ΒΛΑΧΟΣ (Μαθηματικός και Φυσικός Παv. Αθήνας) Παπαφλέσσα 3 Ζεφύρι Τ.Κ. 13451 Τηλ.: 2102384834, Fax: 210 2320954

ΕΥΚΛΕΙΔΗΣ Β ' λζ' τ.2/3


ΕΥ ΔΗΜΟΣ Ο ΡΟΔΙΟΣ

Αν και ο Πρόκλος έζησε τον 5° μ.Χ. αι. η επι­ σκόπησή του θεωρείται αξιόπιστη, γιατί βασίζεται σε μεγάλο βαθμό στη χαμένη σήμερα Γεωμετρική Ιστορία του Εύδημου, ο οποίος έζησε στα τέλη του 4ου π.Χ. αι., δηλαδή σε μια εποχή σχετικά κοντινή προς την περίοδο που εξετάζουμε. 3) Αστρονομική Ιστορία. (μνημονεύεται από τον Διογένη τον Λαέρτιο ως «Περί των Αστρολ<r γουμένων Ιστορία))). Χρησιμοποιήθηκε και αυτή ευρέως από μετέ­ πειτα συγγραφείς. Η θεωρία του Θαλή για την έκλειψη περιγρά­ φεται σ' αυτό το έργο του Εύδημου και πιστεύεται ότι το σύστημα ομόκεντρων σφαιρών του Ευδόξου περιγράφεται για πρώτη φορά εκεί και αργότερα μεταδόθηκε σε μας μέσα από τα γραπτά του Σι­ μπλίκιου τον 2° μ.Χ. αι. Άλλα θέματα που περιλαμβάνονται σ' αυτό το βιβλίο είναι: Ο κύκλος του μεγάλου έτους μετά το οποίο όλα τα ουράνια σώματα βρίσκονται στις ίδιες σχετικές θέσεις η αντίληψη του Αναξίμανδρου ότι η σελή­ νη αντανακλά το φως του ήλιου και η εξήγηση της σεληνιακής έκλειψης η ανισότητα των χρόνων α­ νάμεσα στα ηλιοστάσια και τις ισημερίες κ.α. Όμως εκτός από το μαθηματικό έργο του ο Ε&. δημος είναι πολύ πιο γνωστός για τη συνεισφορά του στο να σωθεί το έργο του Αριστοτέλη. Έγραψε τα «Αναλυτικά)), σύγγραμμα λογικής, στο οποίο κατά την κρίση του Βοηθίου τελειοποιεί τη θεωρία των συλλογιστικών τρόπων και διατυ­ πώνει τους κανόνες του υποθετικού συλλογισμού, τα οποία ο Αριστοτέλης παραμέλησε στο «Όργα­ νΟ)) του. Το έργο του Αριστοτέλη «Ηθικά Ευδή­ μιω) αποδίδεται από πολλούς νεώτερους συγγρα­ φείς στον Εύδημο. Άλλα έργα του είναι: Το «περί λέξεωρ), αγνώστου περιεχομένου. Μια μελέτη για τα «Φυσικά)), διατριβή τεσσά­ ρων βιβλίων ομότιτλο αυτού του Αριστοτέλη. Ο Σιμπλίκιος είχε αντίγραφο αυτού του έργου το <r ποίο του φάνηκε πολύ χρήσιμο στην κατανόηση των «Φυσικών)) του Αριστοτέλη και ίσως αυτός ή­ ταν ο ρόλος που επεδίωξε ο Εύδημος να παίξει το έργο του αυτό.

Τέλος υπάρχει μια αναφορά που υπαινίσσεται ότι ο Εύδημος έγραψε μια Ιστορία της Θεολογίας κι αυτό φαίνεται πολύ πιθανό. Πολλοί συγγραφείς αναφέρονται στον Εύδημο ως ο «ευσεβής Εύδη­ μορ) εξ' αιτίας της πίστης του στην «παρατήρηση του Θεού)). Αυτό όμως μπορεί να οφείλεται σε μια μετέπειτα σχολιασμένη έκδοση από κάποιον χρι­ στιανό, ο οποίος πιθανόν θεώρησε ότι όταν ο Ε&. δημος έγραφε «παρατήρηση του νου)) εννοούσε «παρατήρηση του Θεού)) και το διόρθωσε αναλό­ γως. ΥΓ. Στην παρουσίαση αυτού του άρθρου συνει­ σέφερε η καθηγήτρ ι α Α γγλικών Δ έσποι να Κασαπί­ δου κάνοντας τη μετάφραση του άρθρου των Con ­ nor-Robertson, κατά τη δι άρκε ι α της συνυπηρέτη ­ σης μας στο ΤΕ.Ε. Παραδεισ ίου Ρόδου τη σχολι κή

χρονι ά 2002-2003 οπότε και γρά φτηκε το άρθρο αυτό. Πηγές, Βιβλιογραφία

1) Άρθρο των J.J.O'Connor and E.F. Robertson. Schoo1 ofMathematics and Statistics Uniνersity of St Andrews, Scot1and στην ηλεκτρονική διεύθυνση http://www.tmth.edu.gr/en/d/2/45.htm1 3) «Θέματα από την ιστορία των Μαθηματικών)) του Γιάννη Χριστιανίδη, Πανεπιστημιακές εκδόσεις Κρήτης 4) «Η αφύπνιση της επιστήμηρ) B.L. Van der Waerden, Πανεπιστημιακές εκδόσεις Κρήτης, 5) Εγκυκλοπαίδεια Παύλου Δρανδάκη 6) Ιστοσελίδα τεχνικού μουσείου Θεσσαλονίκης στη θέση http://www . tmth.edu.gr/en/d/2/45 .htm1 7) a. Τ .L. Heath, Α history of Greek mathematics Ι, Π (Oxford, 1 93 1 ). b. Α.Τ.Η. Fήtzsche, De Eudemi Rhodii phi1oso­ phi Perpatetici νita et scήptis (Regensburg, 1 85 1 ) c . W. Jaeger, Aήstote1es, Grund1egung einer Geschichte seiner Entwick1ung (Ber1in, 1 95 5). R. Robinson (trs.), Aήstot1e, Fundamenta1s of the History ofthis Deνe1�pment (Oxford, 1 948).

ΕΥΚΛΕΙΔΗΣ Β' λζ' τ.2/4


π

I

I

ΑΝΑΖΗΤΩΝΤΑΣ ΤΟ ΜΑΓΙΚΟ ΚΛΕΙΔΙ... (συνέχεια από το προηγούμενο τεύχος)

M.ed.

Β.Ε. Βισκαδουράκης Διδακτικής και Μεθοδολογίας των Μαθηματικών Πειρ. Λύκειο Ιωνιδείου Σχολής Πειραιά

Ας δούμε τώρα μια λύση του 4ου θέματος των Γi (του Γ ως προς κάποια σημεία). Μεγάλων από τον «Αρχιμήδη» (15 Φεβρ. 2003) υπό Είναι προφανές ότι όλα αυτά τα σημεία είναι με το φως των αναλλοίωτων, το οποίο έχει ως εξής: ακέραιες συντεταγμένες (κόμβοι στο πλέγμα των Στο σύνολο Σ των σημείων ενός επιπέδου (Π) � κάθετων ευθειών του σχήματος). Ας δούμε όμως τις ρ ίζουμε μια πράξη , δηλαδή μια απεικόνιση της συντεταγμένες των Ai, Bi, Γi συγκεκριμένα και ίσως μορφής: * : ΣχΣ � Σ που απεικονίζει κάθε ζεύγος κάποια παρατήρηση σ' αυτές να μας φωτίσει για κάποια γενικότερα συμπεράσματα. Έχουμε λοιπόν: σημείων (Χ, Ψ) σtο σημείο Ζ, (δηλαδή Χ *Ψ = Ζ) Α(Ο, 1), Α ι (2, 1 ), Α2 (2,- 1 ), Α3(Ο,- 1), �(4,-1 ), το οποίο είναι το συ μμετρικό του Χ ως προς το Ψ. Δίνεται τετράγω νο ΑΒΓΔ στο επίπεδο (Π) . Εί­ As(4,-3), Β(1 , 1 ), Β ι (- 1 , 1 ), Β 2 ( 1 ,-1), Β3(3,-1), Β 4( 1 ,ναι δυνατόν με διαδοχικές εφαρμογές της πράξης 3), Γ(1 ,0), Γ ι ( 1 ,2), Γ3 (3,-2), Γ 4(3,0), Γs(- 1 ,-2). Μπορούμε άραγε παρατηρώντας καλά όλα αυ­ ( ) ξεκινώντας με το σύνολο {Α,Β,Γ} και επεκτεί­ νοντας το με την πράξη αυτή, να πάρο υμε ως τά τα ζεύγη των συντεταγμένων να εντοπίσουμε κάποια αναλλοίωτη; εικόνα το Δ; ;------Γ·------� . τ --i -----;··-Ναι! Τα Ai έχουν όλα άρτια τετμημένη και πε­ ,_-+--,----+-- -+---+-�.4f--� ριττή τεταγμένη. , ; r2: . :rι ; • . �------r-··--r------r----r--. f . . .Τα Bi έχουν όλα περιττές και τις δύο συντεταγ­ ! Bi μένες τους. ir4 Τα Γi έχουν όλα περιττή τετμημένη και άρτια τεταγμένη. Είναι λογικό να αναρωτηθεί κανείς αν η παρα­ τήρηση αυτή είναι τυχαίο γεγονός (πράγμα πολύ δύσκολο) ή αν ισχύει γενικά. Λύση Τη γενικότητα (αν υπάρχει) θα πρέπει να την α­ Έστω το τετράγωνο ΑΒΓΔ πλευράς α. Χωρίς ναζητήσουμε στον τρόπο «παραγωγής» των νέων βλάβη της γενικότητας θεωρούμε ότι α 1 . Επίσης σημείων, δηλαδή στη συμμετρία. θεωρούμε ένα σύστημα συντεταγμένων με κέντρο Ας θεωρήσουμε δύο σημεία Μ(α1 ,β ι ) και Κ(κ,λ) το Δ(Ο,Ο), Α(Ο, 1 ), Β(1 , 1 ) και Γ(1 ,0) και μερικά με ακέραιες συντεταγμένες και το συμμετρικό συμμετρικά σημεία. Μ ' (α2 ,β2) του Μ ως προς το Κ. Είναι γνωστό τότε Ai (του Α ως προς κάποια σημεία) ότι αφού το Κ είναι μέσον του ΜΜ ' θα έχουμε: Bi (του Β ως προς κάποια σημεία) *

- - - ---------

:----�----t--

-�---

- --

---

=

ΕΥΚΛΕΙΔΗΣ Β ' λζ' τ.2/5


Το βήμα του Ευκλείδη

α1 + α2 = κ και β 1 +. β 2 . α ι +α2 = 2κ και = λ αρα 2 2 β ,+ β2= 2λ οπότε: α2 = 2κ + (-α1) και β 2 = 2λ + (-β 1 ) . Προσθέτοντας όμως στους ακεραίους (-α1) και (-β ι ) τους άρτιους 2κ και 7λ αντίστοιχα, θα πάρου­ με ακεραίους α2 και β2 που θα είναι της ίδιας κατηγορίας' με τους (-α 1 ), (-β 1 ) δηλ. με τους α1, β 1 • Έτσι όλα τα συμμετρικά του Α(Ο, 1), αλλά και τα συμμετρικά των συμμετρικών του Α ως προς ο­ ποιοδήποτε σημείο Κ(κ,λ) θα έχουν συντεταγμένες της μορφής (2ν,2μ+ 1) δηλαδή (άρτιος, περιττός). Τα συμμετρικά του Β(1 , 1 ) θα έχουν συντεταγ­ μένες της μορφής (περιττός, περιττός) και τα συμ­ μετρικά του Γ(1 ,0) θα έχουν συντεταγμένες της μορφής (περιττός, άρτιος). Μένουν δηλαδή οι συντεταγμένες των Α, Β, Γ κατά την απεικόνιση ( *) αναλλοίωτες ως προς την κατηγορία τους. Έτσι ποτέ δεν μπορούμε να πάρουμε ως εικόνα το Δ(Ο,Ο) του οποίου οι συν/νες είναι και οι δύο άρτιες, (το Ο είναι άρτιος αριθμός). Ύστερα από όσα μέχρι εδώ αναφέρθηκαν είναι πλέον φανερό ότι το κλειδί για το οποίο έγινε λό­ γος στην εισαγωγή έχει να κάνει με την έννοια της αναλλοίωτης. Ε λοιπόν ναι! Το «μαγικό κλειδί» για τη λύση όλων των προβλημάτων που προσπα­ θήσατε(;) να λύσετε είναι αυτή ακριβώς η έννοια. Ας δούμε όπως ποια είναι η αναλλοίωτη σε κα­ θένα πρόβλημα ξεχωριστά. Σχετικά με το 1" πρόβλημα θα ξέρετε ίσως ότι κάθε ζάρι έχει 6 έδρες και σε κάθε έδρα απεικονί­ ζεται ένας αριθμός από 1 μέχρι 6. Έτσι λοιπόν το άθροισμα των ενδείξεων όλων των εδρών ενός ζαριού είναι ίσο με 1 +2 + 3 +4+5 + 6 = 2 1 . Ο αριθμός 2 1 αποτελεί και την αναλλοίωτη του προβλήματος. Τα υπόλοιπα για σας ..... . Στο 2" πρόβλημα περιγράφεται μια διαδικασία που επαναλαμβάνεται σταθερά (και αναλλοίωτα).

Στην πρώτη της εκτέλεση έχουμε 3 κομμάτια χαρ­ τιού. Με τη δεύτερη προκύπτουν συνολικά 2+3 = 5. Με την τρίτη εκτέλεση της διαδικασίας πάλι ο συνολικός αριθμός είναι 4+3 = 7. Με την τέταρτη 6+ 3 = 9 κ.τ.λ. Η αναλλοίωτη προφανώς εδώ είναι ο αριθμός 2. Τόσα νέα κομ­ μάτια προκύπτουν κάθε φορά. Έτσι στην τυχαία νιοστή εκτέλεση της διαδικασίας θα έχουμε 3 + 2 + 2 + .... + 2 = 3 + 2(ν - 1) κομμάτια χαρτιού. Δηλαδή περιττό πλήθος πάντα. Έτσι είναι αδύνατο να πετύχουμε ακριβώς 2004 κομμάτια ..... Στο 3 " πρόβλημα παρατηρούμε ότι σε κάθε ε­ κτέλεση μετάγγισης θα προκύπτει ποσότητα που θα είναι πολλαπλάσιο του Μ.Κ.Δ. των αριθμών 4,6 και 20, δηλαδή πολλαπλάσιο του 2. Αυτός ο αριθ­ μός (δηλ. το 2) είναι η αναλλοίωτη που ζητάμε και προφανώς το 1 1 δεν είναι συμβατός αριθμός με την παρατήρηση αυτή. Αν σκεφτείτε την εικόνα ενός σκακιού (για το 4" πρόβλημα) θα παρατηρήσετε ότι ανά δύο τα απέναντι γωνιακά τετράγωνα έχουν ίδιο χρώμα. Τα δύο είναι μαύρα και τα άλλα δύο λευκά. Επίσης σε κάθε σκάκι έχουμε 32 λευκά και 32 μαύρα τετρά­ γωνα. Αφαιρώντας λοιπόν δύο απέναντι γωνιακά τετράγωνα, τα υπόλοιπα είναι 32 και 30 από κάθε χρώμα, συνολικά 62 τετράγωνα. Για να καλύψου­ με 62 τετράγωνα χρειαζόμαστε 3 1 ντόμινο. Όμως κάθε ντόμινο έχει 1 λευκό και ένα μαύρο τετράγοr νο (το γεγονός αυτό αποτελεί την αναλλοίωτη του προβλήματος) κ.τ.λ. κ.τ.λ. ..... . Για το 5" πρόβλημα ο πειραματισμός με μικρό­ τερων διαστάσεων «σκάκυ) ενδεχομένως να μας αποκαλύψει το «μυστικό))

mς. 'Ετσι για ένα σκάκι 2χ2 m το «αδύνατΟ))

a

είναι προφανές. Επίσης για ένα σκάκι 3χ3 το «δυνατό)) επίσης είναι προφανές. Πειραματιζόμενοι με σκάκι 4χ4 θα βρεθείτε πάλι σε αδιέξοδο, ενώ με σκάκι 5χ5 πάλι το «δυ­ νατό)) είναι άμεσα προφανές. ·

1 Τους ακεραίους τους διακρίνουμε σε δύο μεγάλες κατηγοιf­ ες: τους άρτιους και τους 'περιττούς. Στη διεθνή βιβλιογραφία χρησιμοποιείται ο όρος paritte ή paήty για τη διάκριση αυτή.

ΕΥΚΛΕΙΔΗΣ Β' λζ' τ.2/6


Το βήμα του Ευκλείδη

Γιατί όμως άραγε απ' ότι φαίνεται σε σκάκι με διαστάσεις άρτιες το πρόβλημα είναι αδύνατο; Ποια τέλος πάντων είναι η αναλλοίωτη στη διαδι­ κασία που περιγράφει το πρόβλημά μας; Θα συμφωνείτε ασφαλώς ότι η συνεχής μον� κοντυλιά και η μοναδική διέλευση από κάθε τε­ τράγωνο είναι αυστηροί περιορισμοί και σ' αυτούς θα πρέπει να αναζητήσουμε την αναλλοίωτη. Είναι φανερό ότι η μετακίνηση από τετράγωνο σε τετρά­ γωνο σημαίνει και αλλαγή χρώματος (να η αναλ­ λοίωτη !), αφού δεν υπάρχουν διαδοχικά τετράγω­ να με ίδιο χρώμα. Ξεκινώντας λοιπόν από μια γω­ νία (έστω μαύρου χρώματος) η οποιαδήποτε δια­ δρομή θα ακολουθεί σταθερά την εναλλαγή μ-λ-μ­ λ-μ-λ... , οπότε αφού τα τετράγωνα είναι άρτιου πλήθους (8χ8 64) το τελευταίο θα πρέπει να εί­ ναι λευκό, πράγμα που δεν ισχύει αφού τα απένα­ ντι γωνιακά τετράγωνα είναι του ίδιου χρώματος. Στο 6° π ρόβλημα αν προσθέσουμε τον αριθμό των χειραψιών που έχουν κάνει όλα τα άτομα το αποτέλεσμα θα είναι άρτιος αριθμός αφού την κάθε χειραψία την έχουμε καταμετρήσει δύο φ� ρές. Αυτός ο (άρτιος) αριθμός είναι και η αναλ­ λοίωτη του προβλήματος. Έστω τώρα ότι χωρί­ ζουμε τα άτομα σε δύο ομάδες. Την ομάδα α 1 ,α2 , ... ,ακ που όλοι τους έχουν κάνει άρτιο αριθμό χειραψιών και την ομάδα Πι ,Π2 ,Π3, ... πλ που όλοι τους έχουν κάνει περιττό αριθμό χειραψιών. Το άθροισμα όλων των χειραψιών τότε θα είναι: (Χα, + Χα2 + ... + Χακ ) + (Χπ + Χπ + ... + Χπ ) 2 άρτιος + περιττός =: περιττός, πράγμα άτοπο. Άρα τα άτομα με περιττό αριθμό χειραψιών εί­ ναι οπωσδήποτε άρτιου πλήθους. Για το 7° πρόβλημα είναι φανερό ότι για να εί­ ναι το γινόμενο Ρ = (α, - 1)(α2 - 2) .... (α" -η ) άρτιος, αρκεί ένας τουλάχιστον παράγοντας να είναι άρτιος. Αυτό θα ισχύει αν δείξουμε ότι κάποια διαφορά ακ - κ είναι διαφορά δύο αριθμών της ί­ διας κατηγορίας (ή και οι δύο άρτιοι ή και οι δύο περιττοί). Αυτό πραγματικά ισχύει γιατί αναδια­ τάσσοντας τους αριθμούς 1 ,2,3, ... ,η το πλήθος =

άρτιος

=

I

2

λ

περιττός αν λ= ν+ Ι

=

των αρτίων και των περιττών, προφανώς δεν αλλάζει, (παραμένει αναλλοίωτο).

Όσοι είναι λοιπόν οι άρτιοι στους 1 ,2,3,... ,11 τό­ σοι είναι και στους αι ,α2 ,α3 , .... ,αη. Έτσι αν υποθέ­ σουμε ότι όλοι οι παράγοντες του γινομένου Ρ εί­ ναι περιττοί θα πρέπει οι α ι ,α3 ,αs,α7, .. .,αη να είναι όλοι άρτιοι, πράγμα άτοπο αφού οι άρτιοι είναι ένας λιγότ.εροι από τους περιττούς 1 ,3,5,7, . .. ,η. Μία άλλη αντιμετώπιση του ίδιου προβλήματος βασισμένη και αυτή στην ιδέα της αναλλοίωτης είναι η εξής: Αν ο Ρ ήταν περιττός θα έπρεπε όλοι οι η παρά­ γοντες του να είναι περιττοί. Όμως αφού και το πλήθος η είναι περιττός, θα έπρεπε το άθροισμα S = (α 1 - 1) + (α2 - 2) + .... + (α" -η ) να είναι περιττο τός, δηλαδή S = (α, + α 2 + .... + α" ) - (1 + 2 + ... + η ) να είναι περιττός. Αυτό είναι άτοπο γιατί S Ο μιας και οποιαδήποτε αναδιάταξη των 1 ,2,3,...,η αφήνει το =

άθροισμα τους αναλλοίωτο.

Έτσι δεν μπορεί όλοι οι παράγοντες του Ρ να είναι περιττοί, οπότε τουλάχιστον ένας θα είναι άρτιος και κατά συνέπεια και το Ρ θα είναι άρτιος. Για το πρόβλημα 8, όπως και σε αρκετά άλλα προβλήματα, ο πειραματισμός είναι καθοριστικής σημασίας και δυστυχώς από πολλούς υποτιμάται ή τουλάχιστον δεν ενθαρρύνεται. Κι αυτό παρά το γεγονός ότι στις μέρες μας τα Πειραματικά Μαθη­ ματικά τείνουν να αποτελέσουν ξεχωριστό κλάδο των Μαθηματικών. Εξάλλου ολόκληρο το δέντρο των Μαθηματικών έχει τις ρίζες του στέρεα και βαθιά χωμένες στο έδαφος του πειραματισμού. Ας μην ξεχνάμε ότι ο Μέγιστος των μαθηματικών, ο Αρχιμήδης ήταν ένας, (με τη σημερινή έννοια του όρου), πειραματικός μαθηματικός. «Πολλές πεποιθήσεις αρχικά μου δημιουργού ­ νται με κάποια μηχανική μέθοδο. Έστω κι αν αυτές πρέπει να αποδειχθούν με Γεωμετρία στη συνέχεια, καθότι η ανακάλυ ψη τους με τη μηχανική μέθοδο δε συνιστά μια αποδεκτή απόδειξη, είναι φυσικά ευκ� λότερο, όταν έχουμε προηγουμένως συμπεράν,ει κά­ ποια απάντηση, μ ' αυτή τη μέθοδο στο ερώτημά μας, να κάνουμε την απόδειξη που θέλουμε παρά να

ΕΥΚΛΕΙΔΗΣ Β' λζ' τ.2Π


Το βήμα του Ευκλείδη

πετύχουμε κάτι τέτοιο χωρ ίς κα μία προηγούμενη ένδειξη και γνώση για την απάντηση» .

Ναι! σωστά μαντέψατε. Είναι λόγια του Αρχι­ μήδη, γι' αυτό και παρακαταθήκη αναλλοίωτης στο χρόνο σημασίας . . . . Ας φέρουμε τώρα κάτω από το φως αυτής της παρακαταθήκης το πρόβλημα 8. Παίρνουμε ένα τυχαίο ζεύγος αριθμών, έστω το (3,24), το διαγράφουμε και στη συνέχει συμπλη­ ρώνουμε με τον αριθμό 1 24 - 3 1 = 1 2 1 1 = 2 1 το σύνολο των υπολοίπων αριθμών. Επαναλαμβάνουμε την ίδια διαδικασία για με­ ρικά ακόμα ζεύγη και την καταγράφουμε σ' ένα πίνακα: Ζεύγος αριθμώ ν που διαγράφεται

3, 24) 5, 10) 7, 39) (1575, 359) 2002, 102) 1821, ?004) (1940, 2003)

Αριθμός που συμπληρώνουμε

21 5 32 1216

1900 183 63

Συνολικό προκύπτον άθροισμα

�005003-(3+24)+21 200499 2004997-(5+10+5 2004987 2004987-(7+39)+32 2004973 �004973-(1575+359)+1216 1200425 �004255-(2002+102)+1900 12004051 �004051-(1821+2004)+183 12000409 12000409-(1940+2003)+63 1996529 =

=

=

=

=

=

=

Επειδή (όπως θα συμφωνείτε) η διαγραφή και η συμπλήρωση παραπέμπουν άμεσα στην αφαίρεση και την πρόσθεση, είναι λογικό να θεωρήσουμε το συνολικό άθροισμα των αριθμών 1 + 2 + 3 + ... + 2000 + 20 01 + 200 2 = 1 00 1 φορές το 2003 = 2.005.003 (θυμηθείτε τον Gauss στην Α ' δημοτικού) και να παρατηρήσουμε πως μεταβάλλεται από κάθε φορά εκτέλεσης της διαδικασίας. Σίγουρα θα έχετε ήδη παρατηρήσει κάτι που παραμένει σταθερό στην τελευταία στήλη. Όλοι οι αριθμοί που προκύπτουν είναι περιττοί. Αυτό α­ ποτελεί και ·την αναλλοίωτη του προβλήματος. Πραγματικά εφαρμόζοντας στο αρχικό άθροισμα 2.005.003 τις αλλαγές που υπαγορεύει το πρόβλη-

μα και για ένα τυχαίο ζεύγος (α, β), η μεταβολή (μείωση) που θα υποστεί είναι ίση με � - β - α - β = -2β για α>β Ι α - βΙ - α - β = η β - α - α - β = -2α για α < β δηλαδή έτσι κι αλλιώς άρτια. Όταν όμως έχουμε ένα περιττό και τον μειώνουμε συνέχεια κατά ά� τιο το αποτέλεσμα θα είναι πάντα περιττός. Έτσι φτάνοντας στο προτελευταίο βήμα και έχοντας τους αριθμούς κ,λ θα είναι κ+λ περιττός 2ν+ 1 . Εφαρμόζοντας τη διαδικασία για το ζεύγος (κ,λ) θα μείνει τελικά ο αριθμός 2ν + 1 - 2κ ή ο αριθμός 2ν + 1 - 2λ δηλαδή ο 2(ν - κ) + 1 ή ο 2(ν - λ) + 1 που προφανώς είναι σε κάθε περίπτωση περιττός. Για τα προβλήματα 9 και I Ο θυμηθείτε το κριτήριο διαιρετότητας με το 3 και παρατηρείστε ότι στους η, σ(η), σ(σ(η)) το υπόλοιπο της διαίρε­ σης τους με το 3 (έστω υ) είναι το ίδιο, (παραμένει αναλλοίωτο). Τότε όμως αν η = 3κ + υ, σ(η) = 3λ + υ, σ(σ(η)) = 3μ + υ θα έχουμε η + σ(η) + σ(σ(η)) = 3ρ = 2003 άτοπο, αφού το 3 δεν διαιρεί το 2003 . Επίσης για το 1 Οο παρατηρείστε ότι 1 + 2 + 3 + .... + 9 = 45 = 3 · 1 5 . Έτσι αν χρησιμο­ ποιώντας τα ίδια αυτά ψηφία και το καθένα μια φορά, φτιάξουμε μερικούς αριθμούς (διψήφιους και μονοψήφιους) και αυτοί θα έχουν άθροισμα πολλαπλάσιο του 3 κάτι που δεν συμβαίνει με το 100. Για παράδειγμα: 2 + 3 + 7 + 5 = 1 7 = 3 · 5 + 2 επίσης 23 + 75 = 98 = 96 + 1 = 3 · 32 + 2 . Το πρόβλημα Ι Ι είναι ένας κλασικός γεωμε­ τρικός τόπος που μπορεί να απαντηθεί (και να κα­ τανοηθεί πλήρως) μέσα από την αναζήτηση κά­ ποιας αναλλοίωτης που θα σχετίζεται με το μέσον

[

=

=

Σ�

Μ του ΣΑ. Καθώς το Μ προσδιορίζεται άμεσα και μονοσήμαντα από το Α, είναι λογικό αν υπάρχει αναλλοίωτη που σχετίζεται με το Μ, θα πρέπει να υπάρχει κάποια αντίστοιχη αναλλοίωτη που να

ΕΥΚΛΕΙΔΗΣ Β' λζ' τ.2/8


Το βήμα του Ευκλείδη

35 =7 · 5 30=6 · 5 σχετίζεται με το Α. Πραγματικά το σημείο Α κινεί­ . . 3 1 =3 5 + 2 8 36=4 . 5 + 2 . 8 ται πάνω στον κύκλο (O,R), άρα η απόσταση του 37=5 + 4 · 8 32=4 · 8 από το Ο παραμένει αναλλοίωτη και ίση με R. Φέρνουμε το ΣΟ που είναι σταθερό τμήμα και θε­ Ο Μπάμπης Μανουσιάδης (μαθητής Β' γυμνα­ ωρούμε το μέσον του Κ που είναι επίσης σταθερό σίου) παρατήρησε ότι δεν χρειάζεται ούτε μία επι­ σημείο. Στο τρίγωνο τώρα ΣΟΑ τα Κ, Μ είναι μέ- πλέον επαλήθευση για να συμπεράνουμε το ζη­ , πλευ ρων , του αρα , ΚΜ ΟΑ R =τ=2" τούμενο. Κι αυτό γιατί οι αριθμοί 28,29, ... ,37 α­ σα των δυο ντιπροσωπεύουν ως προς το τελευταίο ψηφίο κάθε Παρατηρούμε έτσι ότι η απόσταση των σημείων άλλο ακέραιο. Η δεκάδα δηλαδή των ψηφίων Μ από το σταθερό σημείο Κ παραμένει σταθερή Ο, 1 ,2, ... ,9 επαναλαμβάνεται αναλλοίωτα, άρα .... (αναλλοίωτη), άρα το Μ θα ανήκει σε κύκλο κέπαράδειγμα: Για κ , R , , . ντρου και ακτινας 2" που ειναι τελικα και ο 2003 =33 + 1 970= 5 5 + 8 + 394 · 5 = 299 · 5 + 8. Τα προβλήματα 13 και 14 αποφεύγουμε να τα ζητούμενος γεωμετρικός τόπος. συζητήσουμε ώστε η χαρά της λύσης τους να είναι Για το πρόβλημα 1 2 ξεκινάμε πειραματικά εξ ολοκλήρου δική σας. Όσοι πάντως γευτείτε αυ­ (μην ξεχνάτε τα λόγια του Αρχιμήδη) και έχουμε: τή τη χαρά μπορείτε να τη μοιραστείτε αν θέλετε 28=4 · 5 + 8 33=5 · 5 + 8 με τον Ευκλείδη Β' και όλους τους αναγνώστες 29 =5 + 3 · 8 34= 2 · 5 + 3 · 8 του στέλνοντας μας τη λύση σας.

Η χαρά « Τα μαθηματι κά ασφαλώς δεν θα είχαν εμφα­ νιστεί, αν ή ταν γνωστό από την αρχή ό τι δεν υπάρ­ χει ακριβώς ευθεία γραμμή στη φύση, ούτε πραγμα­ τι κός κύκλος, ούτε απόλυτη μονάδα μέτρησης» ι­

σχυρίζεται ο Νίτσε. Ενώ ο Φ.Τζ.Ντέιβις σημειώνει πως "το μυστηριώδες και θαυμαστό π, ξεπέφτει σε γαργάρα που βοηθά τους υπολογιστές να καθαρί­ ζουν το λαιμό τους". Στο βιβλίο του Ντέιβιντ Μπλάντερ (ΝΜ) 'Ή χαρά του π", ο αναγνώστης θα συναντήσει αρκετές αναφορές επιφανών ανδρών για τα μαθηματικά, τους υπολογιστές που βασίζονται σ' αυτά, και τον κύκλο, όπως οι προηγούμενες. Ακόμα θα βρει πολλούς , αφορισμούς και ατάκες, και γι' αυτό η ανάγνωσή του πιστεύουμε ότι θα αποτελέσει μια ευχάριστη ενασχόληση. Το βιβλίο, που προαναφέραμε, είναι μια ε-

του π Γιάννης Σταμέλος

κλαϊκευτική μονογραφία για το ευρύτατα αναγν� ρισμένο ελληνικό γράμμα -εκτός βέβαια της πανε­ πιστημιακής κοινότητας- το οποίο μπορεί να συ­ ναντήσει κανείς όχι μόνο στα εγχειρίδια μαθημα­ τικών και στατιστικής, αλλά και στα αντίστοιχα φυσικής και αστρονομίας. Το ελληνικό αυτό γράμμα, με το οποίο συμ­ βολίζουμε το λόγο του μήκους τού κύκλου προς τη διάμετρό του - γνωστό τόσο στους μαθητές του δημοτικού όσο και στους πολυμαθείς καθηγητές πανεπιστημίου - είναι συνυφασμένο με πάμπολλες ιστορίες μυστηρίου, ρομαντισμού, αντιπαραθέσε­ ων, υπερβολών αλλά και γραφικών περιστατικών και καταστάσεων. Ακόμα και στη σύγχρονη παγκόσμια λογοτε­ χνία μπορεί κανείς να βρει επιδράσεις. Στο εκκρε­ μές του Φουκώ, για παράδειγμα, ο Ουμπέρτο Έκο,

ΕΥΚΛΕΙΔΗΣ Β' λζ' τ.2/9


Το βήμα του Ευκλείδη

γράφει σχετικά:

'Έγώ ήξερα [...] ότι η περίοδος

[του εκκρεμούς] ρυθμιζόταν από τη σχέση ανάμεσα στην τετραγωνική ρίζα του μήκους του νήματος, και σ' εκείνο τον αριθμό π, πού με τρόπο παράλογο για τις γήινες διάνοιες, συνδέει αναπότρεπτα, χάρη στη θεία φρόνηση, την περιφέρεια με τη διάμετρο όλων των δυνατών κύκλων, ώστε ο χρόνος του ταξιδιού [η περίοδος του εκκρεμούς]

από τον ένα πόλο στον

άλλο να είναι αποτέλεσμα μυστηριώδους συνωμοσί­ ας των πιο άχρονων μέτρων, της μοναδικότητας του σημείου εξάρτησης [λόγω της σταθεράς της βαρύτη­ τας g του τόπου], της δυαδικότητας της μιας αφηρη­

μένης διάστασης, της τριαδικής φύσης του π, του μυ­ στικού τετραγώνου της ρίζας, της τελειότητας του

διερμηνεύοντας τη σχέση που εκφράζει ο τύπος της περιόδου του εκκρεμούς Τ 2π κύκλου"

=

Η.

Ο υπολογισμός του π αφορά το μαθηματικό πρόβλημα του υπολογισμού του λόγου του κύκλου προς την διάμετρο, όπως προαναφέραμε, και η τιμή του είναι η ίδια, σε οποιοδήποτε μέρος του σύμπαντος και για οποιονδήποτε κύκλο χρησιμο­ ποιήσουμε, όπως γνωρίζουμε ακόμα και από τα στοιχειώδη μαθηματικά των σχολείων της Μέσης Εκπαίδευσης. Η αρχαιότερη γνωστή καταγραφή για την τιμή του π υπάρχει στον πάπυρο Ριντ και έγινε από τον Αιγύπτιο γραφέα Αχμές γύρω στα 1 650 π.Χ., είναι δε 3,1 6049. Παρόλα αυτά δεν υπάρχουν στοιχεία που να πιστοποιούν ότι οι Αιγύπτιοι γνώριζαν πως το π είναι σταθερά. Όσο για τον συμβολισμό του με π, αυτός είναι του 1 706. Τότε χρησιμοποίησε πρώτος ο Ουίλιαμ Τζόουνς το π, για να συμβολίσει τον σταθερό αυτό λόγο. Όμως αυτός που τον κα­ θιέρωσε ήταν ο Όιλερ, με το τεράστιο κύρος του. Στο βιβλίο του ΝΜ υπάρχει μια αρκετά εκτε­ νής αναφορά σε επιστήμονες και μεθόδους που αναπτύχθηκαν, για να προσδιορίσουν ακριβέστερα την τιμή του π. Το π είναι άρρητος, δηλαδή δεν μπορεί να γραφεί σαν κλάσμα, ακεραίων αριθμών με παρα-

νομαστή διαφορετικό απ' το μηδέν · δεν είναι όμως αλγεβρικός (υπάρχουν άπειροι άρρητοι αλγεβρικοί πχ το .J2 ), είναι υπερβατικός δηλαδή δεν υπάρ­ χει πολυωνημική εξίσωση με ακεραίους συντελε­ στές, της οποίας να είναι ρίζα. Όπως ακόμα και οι μαθητές του Γυμνασίου γνωρίζουν, η γραφή των άρρητων ως δεκαδικών οδηγεί σε μη τερματιζόμε­ νο αριθμό και χωρίς περιοδικότητα ψηφίων. Οι Αναξαγόρας, Αρχιμήδης, Κλαύδιος Πτο­ λεμαίος, Αλ Κβαράσμι, Φιμπονάτσι, Βιέτ, Σνελ, Ουόλις, Γκρέγκορι, Λάιμπνιτς, Νεύτωνας, Όιλερ, είναι μερικοί από τις μεγάλες μορφές που ασχολή­ θηκαν με το π. Αναφορές στο π μπορεί να βρει κανείς στη Βίβλο, η τιμή που αναφέρεται σ' αυτήν είναι 3, αλλά και στις αναλογίες της Μεγάλης Πυραμίδας της Γκύζας: Ο λόγος του μήκους μιας πλευράς προς το ύψος της είναι περίπου π/2. «Για αιώνες αιγυπτιολόγοι και οπαδοί του μυ­ στικισμού, διατύπωναν υποθέσεις σχετικές με τη σημασία και την αιτία αυτής της σχέσης. [ . . . ] Ω­ στόσο ο Ηρόδοτος έγραψε πως η πυραμίδα κατα­ σκευάστηκε με τέτοιο τρόπο, ώστε το εμβαδόν κάθε έδρας να ισούται με το εμβαδόν τετραγώνου, που έχει πλευρά ίση με το μήκος του ύψος της πυ­ ραμίδας. Μπορεί να αποδειχθεί ότι οποιαδήποτε πυραμίδα με αυτόν τον σχεδιασμό προσεγγίζει αυ­ τομάτως το π» σημειώνει ο ΝΜ. Τους Έλληνες του 4ου π.Χ. αιώνα που εξέτα­ σαν το πρόβλημα, δεν τους ενδιέφερε τόσο η μέ­ τρηση εκτάσεων, όσο η διερεύνηση των ιδεών. Την περίοδο εκείνη, και για πρώτη φορά στην ι­ στορία, οι άνθρωποι σταμάτησαν να ρωτούν "πό­ σο;" και άρχισαν να ρωτούν «πως και γιατί;». Ή­ ταν η χρυσή εποχή του στοχασμού, κι ενώ ο λόγος του κύκλου προς τη διάμετρο δεν ήταν από τα ση­ μαντικότερα ζητήματα της εποχής, εντούτοις για μερικούς από τους μεγαλύτερους μαθηματικούς της αρχαιότητας υπήρξε αντικείμενο έρευνας. Ο Αρχιμήδης (287-2 12 π.Χ.) χρησιμοποίησε τη μέθοδο της εξάντλησης, προσεγγίζοντας το μήκος

ΕΥΚΛΕΙΔΗΣ Β ' λζ' τ.2/10


Το βήμα του Ευκλείδη

του κύκλου εγγράφοντας και περιγράφοντας σ' αυ­ τόν κανονικά πολύγωνα, των οποίων διπλασίαζε τον αριθμό των πλευρών. Υπολόγιζε στη συνέχεια τις περιμέτρους των και, μ' αυτόν τον τρόπο, μια αρκετά καλή προσέγγιση του μήκους της περιφέ­ ρειας και του π. Θα πρέπει, εδώ, να επισημάνουμε ότι μέχρι τον Αλ Κβαράσμι ((_/ς μ.Χ. αι) δεν χρησι­ μοποιούνται οι γνωστοί μας Αραβικοί αριθμοί, το μηδέν και η υποδιαστολή για τα δεκαδικά ψηφία. Το 1 579 ο Βιέτ χρησιμοποιεί έναν τύπο που δί­ νει τη τιμή του π με απειρογινόμενο. Τον 1 γ αιώνα ο Ουόλις, χωρίς να χρησιμοποιεί τον απειροστικό λογισμό, επίσης βρίσκει ένα τύπο ανάλογο με εκεί­ νο του Βιέτ, αλλά σαφώς πιο εύχρηστο. Ανάλογη είναι και η συμβολή των Γκρέγκορι, Λάιμπνιτς και Νεύτωνα. «Ήταν ολοφάνερο ότι ο υπολογισμός του λόγου του κύκλου προς τη διάμετρο δεν ήταν πλέον ζήτημα απλών μαθηματικών πράξεων». Στο βιβλίο του ΝΜ επιχειρείται να δοθεί μια απάντηση στο ερώτημα: Γιατί οι άνθρωποι να α­ σχοληθούν με τον ακριβή προσδιορισμό της τιμής του π, αφού ένας μηχανικός χρειάζεται στην πιο ακραία περίπτωση 7 δεκαδικά ψηφία, ενώ για να προσδιορίσουμε με ακρίβεια μερικών εκατοστών την περιφέρεια ολόκληρου του ορατού σύμπαντος απαιτούνται 30 δεκαδικά ψηφία; Ο ΝΜ ισχυρίζεται συν τοις άλλοις στο βιβλίο του ότι "η αναζήτηση του π είναι βαθιά ριζωμένη στο ερευνητικό πνεύμα του ανθρώπου [ . . . ] και στην ασυγκράτητη τάση μας να δοκιμάζουμε τα όριά μας. Όπως και με το Έβερεστ: οι άνθρωποι ανεβαίνουν εκεί πάνω, απλώς και μόνο επειδή υ­ πάρχει". Με την εξέλιξη των υπολογιστών, από το τε­ λευταίο μισό του 20ου αιώνα και εντεύθεν, ο υπο­ λογισμός των ψηφίων του π εξελίχθηκε σ' έναν ανταγωνισμό των εταιρειών κατασκευής υπολογι­ στών στην κατάκτηση της πρώτης θέσης. Στα πολλά αξιοσημείωτα που προκύπτουν από τον υπολογισμό των ψηφίων του π, σημειώνουμε τον τυχαίο τρόπο, χωρίς κανόνα, που εμφανίζονται

τα ψηφία του και το ότι οι αλγόριθμοι που δοκιμά­ ζονται στους υπολογιστές για τον προσδιορισμό των ψηφίων του, αποτελούν το ιδανικότερο τεστ για την καλή λειτουργία του υλικού τους και του λογισμικού. Έτσι, πέρα από τις άλλες γνωστές χρήσεις του π, προστίθεται και εκείνη για τους υπολογιστές οι οποίοι κυριαρχούν σήμερα στη ζωή μας, επιβεβαι­ ώνοντας για άλλη μια φορά ότι θεωρητικά θέματα μπορούν να έχουν εφαρμογές πολύ πρακτικές. Ένα άλλο θέμα, που σχετίζεται με το π και το οποίο πραγματεύεται εκτενώς ο ΝΜ στο βιβλίο του, είναι εκείνο του τετραγωνισμού του κύκλου. «Η ιστορία είναι γεμάτη από προσπάθειες με­ ταβολής ενός στοιχείου της φύσης σε κάποιο άλλο, με τη βοήθεια της αλχημείας, της μαγείας ή της επιστήμης». Ο τετραγωνισμός του κύκλου είναι από τους λίγους μαθηματικούς γρίφους, που έγινε ευρύτερα γνωστός εκτός των ορίων της μαθηματι­ κής κοινότητας. Το πρόβλημα είναι να κατασκευ­ αστεί ένα τετράγωνο μόνο με κανόνα και διαβήτη και πεπερασμένο αριθμό βημάτων, το οποίο να έχει το ίδιο εμβαδόν μ' έναν δοσμένο κύκλο. Η φράση «τετραγωνίζω τον κύκλο» στην κα­ θημερινή γλώσσα εκφράζει ένα σχέδιο καταδικα­ σμένο σε αποτυχία. Ο Αριστοφάνης στους Όρνιθες (414 π.Χ.) σατιρίζοντας το μάταιο του επιχειρήμα­ τος γράφει: "Με έναν ίmο χάρακα έπιασα δουλειά. Να κάνω τον κύκλο με τέσσερις γωνίες". Ο Δάντης επίσης στη Θεία Κωμωδία: '2'αν τον γεωμέτρη που παλεύει και δεν τα καταφέρνει τον κύκλο να μετρή­ σει, και για να βρει τη λύση στήριγμα γυρεύει, έτσι κι εγώ τούτο το πρωτόφαντο θέαμα είχα αντικρίσει".

Το πρόβλημα του τετραγωνισμού του κύκλου αναφέρεται και στον πάπυρο Ριντ, αλλά μόνο μετά τον 1 6° αιώνα οι μαθηματικοί συνειδητοποίησαν ότι κάθε προσπάθεια είναι μάταιη. "Εδώ ίσως ν' αρχίζει και η ρήξη ανάμεσα στους γνήσιους μαθη­ ματικούς και τους «τετραγωνιστές του κύκλου» ". Το 1775 είναι γνωστό ότι η Γαλλική Ακαδημία Επιστημών αποφάσισε να μην δέχεται λύσεις για τον

ΕΥΚΛΕΙΔΗΣ Β' λζ' τ.2/1 1


Το βήμα του Ευκλείδη

τετραγωνισμό του κύκλου, την τριχοτόμηση της γω­ νίας, τον διπλασιασμό του κύβου και το αεικίνητο. Επειδή, εκ πρώτης όψεως, δημιουργείται η ε­ ντύπωση ότι πρέπει να υπάρχει λύση, πάρα πολλοί ήσαν εκείνοι που πίστεψαν στις «καλές τους ιδέες» και την ακάματη φιλοπονία τους, αλλά αντί στην αναγνώριση οδηγήθηκαν στην γελοιοποίηση ! Μεταξύ αυτών και αρκετοί επώνυμοι, όπως ο Χόμπς και η βουλή της Ιντιάνα η οποία ψήφισε, παμψηφεί, νομοσχέδιο, που αναγνώριζε σε κά­ ποιον γιατρό το δικαίωμα της διαχείρισης της λl}. σης του τετραγωνισμού του κύκλου. «0 αγώνας δρόμου των τετραγωνιστών του κύκλου που αψηφούν τις ετυμηγορίες των μαθη­ ματικών [ . . . ] δεν θα τελειώσει ποτέ, όσο παραμέ­ νουν ενωμένες η άγνοια και η δίψα για δόξω) ση­ μείωνε ο Χ.Σούμπερτ (1 898). Ενώ και σήμερα α­ κόμα διαβάζουμε κατά καιρούς στον τύπο για κά­ ποιον σε κάποιο μέρος της Γης, ισχυρίζεται πως τετραγώνισε επιτέλους τον κύκλο! ! ! Στη μαθηματική κοινότητα είναι γνωστό, από το 1 882 που ο Λίντεμαν το απέδειξε, ότι το πρό­ βλημα δεν επιδέχεται λύση με τις προϋποθέσεις που έχουν τεθεί. Το π, λόγω του ότι τα ψηφία είναι άπειρα και επειδή παρατίθενται χωρίς κανόνα, αποτελεί την καλύτερη πρόκληση για τους aριθμομνήμονες. Το 1 995 ο 23-χρονος Χιρογιούκι Γκότο aπήγγειλε από μνήμης 42.000 ψηφία για περίπου εννέα ώρες. Ανάμεσα στις τεχνικές aπομνημόνευσης των αρχικών ψηφίων του είναι και εκείνη με τον αριθ­ μό των γραμμάτων στις λέξεις στιχουργημάτων που δημιουργούνται γι' αυτό το σκοπό. Στα ελλη­ νικά είναι γνωστό το τετράστιχο που δημιούργησε ο Καθηγητής Ν. Χατζηδάκης: «Αεί ο Θεός ο Μέ­ γας γεωμετρεί . . . )) , για το 3,14159 . . . Στο βιβλίο κατατίθεται και η άλλη άποψη για το μάταιο όλων αυτών των επιχειρημάτων με το σχόλιο του Άντεργουντ Ντάνλεϊ: «Ας ευχαριστοl}. με μεγαλόφωνα και καθημερινά τον Θεό, που δεν είμαστε τρελοί να αφιερώνουμε όλη μας την ενερ-

γητικότητα για να πείθουμε τον κόσμο ότι το π ισούται με 3,124, 22/7 ή ρίζα του 10 ακριβώς. Μπορεί να υπάρχουν μικρές πιθανότητες να εί­ ναι η ζωή μας μεστή από νόημα, γεμάτη πλούτη και ευτυχία, είναι όμως μεγαλύτερες από τη σχεδόν μη­ δενική πιθανότητα να τετραγωνίσουμε τον κύκλο)). Ο συγγραφέας στον επίλογο κλείνει το βιβλίο γράφοντας: «Όταν κοιτάζουμε στον καθρέπτη [ . . . ] η έγχρωμη ίρις [των ματιών μας] σχηματίζει ένα κυκλικό παράθυρο όπου μπορούμε να δούμε την ψυχή μας. Ίσως αυτή η κυκλικότητα να ώθησε τους πρώτους ανθρώπους να ατενίσουν με δέος τον κύκλο και, αργότερα, να τον μετρήσουν, κι ίσως μας οδηγήσει βαθύτερα στην έρευνα του Π)). Ο ΝΜ, σημειώνεται στο βιβλίο, έχει γράψει βιβλία για θέματα ψηφιακής εικόνας και εικονικής πραγματικότητας. Το βιβλίο «Η χαρά του Π)) στα ελληνικά κυ­ κλοφορεί από τις εκδόσεις Ωκεανίδα, έκδοση 200 1 , σε μετάφραση του κ Γαβριήλ Μοσχόπου­ λου. Το σχήμα του είναι 16,5χ16,5, αποτελείται από 1 50 σελίδεs, οι οποίες δεν καταλάβαμε γιατί δεν έχουν αρίθμηση. Το μισό σχεδόν της κάθε σε­ λίδας έχει ως διάκοσμο τα ψηφία του π ως το εκα­ τομμυριοστό ψηφίο του, που όμως δεν είναι ευκρι­ νή. Το εξώφυλλό του επίσης, είναι μια σύνθεση με κεντρικό σύμβολο το π. Αν εξαιρέσουμε μερικά σημεία που αφορούν τα απειρογινόμενα, το βιβλίο διαβάζεται άνετα και ευχάριστα απ' όποιον έχέι βασική μαθηματική παι­ δεία δευτεροβάθμιας εκπαίδευσης, και λόγω μεγέ­ θους «μονορούφυ). Θα πρέπει ακόμα να επισημάνουμε ότι στον τύπο του Γκρέγκορι αντί τοξεφ θα πρέπει να γρα­ φεί τοξεφχ. Στα θετικά του βιβλίου θα πρέπει να περιλάβουμε και τον πολύ εύχρηστο αλφαβητικό κατάλογο­ ευρετήριο ονομάτων και όρων, στο τέλος του βιβλίου. Το συνιστούμε σε κάθε φιλομαθή και ιδιαιτέ­ ρως στους μαθητές της Β ' και Γ Λυκείου, όταν θα έχουν ελεύθερο χρόνο.

ΕΥΚΛΕΙΔΗΣ Β' λζ' τ.2/1 2


Το βήμα του Ευκλείδη

r Διερεύνηση ενό ς Να βρεθεί πόσα τρίγωνα υπάρχουν της μορφής του διπλανού σχήματος, αν είναι γνωστό ότι: η Ε Ν * και Ο < χ ::::;; 1 . Α

προβλ ήματος Γιάννης Δ. Στρατής

υα = .J36x(x + 3χ) = 1 2χ = η = (ΑΡ) · δηλαδή η ΑΡ ειναι κάθετη στη ΒΓ. Κι επειδή Ο < χ :::; 1 , συμπεραίνουμε ότι: 0 < η ::; 12 · δηλαδή η ε S με S {1, 2 , . . . 1 2 } . Σε κάθε η του S αντιστοιχεί ένας ακριβώς χ = .Ε_ . Οπότε κατασκευάζεται τρίγωνο ΑΒΓ με 12 ' α = 7 η, β = 5 η, γ = 13 η (4) πλευρες: 4 6 12 ' 9 [ 1 5 αφου: α - γ = ϊ2 η < 4 η = β < 4 η = α + γ ] . Α ·

β = n+3χ r �------�--� 8

α = n + 2χ

Σχήμα 1

Το πρόβλημα είχε προταθεί απ ' τον Α καδημα ϊκό κύριο Νικόλαο Αρτεμιάδη στο τεύχος 48 στη στήλη « 0 Ευκλε ίδης προτε ίνε ι . . . Ευκλείδη και Δ ιόφαντο». Στο σχήμ α ωστόσο που συνόδευε την εκφώνηση του προβλήματος, παραλείφθηκε η ένδειξη ότι η ενδιάμεση ευθεία ΑΡ είναι κάθετη στη ΒΓ.

Σχήμα 3 15

12

Η διαπραγμάτευση που ακολουθεί αντι μετωπίζει

το πρόβλημα σε κάθε περίπτωση όπου το ΑΡ είναι

lf----�+-----J.P l l

βέβαια συγκεκρ ιμένο όχι όμως αναγκαστικά κάθετο στη ΒΓ.

<------1-4

--='"'...,_________� Βιz

__

( �) ( �) Και επειδή: Ε = � αυα = ( χ + � η ) υα

Εύκολα

'Εχουμε: Ε = χ + η 3η χ + η .

εσωτερικό της ΒΓ.

α = η +2χ

Σχήμα 2

( �)

,---,-----,-

συμπεραίνουμε ότι: υα = 3η χ + η

(1)

Πρέπει: υα :::; (ΑΡ) = η ή ισοδύναμα:

(

)

3η χ + .!.. η -< η {::} 12χ -< η 4 Έστω η = 2χ τότε:

διαπιστώνουμε

' 5 . Άρα το ιχνος συνΒ = }3

Α

1.

13

(2)

(

Δ

συνΓ = -3 ' 5 ' ' του υψους υα ειναι ότι:

)

Κι επειδή: υα = 3η .Ε_ + .!.. η δηλαδή υα. = η, 12 4 συμπεραίνουμε ότι υπάρχουν 12 ακριβώς τρίγωνα, όμοια μεταξύ τους, που ανταποκρίνονται στο σχή­ μα του προβλήματος. 2. Έστω η > 1 2χ {::} (ΑΡ) > υα . Ωστόσο ισχύει: Ο < χ ::Ξ; 1 . Κι επομένως συ­ μπεραίνουμε ότι: «για κάθε η ε Ν με η ;::: 1 3 ισχύ­ ει η> 12χ · δηλαδή (ΑΡ) > υα. Κι επειδή για κάθε η Ε Ν * και χ ε (0, 1] ισχύει: IC η + 2χ) - (η + x) l < η + 3χ < (η + 2χ) + (η + χ) , συμπεραίνουμε ότι υπάρχουν άπειρα τρίγωνα, που ανταποκρίνονται στο σχήμα 1 .

ΕΥΚΛΕΙΔΗΣ Β ' λζ ' τ.2/13


Το βήμα του Ευκλείδη

Ενδιαφέρον λοιπόν έχει να μελετήσουμε το

πρ όβλημα, όταν το ευθύγραμμο τμήμα ΑΡ είναι καθορισμένο δηλαδή όταν το σημείο Ρ είναι σ� γκεκριμένο και δεν είναι το ίχνος Δ το υ υa-

Κατ' αρχάς παρατηρούμε ότι το Ρ πρέπει να εί­ ναι εσωτερικό σημείο το� ευθυγράμμου τμήματος Β Ή · όπου Β ' το συμμετρικό του Β ως προς το ί­ χνος Δ του ύψους υα. Α

Για το πρώτο έχουμε: η<η. 12χ = � ( .Jϊ3 - 3 ) η = 2 13 + 3 Για το δεύτερο έχουμε: 4μ : = z 3 2

Jil

[ ( � - I ) + ( �+ 5 ) - ( �+ I )} :: '

=2

Β'

Μ Σ

Δ

Μ'

Β

α = η+2χ 2. 1 Έστω ότι το σημείο Ρ είναι το μέσο Μ της ΒΓ · δηλαδή � = η. Καθώς είναι γνωστό (Πρώτο Θεώρημα Διαμέσων) ισχύει: μ � = -1 ( 2 β 2 + 2 γ 2 - α) . 4 Έτσι έχουμε: 4μ� = 3η2 + 1 2ηχ + 1 6χ2 (5) Συνεπώς έχουμε να επιλύσουμε την εξίσωση: 1 6χ2 + 1 2nx - η2 = Ο , (6) που έχει μια μόνο θετι­ κή λύση

Κι επειδή Ο < χ ::; 1 συμπεραίνουμε ότι: (8) η ::; 2.Jϊ3 + 6 Αλλά 13 < 2.Jϊ3 + 6 < 14 . Άρα η ::ς 13 · δηλαδή η ε S' με s' = {1, 2, ... , 1 3} . Σε κάθε η του συνόλου S ' αντιστοιχεί μια ακρι­ βώς τιμή του χ, που δίνεται από τον τύπο (7). Και συνεπώς κατασκευάζεται τρίγωνο ΑΒΓ με πλευρές α = .Jϊ3 + 1 η ' β = 3.Jϊ3 - 1 η ' γ = .Jϊ3 + 5 η 8 8 4 και είναι προφανώς όμοια μεταξύ τους. Για νάναι ·δεκτή η τιμή του χ, που δίνει ο τύπος (7) πρέπει και αρκεί να ισχύουν οι συνθήκες: 12χ < η και μα = η .

0,

δηλαδή 4μ � = 4η2 • Οπότε μα = η . Άρα στην περίπτωση που η ΑΡ είναι η διάμεσος του τριγώνου, υπάρχουν 1 3 ακριβώς τρίγωνα, όμοια μεταξύ τους, που ανταποκρίνονται στο σχήμα 1 . 2.2 Έστω

Γ

'

ότι το σημείο Ρ είναι το σημείο Σ, που η διχοτόμος της γωνίας ΒΑΓ συναντάει τη ΒΓ· δηλαδή: δα = η. (9) 'Εχουμε (ΓΣ) = � . (ΣΒ) γ η + 3χ . , (ΓΣ) = β α = -Και συνεπως 2 β+γ Έτσι παίρνουμε: (ΓΣ) = (ΓΔ) - (ΔΣ) = �β 2 - υ� - �δ � - υ� (I) 1 = - (η + 6χ). - -1 Jη(η - 12χ) (9) 2 2 Συνεπώς αναζητούμε τη λύση της εξίσωσης: 9χ2 + 12ηχ - η2 = 0 ( 1 0). JS-2 ' μια ' μονο ' θετικη' λ:υση ' χ=Η (1) εχει -η (1 1 ) . 3 Κι επειδή Ο < χ ::; 1 συμπεραίνουμε ότι: (12). η ::; 3 = 3 ( J5 + 2 ) J5 _ 2 Ωστόσο 1 2 < 3J5 + 6 < 1 3 .Άρα η ::; 1 2 · δηλα­ δή η Ε S [όπου S = { 1 ,2, ... ,12}] Σε κάθε η Ε S αντιστοιχεί ένας ακριβώς χ, που δίνεται απ' τον τύπο ( 1 1) και συνεπώς κατασκευάζεται τρίγωνο ΑΒΓ με πλευρές: α = 2J5 - 1 η , 3 ( r; J5 + 1 αφου:' β = ν 5 - 1 ) η , γ = -3-η J5 α - γ = - 2 η < ( JS - 1 ) η = β < JSη = α + γ 3

ΕΥΚΛΕΙΔΗΣ Β ' 'λl,' τ.2/14

Λ

--

,


Το βήμα του Ευκλείδη

Το τρίγωνο αυτό ανταποκρίνεται στο σχήμα 1 δι& 2- βγτ(τ α) ]. τι: δa = η. [Θυμiζουμε ότι: δα = .j β+γ Τελικά, στην περίπτωση που η ΑΡ είναι η διχοτόμος της γωνίας Α του τριγώνου, υπάρχουν 12 ακριβώς τρίγωνα, όμοια μεταξύ τους, που ανταπο­ κρίνονται στο σχήμα 1 . Λ

2.3 Έστω

τώρα ότι το σημείο Ρ διατρέχει το ευ� = κ ό­ γραμμο τμήμα ΜΔ . Κι ας είναι (ΜΡ) (ΡΔ ) που κ συγκεκριμένος θετικός αριθμός. Τότε: (ΜΡ) = _κ_ (ΜΔ ), (Ρ Δ ) = -1- (ΜΔ ) . κ+1 κ+1 (5) Κι επειδή: ΜΔ = �μ� - υ� = ·

(I)

Οπότε:

' ..,. Για Ο < κ -< 4Ji3 - 1 3 ισχuει: 13 �9(κ + 1)2 + 4 + 3(κ + 1) 1 3 <- 2 < 14 και συκ+1 νεπώς: η :::; 1 3 δηλαδή η Ε S ' (όπου S ' = { 1 ,2, ... , 1 3 } ). , ,... Για κ >. 4Ji3 - 1 3 ισχuει: 13 �9(κ + 1)2 + 4 + 3(κ + 1) 12 < 2 < 13 κ+1 και συνεπώς η ::; 12 δηλαδή η ε S (όπου S { 1 , 2, ... , 12} ). Για να είναι τώρα δεκτή η λύση, που δίνει ο τύ­ πος ( 1 6), πρέπει και αρκεί να επαληθεύει τις συν­ θήκες: υα < η < μα . 8 Η πρώτη είναι ισοδύναμη, καθώς έχουμε απο­ δείξει, με τη συνθήκη: 12χ < η . Έχουμε: 12χ = (κ+ 1 �9(κ+ 1 )2 +4+3(κ+ 1 η<η � ·

ιιι...

·

=

/]_ (3η2 + 12ηχ + 1 6χ2 ) - 3ηχ - l η2 2χ ( 1 3), 4 V4 2 συμπεραίνουμε ότι (ΜΡ) = κχ , (Ρ Δ ) = � (14) κ+1 κ+1 Τώρα από το ορθογώνιο τρίγωνο ΑΔΡ έχουμε: 3(κ + 1)�9(κ + 1)2 + 4 < 9(κ + 1)2 + 2 (ΑΡ)2 = (ΑΔ )2 + (ΔΡ)2 . Οπότε αναζητούμε τη (\) που είναι αληθής (αποδεικνύεται εύκολα). λύση της εξίσωση: η2 3ηχ+ -3 η2 + 4χ2 ( 1 5) ( 1 4) • Η δεύτερη είναι ισοδύναμη με τη συνθή κη : 4 (κ+1) 2 4η2 < 3η2 + 12ηχ + 1 6χ2 ή ή 1 6χ2 + 12(κ + 1)2 ηχ - (κ + 1)2 η2 = 0 , που έχει 1 6χ2 + 12ηχ - η2 > 0 ( 19). μία μόνο θετική λύση: Το τριώνυ μο φ( χ) = 1 6χ2 + 1 2nx - η2 έχει ρί�-9(_κ_+-1)--:2 :- +-4 - 3( κ + 1) χ = η (κ + 1) (16). ζες ρ ι = -3 - Ji3 η , ρ = -3 + Ji3 η 8 2 8 8 Κι επειδή Ο < χ :::; 1 , συμπεραίνουμε ότι Συνεπώς για να δείξουμε τη ( 1 9) είναι αρκετό 8 η <να δείξουμε ότι η τιμή του χ, που δίνει ο τύπος (κ + 1) �9(κ + 1)2 + 4 - 3(κ + 1) ( 1 6), είναι μεγαλύτερη της Ρ2 δηλαδή: �9(κ+1)2 +4+3(κ+1) -3 +Ji3 �9(κ + 1)2 + 4 + 3(κ + 1) η η(κ+1) > η ::; 2 ή (1 7) 8 8 κ+1 � 2 ( 1 1 - 3Ji3 ) ( κ2 + 2κ ) > 0 Εύκολα αποδεικνύεται ότι: που είναι αληθής. Ας είναι τώρα Ν το ακέραιο μέ�9(κ+ 1)2 +4+3(κ+1) 12 :::; 2 . < 14 , για κάθε κ>Ο. �9(κ+1 )2 +4+3(κ+1) κ+1 ρος του 2 , δηλαδη, κ+1 Επιλύουμε τώρα την ανίσωση: �9(κ + 1)2 + 4 + 3(κ + 1) �9(κ + 1)2 + 4 + 3(κ + 1) , 2 < 13 Ν= 2 , τοτε για κάθε κ+1 κ+1 4Ji3-1 3 . η Ε {1, 2, ... , Ν} αντιστοιχεί ένας ακριβώς χ, η τιμή ή ισοδύναμα �9(κ+1 )2 +4<7 (κ+1) <::::> κ> 13 =

%

)[

)]

=

·

[

]

ΕΥΚΛΕΙΔΗΣ Β ' λζ' τ.2/15


Το βήμα του Ευκλείδη

λη > 2(λ - 2)χ . του οποίου δίνεται από τον τύπο ( 1 6) και συνεπώς • Αν η :?: 2 τότε η :?: 2χ [ αφού: 2 :?: 2χ ] . κατασκευάζεται τρίγωνο ΑΒΓ με πλευρές α = η + 2χ , β = η + 3χ , γ = η + χ . Κι επειδή λ> λ- 2 > Ο, συμπεραίνουμε ότι λη > 2(λ - 2)χ . Αν λοιπόν το σημείο Ρ χωρίζει το ΜΔ σε λόγο κ, τότε (ΑΡ) = η [προκύπτει αμέσως από την σχέση (15)]. • Αν η = 1 , τότε 12χ < 1 {::} χ < _!_ . Συνεπώς 12 Τελικά υπάρχουν Ν _το πλήθος τρίγωνα με 1 (λ - 2) = 2χ(λ - 2) . λη = λ > -1 (λ - 2) = 2 · 1 2 , αν κ > 4JΊ3 - 1 3 6 12 . αντα13 Ν= , τα οποια Η (2 1) είναι ισοδύναμη με την εξίσωση: r;-;:; 3 1 3 4ν1 ο 13 , αν < κ <(λ + 1) 2 (η 2 - 12ηχ ) = [ λη + 2(2 - λ)χ] 2 ή μετά τις 13 ποκρίνονται στο σχήμα 1 , στην περίπτωση που το πράξεις: 2 2 4(λ-2) χ +4(2λ2 +8λ+3)ηχ-(2λ+ 1)η 2 =Ο (22). Ρ χωρίζει το ΜΔ σε λόγο (ΜΡ) = κ (κ > Ο) . (α): Για λ = 2 η (22) γίνεται: 1 0 8χ = 5η ή (Ρ Δ ) 5 η (23) . . 2.4. Αν το Ρ διατρέχει το ευθύγραμμο τμήμα Δ Μ ' χ = 108 όπου Μ ' το συμμετρικό του Μ ως προς το ίχνος Κι επειδή Ο < χ ::::; 1 συμπεραίνουμε ότι: Δ του ύψους υα, θα έχουμε, λόγω συμμετρίας 1 8 = 2 1 .6. Άρα η ::::; 2 1 · δηλαδή τα ίδια αποτελέσματα με ό,τι βρήκαμε στο 2. 1 η ::::; για το Μ και στο 2.3 για οποιοδήποτε σημείο Ρ η Ε {1, 2, ... , 2 1} . που χωρίζει το ΜΔ σε λόγο κ(κ>Ο). Σε κάθε η Ε {1, 2, ... , 2 1} αντιστοιχεί ένας ακρι2.5. Έστω ότι το σημείο Ρ διατρέχει το ευθύγραμ­ βώς χ, που δίνεται από τον τύπο (23) . Οπότε καταμο τμήμα ΜΉ κι ας είναι ' ' α = 1 18η , ΑΒΓ με πλευρες: σκευα. ζεται τριγωνο 1 08 (Μ'Ρ) = λ · όπου λ>Ο συγκεκριμένος. . . οπως (ΡΒ) ' θα αυτες, β = 123 η , γ = 1 1 3 η . [ Ο ι λυσεις 1 08 108 λ Έχουμε: (Μ'Ρ) = __(Μ'Β), δούμε παρακάτω, εμπεριέχονται στο γενικό τύπο λ+1 (24)]. (ΡΒ) = -1-(Μ'Β) κι επειδή (β): Η (22) για λ � 2 έχει μία μόνο θετική λύση: λ+1 2 +8λ+3) 2 +(2λ+ 1)(λ-2) 2 -(2λ2 +8λ+3) 1 1 �(2λ (Μ Β)=(ΜΒ }-(ΜΜ )= (η+2χ}-4χ= (η-6χ) συ- χ = η .... --------:---._ .:. 2 2 2(λ-2) 2 μπεραίνουμε ότι: (2λ+1)η = λ( η 6χ) η 6χ 2 +8λ+3) 2 +(2λ+ l )(λ-2) 2 +2λ2 +8λ+3 (24) (20). (Μ'Ρ) = (ΡΒ) = 2 � (2λ 2(λ + 1) ' 2(λ + 1)

1

---

Ι

Ι

[

Ωστόσο: (Μ'Ρ) = ( ΔΡ) - (ΔΜ') =

_!_� η( η - 1 2χ) - 2χ . Συνεπώς αναζητούμε τη 2

λύση της εξίσωσης:

_!_� η(η - 1 2χ) - 2χ = 2

= λ(η-6χ) {::} (λ+1)νιη(η-12χ) =λη+2(2-λ)χ (2 1). 2(λ+1 Το δεύτερο μέρος της (2 1) είναι θετικό. Διότι: • Για 2 - λ � Ο {::} λ ::::; 2 ο ισχυρισμός είναι προφανώς αληθής. Για 2 - λ < Ο {::} λ > 2 αποδεικνύουμε ότι:

]

Ο τύπος (24) έχει νόημα για λ = 2 και γίνεται χ = __2_ η . 'Ετσι στον τύπο (24) συγκαταλέγονται 108 όλες οι λύσεις για την περίπτωση που μελετάμε. Επειδή Ο < χ ::::; 1 συμπεραίνουμε ότι: �(2λ2 +8λ+3) 2 +(2λ+ 1 )(λ-2) 2 +2λ2 +8λ+3 η <- 2 (25) 2λ+1 Θεωρούμε τη συνάρτηση J(2λ2 +8λ+3)2 +(2λ+ ι )(λ-2)2 +2λ2 +8λ+3 f(λ)=2

ΕΥΚΛΕΙΔΗΣ Β ' λζ' τ.2/1 6

2λ+ Ι

/(Ο,+οο)


Το βήμα του Ευκλείδη

ή μετά τις πράξεις 3 4 2 2 f(λ)=2 �4λ +34λ +69λ +52λ+ 1 3 +2λ +8λ+3 (26). 2λ+ Ι Οπότε: f'(λ)

)

3 3 6 -2 _;_ 2 -'-'----"-'------:-r==� =f �=.,.;.. ====::===� Βλ4 +42λ3 +5 Ι λ2 + Ι 7λ+2( λ2 +2λ+ Ι ) 4λ4 + 4λ3 + 9λ2 +5 λ+Ι

)

(2λ+ Ι ) 2 4λ4 + 3 4� + 6 9λ2 + 52λ + 1 3

Τελικά, στην περίπτωση που το σημείο Ρ χωρί­ ' ζει το Μ ' Β σε λόγο (Μ Ρ) = λ (λ>Ο, συγκεκριμέ(ΡΒ) νος), υπάρχουν Ν το πλήθος (Ν � 1 3) όμοια μετα­ ξύ τους τρίγωνα, που ανταποκρίνονται στο σχήμα 1 , αφού σύμφωνα με το Θεώρημα Stewart, ισχύει: (ΑΜ') 2 (ΡΒ) + (ΑΒ) 2 (Μ 'Ρ) (ΑΡ ) 2 (Μ 'Β) + (Μ 'Β)(Μ 'Ρ)(ΡΒ) =

Κι επομένως: f'(λ) > Ο για κάθε λ Ε (Ο, +οο) και συνεπώς η συνάρτηση είναι γνήσια αύξουσα. Οπότε, λόγω των (20) και την εκτέλεση απλών Κι επειδή: lim0 f(λ) = 2 Ji3 + 3 και λ--+ πράξεων καταλήγουμε στην ισότητα: lim f(λ) = +οο συμπεραίνουμε ότι το σύνολο 4(ΑΡ )2 (λ + 1)2 = (λ + 1)(3η2 + 1 2nx + 1 6χ2 )+ λ-++οο τιμών της συνάρτησης αυτής είναι το ανοικτό διά­ +4λ(λ + 1)(η2 + 2nx + χ2 ) - λ(η2 - 12nx + 36χ2 ) στημα 2Ji3 + 6, +οο . 4(ΑΡ )2 + (λ + 1)2 = 4(λ - 2)2 χ2 + Άρα για κάθε λ Ε (0, +οο) ισχύει: +4(2λ2 + 8λ + 3)nx + (4λ2 + 6λ + 3)η2 , =

(

(

)

)

+2λ2 +8λ+3 >2ν13+16 2 �(2λ2+8λ+3)2+(2λ+Ι)(λ-2)2 2λ+Ι r.;;

και συνεπώς

Ι 2λ+Ι -2 ( JΪ3+3 ) 2 [ �(2λ2+8λ+3)2+(2λ+ Ι)(λ-2)2 +2λ 2+8λ+3 ]

��� > -rr===============�

(

--,

)

, Ji3-3 η > η 8

(2λ+1)η

Από την τελευταία συμπεραίνουμε ότι: «η τιμή του χ, που δίνει ο τύπος (24), ανήκει στο εσωτερι­ κό του διαστήματος, που ορίζουν οι ρίζες ρ,, ρ2 (ρ 1 < Ο < ρ ) του τριώνυ μου: 2 φ( χ) = 1 6χ2 + 12nx - η 2 ». Και συνεπώς, γι' αυτή την τιμή του χ, ισχύει φ( χ) < Ο ή 1 6χ2 + 12ηχ + 3η2 < 4η2 ή 4μ � < 4η2 ή μα < η · δηλαδή η τιμή του χ, που δίνει ο τύπος (24) είναι δεκτή. Θέτουμε Ν = [f(λ)] . Οπότε Ν � [2Ji3 + 6] = 13 . Σε κάθε η Ε {1, 2, ... , Ν} αντιστοιχεί, σύμφωνα με τον τύπο (24), ένας ακριβώς χ και κατασκευά­ ζεται τρίγωνο ΑΒΓ με πλευρές α = η + 2χ , β = η + 3χ , γ = η + χ .

που είναι ισοδύναμη, λόγω της (22), με τη σχέση: 4(ΑΡ )2 · (λ + 1)2 = = (2λ + 1)η2 + (4λ2 + 6λ + 3)η2 = 4(λ + 1)2 η2 Άρα: (ΑΡ )2 = η2 ή (ΑΡ ) = η . Π αραη]ρηση :

Όπως αποδείξαμε παραπάνω, το σύνολο των τιμών της συνάρτησης f(λ) είναι το ανοικτό διά­ στημα: (2Ji3 + 6, +οο) . Συνεπώς για οποιονδήποτε ακέραιο Ν0, με Ν 0 � 14 υπάρχει λ Ε (Ο,+οο) τέτοιος ώστε: f(λ) = Ν0 . Πιο συγκεκριμένα: επειδή η συνάρτηση f είναι γνήσια αύξουσα, η συνθήκη : Ν0 ::; f(λ) < Ν0 + 1 (27) είναι ισοδύναμη με την συνθήκη: Γ 1 (Ν 0 ) ::; λ < Γ 1 (Ν0 + 1) . Δηλαδή υπάρχουν άπειρες τιμές του λ, για τις οποίες ισχύει η σχέση: Ν0 = [f(λ)] . Οπότε από τη συνθή κη (25) προκύπτει η ::; Ν 0 δηλαδή η Ε {1, 2, ... , Ν0 } . Και συνεπώς υπάρχουν Ν0 το πλήθος, όμοια μεταξύ τους τρίγω­ να, που ανταποκρίνονται στο σχήμα 1 . 2 . 6 Αν το σημείο Ρ χωρίζει το ΜΒ ' σε λόγο (Μ ) = μ (μ>Ο, συγκεκριμένος) έχουμε τα ί­ (ΡΒ ) δια αποτελέσματα.

ΕΥΚΛΕΙΔΗΣ Β' λζ' τ.2/1 7


Ο Εuκλείδnς nροτείvει Εuκλείδn

. .. και . .. Διόφαvτο

θέματος, σύμφωνα με το διπλανό σχήμα. Δ ιέ­ κρινε στην ουσία δύο περιπτώσεις: (α) το Μ εί­ ναι οποιοδήποτε εσωτερικό σημείο του ευθ� γραμμου σχήματος ΒΓ και (β) το Μ είναι το μέσο του ΒΓ.

Ι. Λυ μένα θέ ματα

1 4.

(το είχε προτείνει ο Ακαδημαϊκός κύριος Νι­

κό λαος Αρτεμ ιάδ ης) .

Πόσα τρίγωνα έχουν τη μορφή του διπλα­ νού τριγώνου όπου η είναι ένας θετικός α­ κέραιος αριθμός και χ ένας πραγματικός α­ ριθμός με ο < χ � ι ;

Α

Α

Απάντη ση

Το εμβαδόν τριγώνου με πλευρές α, β, γ δίνεται από τον τύπο (του Ήρωνα): Ε= �τ(τ - α)(τ - β)(τ - γ) όπου: τ= α+β+y . 2 Αντικαθιστώντας στον τύπο αυτό τις τιμές των πλευρών α = η + 2χ, β = η + 3χ, γ = η + χ βρίσκουμε: Ε = 2_ (η+2χ) 2 (η+4χ)η = ! (η+2χ) · η 16 2 [αφού Ε = αυα ]. 'Ετσι μετά από μερικές απλές

2x+ n

Για το (α): Από το Νόμο των συνημίτονων στα τρίγωνα ΑΒΓ και ΑΜΓ παίρνουμε 2 2 2 συνΓ = (χ + η) + (2χ + η) - (3χ + η) 2(χ + η)(2χ + η) και (1) 2(χ + η)(2χ + η) 2(χ + η) =

2 - η 2 χ 2 + 2nx + y 2 (χ + η) 2 + y'(2) - συνΓ = -'---'-----' 2 y(x + η) 2(χ + η)y όπου y = (ΜΓ). Από τις (1) και (2) προκύπτει η εξίσωση: χ 2 + 2nx + y 2 = ηy = 2xy {::} πράξεις έχουμε: 12χ = η . Επειδή: Ο < χ � 1 και ο η είναι θετικός ακέραι/ + (2χ - η) y + (χ 2 + 2nx) = O (3) ος, παρατηρούμε ότι υπάρχουν μόνο 12 τρίγωνα Η διακρίνουσα της (3) είναι: Δ = η(η - 12χ) . της ζητουμένης μορφής δηλαδή: Και συνεπώς πρέπει: η - 12χ 2:: Ο ή η 2:: 12χ (4) ...,_ Αν η = l2 χ , τότε y = 5x (5) 2, , ... , (12, 1) . (η, χ) 1, 1 1 Και συνεπώς: (ΑΜ) = 12χ , (ΜΓ) = 5χ , (ΑΓ) 13χ . Οπότε: (ΑΓ) 2 = (ΑΜ) 2 + (ΜΓ) 2 . Δηλαδή ΑΜΓ 90° . Με όJ..λα λόγια η ΑΜ εί• Την ίδια διαπαγμάτευση κάνει ο συνάδελφος ναι κάθετη στη ΒΓ. Γεώργιος Κατσαούνης απ' την Πρέβεζα. η ' ' αυτη' εχουμε: }2 χ . Στην περιπτωση • Ο συνάδελφος Αθανά σ ιος Καλάκος από τα Πατήσια μας έστειλε μια διαπραγμάτευση του ___::__

_ _

Ε!( �Η �)

!

=

Λ

=

=

ΕΥΚΛΕΙΔΗΣ Β' λζ' τ.2/18


Ο Ε κλείδης προτείνει... Ευκλείδη

•..

και Διόφαντο

Κι επειδή Ο < χ � 1 , συμπεραίνουμε ότι:

� 1 ή η � 12 δηλαδή: η Ε {1, 2, ... , 1 2} . 1 Σε κάθε τιμή του η αντιστοιχεί, λόγω της (6), μια ακριβώς τιμή του χ. Και συνεπώς υπάρχουν 12, μη ίσα, τρίγωνα της ζητούμενης μορφής. Τα τρίγωνα αυτά είναι οξυγώνια, οπότε και τα τρία ύψη βρίσκονται στο εσωτερικό του. Πράγματι 5 Γ > Α > Β και συνΓ = - . 13 � Αν η > 12χ , τότε από την (3) παίρνουμε: η - 2χ ± η(η - 12χ) y= (7 ) 2 Και οι δύο τιμές, που μας δίνει η (7), είναι δε­ κτές (εύκολο). Άρα ορίζεται εσωτερικό σημείο Μ του ευθύγραμμου τμήματος έτσι ώστε (ΑΜ) = η. Η περίπτωση αυτή εξασφαλίζει την ύπαρξη α­ πείρων, μη ίσων, τριγώνων που έχουν τη ζητούμε­ νη μορφή: για δεδομένο χ Ε (0, 1] αρκεί να είναι: η 2:: [12χ] + 1 . Λ

Λ

Ο συνάδελφος Γιώργος Τρ ιάντο ς, μέλος της Συντακτικής Επιτροπής, διαπραγματεύθηκε το θέμα· στην περίπτωση που η ενδιάμεση ευθεία είναι η διάμεσος ΑΜ του τριγώνου.

Ο συνάδελφος Γιάννης Δ. Στ ρ ατής, μέλος της Συντακτικής Επιτροπής, διαπραγματεύεται το ζήτημα πιο γενικά (Δες «Το Βήμα του Ευκλεί­ δψ> στο παρόν τεύχος).

Λ

.J

Για το (β): η , Για y = 2χ + η (3) γινεται: 2 2 (2χ+η) +2(2χ-η)(2χ+η)+4(χ 2 +2nx)=O

1 5.

(το είχε προτείνει ο Ακαδημαϊκός κύριος Νι­

κόλα ος Αρτεμιά δη ς) .

Έστω ΑΒΓΔ κυρτό τετράπλευρο. Να εξε­ ταστεί αν αληθεύει η παρακάτω πρόταση: Υπάρχει σημείο Ε στο επίπεδο του τετρα­ πλεύρου τέτοιο ώστε «τα τρίγωνα ΑΒΕ, ΓΔΕ είναι όμοια» Απάντη ση

Το σημείο Ε υπάρχει πάντοτε. ΑΝ η ΑΒ είναι παράλληλη προς τη ΓΔ, τότε το ζητούμενο σημείο Ε είναι η τομή των διαγωνίων ΑΓ και ΒΔ. ΑΝ η ΑΒ δεν είναι παράλληλη προς τη ΓΔ, τότε τέμνιr­ νται σε κάποιο σημείο Ζ.

ή ισοδύναμα: η 2 - 12ηχ - 16χ 2 = Ο {::} η = χ 6 + 2.JU (8).

(

Κι επειδή Ο < χ � 1 , συμπεραίνουμε ότι: η = χ 6 + 2.Ji3 (9)

(

)

)

Ωστόσο: 13 < 6 + 2.Ji3 < 14 . Οπότε η � 1 3 · η.Ji3 . δηλαδή η Ε {1, 2, ... , 1 3} και χ = 6 + 2 13 Θέτουμε: λ = 6 + 2.Ji3 . Οπότε το χ παίρνει τις 1 2 ... 'λ 1 3 . κι ετσι , , , συμπεραινουμε , λ'λ' τιμες: οτι υπάρχουν 13, μη ίσα, τρίγωνα της ζητούμενης μορφής. Αξιοσημείωτο είναι ότι τόσο στην περίπτωση που το Μ είναι το ίχνος του ύψους από την κορυφή Α όσο και στην περίπτωση που το Μ είναι το μέσο του ΒΓ τα τρίγωνα που προκύπτουν είναι όμοια μεταξύ τους.

Έστω Ε το δεύτερο σημείο τομής των περιγε­ γραμμένων κύκλων στα τρίγωνα ΑΓΖ και ΒΓΖ. Το τετράπλευρο ΑΕΓΖ είναι εγγεγραμμένο σε κύκλο · άρα οι γωνίες ΕΑΖ και ΕΓΖ είναι παρα- πληρωματικές. Συνεπώς ΕΑΒ ΕΓΔ . Όμοια το τετράπλευρο ΕΔΓΒ είναι εγγεγραμ· μένο σε κύκλο άρα οι γωνίες ΕΔΖ και ΕΒΖ είναι παραπληρωματικές. Συνεπώς ΕΔΓ ΕΒΑ .

ΕΥΚΛΕΙΔΗΣ Β ' λζ' τ.2/19

=

=


Ο Εκλείδης προτείνει ... Ευκλείδη

•..

και Διόφαντο

Επομένως τα τρίγωνα ΑΕΒ και ΕΔΓ έχουν δύο είναι εγγράψιμα. Άρα το Ε είναι η δεύτερη τομή των σταθερών κύκλων ΖΑΒ και ΖΓΔ. γωνίες ίσες (μια προς μια· δηλαδή είναι όμοια). β) Κατ α σ κευή

Βρίσκουμε τη τομή Ζ των διαγωνίων ΑΓ και συνάδελ­ οι κάνουν Την ίδια διαπραγμάτευση ΒΔ. Το Ε είναι τότε το δεύτερο σημείο τομής των φοι Αντώνη ς Ιωαννίδ η ς απ ' τη Λάρισα, Γ ε­ κύκλων ΖΑΒ και ΖΓΔ. ώργιος Κατ σ αούνης απ ' τη Πρέβεζα. γ) Απ ό δειξη

Τα τετράπλευρα ΑΒΕΖ και ΓΔΖΕ είναι εγγε­ Ο συνάδελφος Ν ίκος Κυ ριαζής, απ ' τη Καλα­ γραμμένα σε κύκλους. μαριά Θεσσαλονίκης μας έστειλε τη παρακάτω Άρα: ΕΑΒ = ΕΖΒ = ΕΓΔ . διαπραγμάτευση. Οπότε: ΕΑΒ = ΕΓΔ (4) α) Ανά λυ ση Όμοια δείχνουμε ότι: ΑΕΒ = ΓΕΔ (5) Υποθέτουμε ότι το ζητούμενο σημείο υπάρχει Απ ' τις (4), (5) συμπεραίνουμε ότι: «τα τρίγωνα και ότι είναι το Ε. ΕΑΒ και ΕΓΔ είναι όμοια». ΑΒ ΑΕ ΒΕ . ,Εχουμε: =-=δ) Διερε ύν ηση ΓΔ ΕΓ ΕΔ ΑΕ ΓΕ Είναι φανερό ότι: «το πρόβλημα έχει λύση όταν Οπότε : (1) = ΕΒ ΕΔ οι κύκλοι τέμνονται». Κι ακόμα: ΑΕΒ = ΓΕΒ . Αν οι κύκλοι εφάπτονται, τα σημεία Ε και Ζ Οπότε: (2) συμπίπτουν. Οπότε η ΑΒ είναι παράλληλη προς ΑΕΓ = ΒΕΔ Απ ' τις συνθήκες (1) και (2). την ΓΔ δηλαδή' το τετράπλευρο ΑΒΓΔ είναι τρα­ πέζιο. Α •

-

-

-

-

-

-

-

Σ χό λιο :

Το πρόβλημα έχει λύση και στη περίπτωση που το τετράπλευρο ΑΒΓΔ είναι μη κυρτό (Σχήμα 2).

Σχήμα 1 Συμπεραίνουμε ότι: « Τα τρίγωνα ΕΑΓ και ΒΕΔ είναι όμοιω). Οπότε: (3) ΕΑΓ = ΕΒΔ και ΕΔΒ = ΕΓΑ Σχήμα 2 Ας είναι τώρα Ζ η τομή των διαγωνίων ΑΓ και ΒΔ. Δηλαδή: ΕΑΒ = ΓΔΕ, ΑΒΕ = ΓΔΕ . Οπότε τα Η (3) γίνεται: ΕΑΖ = ΕΒΖ και ΕΔΖ = ΕΓΖ . Και συνεπώς τα τετράπλευρα ΑΒΕΖ και ΓΔΖΕ τρίγωνα ΑΒΕ και ΓΔΕ είναι όμοια . -

-

---

---

---

----

---

.. . . . . . . . . . . . . . . . . . . . . . . . . . . . . . . . . . . . . . . . . . . . . . . . . . . . . . . . . . . . . . . . . . . . . . . . . . . . . . . . . . . . . . . . . . . . . . . . . . . . . . . . . . . . . . . . . . . . . . . . .

: Η Ελληνική Μ α θ η μ ατική Ε ται ρεί α σ α ς εύχεται:

. .

:

:.

Ε Υ Τ Υ Χ Ι Σ Μ Ε Ν Ο ΚΑΙ Δ Η Μ Ι Ο Υ Ρ Γ Ι Κ Ο Τ Ο

2004

:. .

: .

. . . . . . . . . . . . . . . . . . . . . . . . . . . . . . . . . . . . . . . . . . . . . . . . . . . . . . . . . . . . . . . . . . . . . . . . . . . . . . . . . . . . . . . . . . . . . . . . . . . . . . . . . . . . . . . . . . . ..

ΕΥΚΛΕΙΔΗΣ Β' λζ τ.2/20


Η Homo Mathematίcus είναι μια στήλη στο περιοδικό μας, με σκοπό την ανταλλαγή απόψεων και την ανάπτυξη προβλημα-η­ σμού πάνω στα εξής θέματα: l) Τι είναι τα Μαθηματικά, 2) Πρέπει ή όχι να διδάσκονται, 3) Ποιοι είναι οι κλάδοι των Μαθημα­

τικών και ποιο το αντικείμενο του καθενός, 4) Ποιες είναι οι εφαρμογές τους, 5) Ποιες επιστήμες ή κλάδοι επιστημών απαιτούν καλή γνώση των Μαθηματικών για να μπορέσει κάποιος να τους σπουδάσει. Για

τους συνεργάτες της στήλιις: παράκληση ! τα κείμενα της στήλης αυτής, ως προς το περιεχόμενό τους και ως προς το επi­

πεδό τους, θα πρέπει να είναι συμβιβαστά με τα ενδιαφέροντα και το επίπεδο κατανόησης από μέρους των παιδιών.

Ι . "πο υ και με ποιο τρόπο εφαρμό(ο νται τα .Ηα θψι α τιh·ά,· " Θέμα : \ Ι α θ η ματικ ά και Δ ι ά σ τ η μα

Οι στρατιωτικές υπηρεσίες, οι αεροναυτικές και οι ηλεκτρονικές βιομηχανίες είναι, όπως και τα πανεπιστήμια, οι σπουδαιότεροι εργοδότες των μαθηματικών στις ΗΠΑ. Ένα από τα μεγαλύτερα πεδία για τους επιστήμονες αυτούς είναι η κατ ά­ κτηση του διαστήματος . Οι μαθηματικοί προπ α­ ρασκευάζουν την εξωγήινη περιπέτεια. Για μια μόνο πτήση στο διάστημα πρέπει να υ­ πολογίσουν περί τις 100 τροχιές, για να διαλέξουν την καλύτερη δυνατή, καθώς και άλλες 20 διαζε υ­ κτικές για περιπτώσεις ανάγκης. Έχουν να λάβουν υπ' όψη τους μύριους παράγοντες, όπως τις πιθ α­ νότητες σύγκρουσης με μετεωρίτες, τις ασφαλ έ­ στερες πορείες διαμέσου των ακτινοβολιών, το βάρος και την κατανάλωση καυσίμων. Ο υπολ ο­ γισμός της τροχιάς είναι ζήτημα δευτερολέπτων για ένα ηλεκτρονικό υπολογιστή- αφού όμως έ­ χουν εργασθεί προηγούμενα πέντε άτομα επί 4-5 μήνες για να τον τροφοδοτήσουν με σωστές πλ η-

ροφορίες. Μόνο όταν τελειώσουν οι μαθηματικοί αναλαμ βάνουν οι μηχανικοί του διαστήματος την κατασκευή του διαστημοπλοίου και την εκτόξευσή του. Η σημασία των Μαθηματικών φάνηκε καθ α­ ρά στην πρώτη πτήση του "Μάρινερ" προς την Α­ φροδίτη. Ένα μηδαμινό τυπογραφικό λάθος σε μια μαθηματική εξίσωση, που είχε τοποθετηθεί στον ηλεκτρονικό υπολογιστή, κατέστρεψε ολ ό­ κληρη την πτήση- ο "Μάρινερ" που είχε στοιχίσει 17.200.000€, παρεξέκλινε από την τροχιά του και κατά συνέπεια απότυχε. Ο μαθηματικός Βίκτωρ Σέμπελυ, ανώτατο στ έ­ λεχος της Τζένεραλ Ελέκτρικ, ασχολείται με την πρόβλεψη ουρανίων φαινομένων και αφήνει στους άλλους τη φροντίδα της παρατήρησης. «αν οι π α­ ρατηρήσεις τους δεν επιβεβαιώνουν τις προβλ έ­ ψεις μου, αυτό δεν σημαίνει ότι έχω _κάνει λάθος. Δεν ξέρουν ίσως να παρατηρήσουν>> [πηγή: LΙFΕ­ Επιστημονική Βιβλιοθήκη , τόμ. "Μαθηματικά"]

11. "Α υ τό το ξέρατε; "

Ποια καμπύλη ονομάσθηκε "η Ελένη της Γεωμετρίας"» (εμπνευσμένο από την "ωραία λέμου) [η απάντηση στο τέλος της στ ήλης]

Ελένη" του τρωικού π

ο­

ΙΙΙ. "Οι συνεργάτες τιις στήλης γράφο υ ν-ερωτο ύ ν "

Α. Τα Μ α θ η ματικά, το η μερολόγιο και οι π ρο ληπτικοί άνθρωπ οι «Προληπτικοί όλου του κό σ μου . . . Κουράγιο>>, Δρ Μ πάμπη ς Τουμά σης, σχολικός σ ύ μβου λος [ Π� τραJ Ι" θέμα: σε ποια ημέρα τη ς εβδομάδας θ α πέσει η / 8 Α πριλίο υ το υ 2008;

«Αφορμή για το άρθρο αυτό αποτέλεσε ένα ερ � τημα που τέθηκε από τη σύνταξη της στήλης 'Ήοmο mathematicus ' ' του τεύχους του Ευκλείδη Β ' (τ. 45). Το ερώτημα ήταν: « σε ποια ημέρα της εβδομάδας θα πέσει η 181412008; >> Ο πιο εύκολος τρόπος για να απαντήσει κ άποιος σ' αυτό το ερώτημα θα ήταν να ανατρέξει σ' ένα ημερολόγιο του 2008 και να δει πια ημέρα είναι η 1 8 Απριλίου. Όσο όμως κι αν ψάξει στην αγορά δεν πρόκειται να βρει να αγοράσει ένα τέτοιο ημ ε-

ρολόγιο, για τον απλούστατο λόγο ότι είναι ακόμη πολύ νωρίς για να εκδοθεί, μια και δεν θα υπάρχει καθόλου ζήτηση. Είμαστε αναγκασμένοι λοιπόν να βρούμε την απάντηση μόνοι μας, κάνοντας κ άποιους υπολογισμούς. Κατ' αρχάς ανατρέχουμε σ' ένα ημερολόγιο του 2003 και βλέπουμε ότι η 1 8 Απριλίου του 2003 πέφτει Παρασκευή. Στη συνέχεια, υπολογίζουμε πόσες ημέρες θα περάσουν από τις 1 8 Απριλίου 2003 μέχρι τις 1 8 Απριλίου 2008.

ΕΥΚΛΕΙΔΗΣ Β ' λζ ' τ.2/21


ΗΟΜΟ ΜΑΤΗΕΜΑτiCUS

Ημέρες (18/4/2003-31/12/2003) . . . . . . 257 ημέρες Ημέρες του 2004: (Δίσεκτο έτος). . . . . . . . . . .366 >> Ημέρες του 2005: . . . . . . : . . . . . . . . . . . . . . . . . . . . ..365 » Ημέρες του 2006: . . . . . . . . . . . . . . . . . . . . . . . . . . . . .365 » Ημέρες του 2007 : . . . . . . . . . . . . . . . . . . . . . . . . . . . ..365 » Ημέρες του 2008:(Δίσεκ. έτος)μέχρι1 8/4: 109 »

βολίζεται ως 1 827ΞO(mod7) και λέμε ότι οι αριθ­ μοί 1 827 και Ο είναι ισοϋπόλοιποι ή ισότιμοι modulo 7. Γενικά, αν κ ένα θετικός ακέραιος, τότε οι ακέραιοι α και β λέγονται ισοϋπόλοιποι modulo κ, όταν διαιρούμενοι με κ αφήνουν το ίδιο υπόλοι­ πο και γράφουμε αΞβ(mοdκ). Επομένως, όποια ημέρα της εβδομάδας πέφτει η 1 8 Απριλίου 2003, Σύνολο ημερώ ν18/4/2003 - 18141 2008: 182 7 Παρατηρούμε ότι το υπόλοιπο της διαίρεσης του την ίδια μέρα θα πέσει και η 1 8 Απριλίου 2008, 1 827 δια του 7 είναι .0 . Ισχύει δηλαδή, ότι δηλ. Παρασκευή» 1 827=7·26 1 . Αυτό στη θεωρία των αριθμών συμ2" θέμα: . . . Τρίτη και 13 . . .

Ας αναφερθούμε όμως και σε μια άλλη ημερομηνία που προκαλεί πανικό σε ορισμένους ανθρ� πους, οι οποίοι είναι προληπτικοί και πιστεύουν ότι την ημέρα αυτή θα πάθουν μεγάλες συμφορές. Πρόκειται για την «Τρίτη και δεκατρείς». Η Τρίτη θεωρείται δυσοίωνη ημέρα γιατί είναι η ημέρα που έγινε η άλωση της Κωνσταντινούπολης στις 29 Μαίου του 1453. Ο αριθμός 13 θεωρείται γενικά ένας γρουσούζικος αριθμός, αν και είναι πάρα πολλοί αυτοί που πιστεύουν το αντίθετο και θεωρούν ότι τους φέρνει γούρι. Παρακάτω θα προσπαθήσουμε να παρηγορήσουμε τους προληπτικούς που φοβούνται την ημερομηνία αυτή , δίνο-

ντας απάντηση στο ερώτημα: «Πόσες φορές το χρ όνο μπορεί να πέσει Τρίτη και δεκατρείς;»

Παρατηρούμε ότι το 13 διαιρούμενο δια του 7 α­ φήνει υπόλοιπο 6. Σύμφωνα με τον παραπάνω συμβολισμό γράφουμε 13Ξ6(mod7). Αυτό σημαί­ νει ότι οποία μέρα είναι η 6 Ιανουαρίου, η ίδια η­ μέρα θα είναι και η 13 Ιανουαρίου. Γενικά, κάθε αριθμός διαιρούμενος με το 7 αφήνει υπόλοιπο υ ένα στοιχείο του συνόλου {0, 1 ,2,3,4,5,6} . Ανοίγο­ ντας ένα ημερολόγιο του 2003 βλέπουμε, για πα­ ράδειγμα, ότι η 6 Ιανουαρίου πέφτει Δευτέρα. Α­ ντιστοιχίζουμε, τώρα τα στοιχεία του συνόλου {0, 1 ,2,3,4,5,6} τις μέρες της εβδομάδας ως εξής:

ημέρα I I Τρίτη I Τετάρτη I Πέ μπτη I Παρασκευή I Σάββ ατο I Κυριακή_j Δευτέρα I I αντίστοιχο υπόλοιπο I ο I 3 4 5 6 1 I I I I 2 I Με' χρι τη 1 3η ημέρα του Φεβρουάριου θα έχουν στην Πέμπτη, δηλ. θα πέφτει Πέμπτη. Προχω ρ� περάσει 3 1 + 1 3=44 ημέρες από την αρχη του χρό- ντας με τον ίδιο τρόπο δημιουργούμε τον παρακά­ νου . Δηλ. η 1 3η Φεβρουάριου θα είναι η 44η ημέρα τω πίνακα, ο οποίος μας δίνει ταυτόχρονα την η­ του χρόνου. Το 44 διαιρούμενο δια του 7 αφήνει μέρα του έτους και την ημέρα του έτους(mοd7) για υπόλοιπο υ=2, δηλαδή, 44Ξ2(mod7). Αυτό σημαί- την 1 3η ημέρα κάθε μήνα. νει ότι η 44η ημέρα (13 Φεβpουάριου) αντιστοιχεί Μήνας Ιανουάριος Φεβρουάριος Μάρτιος Απρίλιος Μάιος Ιούνιος Ιούλιος Αύγουστος Σεπτέμβριος Οκτώβριος Νοέμβριος Δεκέμβριος

Ημέρα 13 44 72 1 03 133 1 64 1 94 225 256 286 317 347

Κανονικό έτος Ημέρα(mοd7) 6 2 2 5 ο

3 5 1 4 6 2 4

Από τον πίνακα παρατηρούμε ότι κάθε μια από τις τιμές(mοd7) εμφανίζεται τουλάχιστον μια φορά και το πολύ τρεις φορές. Ειδικά η τιμή Ο, που για το έτος 2003 αντιστοιχεί στην ημέρα Τρίτη, εμφ ανίζεται μια φορά. Αυτό σημαίνει πως για το έτο ς 2003 θα έχουμε μόνο μια φορά Τρίτη και δεκατρείς, το Μάιο . Οποιαδήποτε όμως ημέρα κι αν πέσει η 1 3 Ιανουαρίου θα αντιστοιχεί υποχρεωτικά σε ένα στοιχείο του συνόλου {0, 1 ,2,3,4,5,6} , οπότε

Ημέρα 13 44 73 1 04 1 34 1 65 1 95 226 257 287 318 348

Δίσεκτο έτος Ημέρα(mοd7) 6 2 3 6 1 4 6 2 5 ο

3 5

η διαπίστωση μας ισχύει πάλι. Επομένως αποδείξαμε το: Θεώρημα:

Σε ένα οποιοδήποτε έτος, η ημεΡ� Τ μηνία ρίτη και δεκατρείς εμφανίζεται οπω σ,δήποτε μια φορά, ποτέ πάνω από τρεις.

Το παραπάνω θεώρημα διαβεβαιώνει τους απα­ νταχού προληπτικούς πως τα πράγματα δεν είναι και τόσο σκούρα. Ας κάνουν κουράγιο μια, δυο ή το πολύ τρεις ημέρες κάθε χρόνο. Στο κάτω-κάτω

ΕΥΚΛΕΙΔΗΣ Β' λζ' τ.2/22


ΗΟΜΟ MATHEMAτiCUS

ι

μπορούν να πάρουν αναρρωτική άδεια και να μη εφαρμόζοντας τη λαϊκή παροιμία «φύλαγε τα ρού­ ξεμυτίσουν από τα σπίτια τους τις ημέρες αυτές, χα σου για να έχεις τα μισά» Β. " Απόψεις "

Στο προηγούμενο τεύχος υποσχεθήκαμε ν α σας παρουσιάσουμε τη θέση της συναδέλφου μαθηματικού Γεωργίας Γυφτέα (τΕΕ Μούδρου Λήμνου), σχετικά με ένα θέμα που έθεσε ο μαθητής Μιχάλης Ροδίτης (τεύχος 46). Το κείμενο απηχεί τις πρ­ σωπικές απόψεις της συναδέλφισας και κάθε ένσταση θα είναι δημοσιεύσιμη αρκεί να τηρούνται οuρχές της τεκμηρίωσης και της ευπρέπειας. Η σχετική περικοπή έχει ως εξής:

· « . . . Ποιο σημαντικό όμως μου φαίνεται το θέμα που θίγει ο μαθητής στο πρώτο και μεγαλύτερο μέρος του γράμματός του. Για τη μείωση της αξιοπιστίας του σχολείου πιστεύω ότι σε μεγάλο βαθμό φταίμε εμείς οι ίδιοι. Συχνά δίνουμε στους μαθητές το χειρότερο παράδειγμα φυγοπονίας. Τα παιδιά βλέπουν από τη μία τους καθηγητές των φροντιστηρίων να "τρέχουν πανικόβλητοι" (πρέπει να δικαιολογήσουν την αναγκαιότητα ύπαρξής τους . . . ) και από την άλλη (μερικούς από) τους καθηγητές του σχολείου να τσακώνονται για το ποιος θα πρωτολείψει τις παραμονές - και όχι μόνο - των αργιών, να δυσανασχετούν με συναδέλφους που "βιάζονται" να μπουν στο μάθημα αμέσως μετά το

διάλειμμα, να γκρινιάζουν ότι γίνονται λίγο περισ­ σότεροι περίπατοι τη συγκεκριμένη μέρα που έ­ χουν λιγότερες ώρες να χάσουν κλπ. Επί πλέον δεν είναι λίγοι αυτοί που γνώρισα να δείχνουν ανεν­ δοίαστα τη δυσαρέσκειά τους για τον τόπο όπου διορίστηκαν, εργάζονται και μένουν (στη συγκε­ κριμένη περίπτωση στη Λήμνο). Ο τόπος αυτός όμως είναι η πατρίδα των μαθητών. Δεν είναι προ­ σβλητική τέτοια συμπεριφορά; Πέρα από τα προ­ βλήματα που τυχόν έχει ο καθένας μας άμεσα ή έμμεσα λόγω της δουλειάς - προσωπικά, οικογε­ νειακά, οικονομικά, εργασιακά - πρέπει να θυμά­ ται ότι είναι δάσκαλος: παιδαγωγός και εκπαιδευ­ τής»

Για του λόγου το αληθές προσπαθήστε να μετακινήσετε τον βασιλιά από το τετράγωνο αl στο θl , θα διαπιστώσετε ότι ,είτε πάτε ευθεία είτε πάτε μερικώς τεθλασμένα ο αριθμός των κινήσεων είναι ο ίδιος π.χ. η σειρά αl-β l -γl-δ l -εl-ζl-η l -θ l χρειάζεται 7 κινήσεις του βασιλέα , επίσης η σειρά α1-β2-γ3-δ2-ε1 -ζ2-η 1 -θ1, χρειάζεται επίσης 7 κινήσεις! Δηλαδή είτε κινείται σε ευθεία, είτε κινείται τεθλασμένα, χρειάζεται τον ίδιο αριθμό κινήσεων ! ! Και τώρ α η ώρα των α σ κή σ εων : !)Προσδιορίστε

ση της λύσης; (a,4) , (e,5) , (g,6),(d,7),(h,7),(b,3),(c,8) 2) Αντικατέστησε τους αριθμούς με γράμματα για να σχηματιστούν σκακιστικές λέξεις: (l)ΞΙΩΜΑτΙΚΟΣ, (2)ΑΣΙΛΙΣΣΑ, Ρ70205, 40Α300, 200Α600 3) Αντιστοίχισε τα υπόλοιπα σκακιστικά κομμάτια με κατάλληλο αριθμό στο παρακάτω διάγραμμα (1) 4) Αντικατέστησε τα ερωτηματικά με κατάλληλους αριθμούς στις παρακάτω ισότητες στο διάγραμμα (2)

Γ. «Μα θ η ματικ ά και σ κ άκι», Χρ. Ζ ιώγας-μα θ η ματικός [ΠΡΕΒΕΖΑ] - Μ έρος !', τελευταίο Υ πά ρχε ι αντί θε ση ; Μια αντίθεση μεταξύ μαθητο χρώμα των τετραγώνων (λευκό ή μαύρο) χωρίς ματικών και σκακιού βρίσκεται στη γεωμετρική να βλέπετε την σκακιέρα. Μπορείτε να βοηθήσετε πρόταση ότι η συντομότερη οδός είναι η ευθεία. την μνήμη σας με λογική επεξεργασία και γενίκευ-

��� �� •• • • � � � - - -

διάγραμμα ι

1

? ?

διάγραμμα 2

1111• "Α υ τό το ξέρατε; " [η α π ά ντηση ]

Πρόκειται για την καμπύλη που είναι γνωστή με την ο� μασία κυκλοειδής. Είναι ο γεωμετρικός τόπος των σημείων που ορίζονται από την κίνηση που κάνει ένα σταθερό σημείο της περιφέρειας ενός τροχού, καθώς ο τροχός κυλά κατά ιiJ­ κος μιας ευθείας σ' έναν επίπεδο δρόμο. Αυτή η καμπύλη με τις όμορφες αναλογίες φαίνεται να έχει πρωτοεμφανιστεί στη μαθηματική γραμματολογία στα 1 50 1 από τον Charles Bourelles. Ο Γαλιλαίος τη μελέτησε και υπέδειξε τη χρήση της ως τόξου για γέφυρες. Ο Huygens ( 1 629- 1 695) απόδειξε ότι η ανεστραμμένη κυκλοειδής είναι το ταυτόχρονο, δηλ. η καμπύ­ λη με την εξής ιδιότητα: αν τοποθετήσουμε με τυχαίο τρόπο χάντρες σε διάφορα σημεία της, όλες θα χρειασθούνrον ίδιο y,ρόνο για να κυλήσουν προς το χαμηλότερο σημείο της υπό την επίδραση της βαρύτητας. Λόγω της απλής ομορφιάς, των κομψώ" ιδιοτήτων της και των aτελείωτων διενέξεων που πυροδότησε μεταξύ των μαθηματικών, που προκαλούσαν ο ένας τον άλλ.ο\' για την επίλυση του ενός ή του άλλου προβλήματος που σχετιζόταν μ' αυτήν, ονομάσθηκε η "η Ελένη της Γεωμετρίας" [πηγή : "Οι μαθηματικοί", Ε. Τ. BELL, τόμ. Ι, σ. l 3 3 , Πανεmστημιακές Εκδόσεις Κρήτης] . ΕΥΚΛΕΙΔΗΣ Β ' λζ' τ.2/23


iΓ --· -·-

11

,,

;!

!i

1

: ;::

'! ,I ;:

I

,[ 1 Ι!

i

Μετά από μακρόχρονες προσπάθειες της Ελληνικής Μαθηματικής Εταιρείας υλοποιήθηκε η μόνιμη επιδίωξη μας : τα μέλη των Εθνικών Ομάδων των Μαθηματικών που κατακτούν μετάλλια στους Διεθνείς Μαθηματικούς Διαγωνισμούς (Βαλκανιάδες, Ολυμπιάδες) να εγγράφονται καθ' υπέρβαση του α- 1 ριθμού των εισακτέων σε όλα τα Πανεπιστημιακά Τμήματα όπου τα μαθηματικά εξετάζονται ως μάθη- I 11 μα κατεύθυνσης. Το Υπουργείο Παιδείας κατέθεσε στη Βουλή Πρόταση Νόμου , η οποία έγινε δεκτή παμψηφεί (ΦΕΚ rtil 267/20-1 1 -2003). i1

«Α �

Κάτοχοι απολυτηρίου Ενιαίου Λυκείου, ο ι οποίοι κατά τη διάρκεια της φοίτησής τους στο Λύκειο,

11

έχουν διακριθεί στη ν Βαλκανική ή την Δ ιεθνή Ολυμπιάδα, Μα θημ α τικών, Πληροφορικής, Φυσικής, !i ή Χημείας και τους έχει απονεμεθεί πρώτο, δεύτερο ή τρίτο βρ αβείο (Χρυσό, αργυρό 'η χάλκινο μ� 1

τάλλιο) εγγράφονται κατ ' υπέρβαση του αριθμού εισακτέων σε πα νεπιστημιακά τμήματα, εφόσον για την εισαγωγή τους σε αυτά το μάθημα στο οποίο έχουν διακρι θεί στις Ολυμπιάδες, εξετάζεται ως '

,I :: !i

I

μα' θημα κατευ' θυ νσης. Οι διατάξεις της προηγούμενης περίπτωσης εφαρμόζονται ανάλογα και για τους κατόχους Β ' Κύκλου Σπουδών ΤΕΕ, που έχουν διακριθεί στην Ολυμπιάδα της Πληροφορικής για την εγγραφή τους σε τμήματα ΤΕΙ. Λ"

Η διάκριση βεβαιώνεται από τον αντίστοιχο επιστημονικό φορέα ή ένωση.

Με απόφαση του Υπουργού Παιδείας και Θρησκευμάτων καθορίζονται τα θέματα που έχουν σχέση με την υποβολή αιτήσεων, τα απαιτούμενα δικαιολογητικά και τον έλεγχο αυτών, τη διαδικασία εγγραφής και κάθε άλλη λεπτοl!_έρει�

-1

-

20η Βαλi!-t ανι?{ή J\VΡ,αθη μα'Ή�tκ·�i

C�λυ μ:i: ιάδα

Τl Ψ/�ΝΑ, P.JUE;ANIA , 4 Ι\1Α ΆΟΥ 2003

Υπάρχει σύνολο Β με στοιχεία 4004 θετι­ κούς ακέραιους τέτοιο ώστε για κάθε υπο­ σύνολο Α του Β με 2003 στοιχεία το άθροι-

σμα των στοιχείων του Α δεν διαιρείται με το 2003; Λύση

Η απάντηση είναι ναι και δικαιολογείται ως ε­ ξής:

ΕΥΚΛΕΙΔΗΣ Β ' λζ ' τ.2/24


Μαθηματικοί Διαγωνισμοί - Μαθηματικές Ολυμπιάδ ες

Έστω C ένα σύνολο με 2002 διαφορετικούς θε­ τικούς ακέραιους καθένας από τους οποίους είναι ισοϋπόλοιπος με το 1 mod 2003 , δηλαδή για κάθε η ε C ισχύει ότι η Ξ 1(mod 2003) . Έστω επίσης D ένα σύνολο με 2002 θετικούς ακεραίους καθένας από τους οποίους είναι ισοϋπόλοιπος με το 2 mod2003, δηλαδή για κάθε η ε D ισχύει ότι η Ξ 2(mod 2003) . Ορίζουμε το σύνολο Β = C U D και θεωρούμε τυχόν υποσύνολο Α του Β με 2003 στοιχεία. Έστω ότι το σύνολο Α n D έχει κ στοιχεία, 1 � κ � 2002 , οπότε το σύνολο Α n C θα έχει 2003-κ στοιχεία. Τότε το άθροισμα των στοιχείων του Α είναι: S Ξ (2κ + 2003 - κ)(mod 2003) S Ξ (2003 + κ)(mod 2003);i O(mod 2003) , αφού 1 � κ � 2002 , οπότε το S δεν διαιρείται με το 2003. Π ΡΟΒΛΗΜΑ 2

Έστω τρίγωνο ABC με ΑΒ * AC και έστω D το σημείο στο οποίο η εφαπτομένη του περιγεγραμμένου κύκλου του τριγώνου ABC στο Α τέμνει την ευθεία BC. Έστω α­ κόμη Ε και F σημεία πάνω στις μεσοκαθέ­ τους των πλευρών ΑΒ και AC, αντίστοιχα, τέτοια ώστε οι ΒΕ και CF είναι κάθετοι προς την BC. Να αποδείξετε ότι τα σημεία D,E και F είναι συνευθειακά. Λύ ση

-------

Θα υπολογίσουμε τους δύο λόγους της εξίσω­ σης (1 ). Επειδή η AD είναι εφαπτομένη του περιγε­ γραμμένου κύκλου του τριγώνου ABC, έχουμε DAB = ACB , οπότε τα τρίγωνα DAB και DCA είναι όμοια. Άρα έχουμε: DB ΑΒ AD DA AC DC

-

*

�� = ( � ) 2 = :: '

(2)

όπου έχουμε θέσει ΑΒ c, AC b. Για το δεύτερο λόγο θεωρούμε το μέσο Μ της πλευράς BC. Από το τραπέζιο ΕΒΜΟ με ΕΒΜ = ΒΜΟ = 90° έχουμε ότι ---------ΒΕΚ = 1 80ο - ΕΟΜ = ΚΒΜ = ΑΒC = Β . Ομοίως βρίσκουμε ότι OFC = C . Από τα τρίγωνα ΒΕΚ και CLF έχουμε ΒΕσυνΒ = -c και CFσυνC = -b 2 2 .:_ . συνC * ΒΕ (3). = CF d συνΒ Από τα τρίγωνα ΟΒΚ και OCL έχουμε RσυνΒ = d και RσυνC = -c 2 2 * συνC = .:_ , (4) συνΒ b οπότε από τις (3) και (4 ) έχουμε ΒΕ � = (5). CF b 2 Από τις (2) και (5) προκύπτει η ισότητα (1). =

=

---

,...

-

Π ΡΟΒ Λ Η ΜΑ 3

. Να προσδιορίσετε όλες τις συναρτήσεις f : R -+ R που ικανοποιούν τις επόμενες συνθήκες: (i) f(x + y) - yf(x) - xf(y) f(x)f(y) f(x)f(y) - χ - y + xy , για κάθε x, y Ε Q (ii) f(x) 2f(x + 1) + 2 + χ , για κάθε χ Ε Q και (iii) f(1) + 1 > Ο . =

=

=

=

Τα τρίγωνα DBE και DCF είναι και τα δύο ορθογώνια, οπότε αρκεί να αποδείξουμε ότι είναι Λύση όμοια ή ισοδύναμα αρκεί να αποδείξουμε ότι Θεωρούμε τη συνάρτηση φ(χ): = f(x)+x , χ ε Q , ' DB ΒΕ (1) = η οποία ικανοποιεί σύμφωνα με τα δεδομένα, τις επό­ DC CF μενες συνθήκες: ΕΥΚΛΕΙΔΗΣ Β ' λζ ' τ.2/25


Μαθηματικοί Διαγωνισμοί - Μαθηματικές Ολυμπιάδες

φ( χ + y) = φ(χ)φ(y) , για κάθε χ, y Ε Q , φ( χ) = 2φ(χ + 1) , για κάθε χ Ε Q και (3) φ(l) > ο . Από την (1) για χ = y = Ο έχουμε φ( Ο) = φ(Ο) 2 # φ( Ο) = 1 ή φ(Ο) = Ο . Αν φ(Ο) = Ο , τότε από την (2) έχουμε (1 ) (2)

φ(l) = φ( Ο) = Ο , το οποίο αντίκειται στη συνθή­ (3). Επομένως θα είναι φ(Ο) = 1 και φ(l) = 1/2 . Από την ( 1 ) για x = y = -t έχουμε 2 ' φ(t) = > 0 , αφού φ(α) +" Ο , για κάθε

κη

H�J]

α Ε Q . Πράγματι, αν για κάποιο α Ε Q ίσχυε ότι φ( α) = Ο , τότε φ( χ + α) = φ(χ)φ(α) = Ο , για κάθε χ Ε Q , δηλαδή η φ είναι η μηδενική συνάρτηση, οπότε θα ήταν και φ(l) = Ο , άτοπο. Για χ = y από την (1 ) έχουμε φ(2χ) = [φ(χ)] 2 και με τέλεια επαγωγή προκύπτει ότι: φ(ηχ ) = [φ( χ)] " , για κάθε χ Ε Q και η Ε Ν . Για y = -χ από την (1) έχουμε φ( Ο) = φ(χ)φ(-χ) , οπότε 1 φ(-χ) = -- = [ φ( χ)]- 1 , για κάθε χ Ε Q . φ( χ) Άρα έχουμε = = [φ(χ) Γ " , φ(-ηχ) = φ( χ) [φ( )] " για κάθε χ Ε Q και η Ε Ν , οπότε ισχύει ότι φ(κχ) = [φ(χ)] Κ , για κάθε x E Q και κ Ε Ν . m 'Εστω χ = -, mΕ ηΕ η Τότε η · x = m · l και φ(η · χ) = φ(m · 1) # [ φ( χ)]" = [ φ(l)]m # φ( χ) = [φ(l)] m/n

'71* !LJ

,

I ΞI� = [Η για κάθε χ Ε Q . (1Γ

�τ * 1" .

# φ(χ) =

Άρα έχουμε f( χ) =

- χ , Ύtα κάθε

χΕ

Q.

Π ΡΟΒΛΗΜΑ 4

-------

Έστω ABCD ορθογώνιο με πλευρές μήκους m, η το οποίο έχει χωριστεί σε m · η μονα­ διαία τετράγωνα. Υποθέτουμε ότι οι αριθ­ μοί m και η είναι περιττοί και πρώτοι με­ ταξύ τους θετικοί ακέραιοι. Τα σημεία το­ μής μεταξύ της κυρίας διαγωνίου AC και των πλευρών των μοναδιαίων τετραγώνων είναι διαδοχικά τα Αι, Α2, ...,Ακ με κ � 2 και Αι = Α, Ακ = C . Να αποδείξετε ότι Α 1 Α 2 - Α 2 Α3 + Α3 Α4 - ... + ( - 1)

κ Ακ _ι Ακ = �m 2 + η 2 mη

(1)

Λύ ση

D

Α6�

Α3 /

� � Α-Αι

�As

C=As

Β

Επειδή οι m, η είναι θετικοί ακέραιοι πρώτοι μεταξύ τους, μπορούμε χωρίς περιορισμό της γενι­ κότητας να υποθέσουμε ότι m 2: η . Λέμε ότι το ευ θύγραμμο τμήμα ApAp+t είναι τύπου 1 , αν τα σημεία Αρ και Αρ+ι είναι σημεία τομής της AC με κατακόρυφες πλευρές του πλέγματος (στο σχήμα είναι τα ΑιΑ2,ΑιΑs και ΑιΑs και A1As). Αν το σημείο Αρ ή το Αρ+ι είναι σημείο τομής της AC με οριζόντια πλευρά του πλέγματος, τότε το ApAp+t λέμε ότι είναι τύπου 2. Συνολικά υπάρχουν m- 1 σημεία τομής της AC με κατακόρυφες γραμμές του πλέγματος και η- 1 σημεία τομής με οριζόντιες γραμμές του πλέγ­ ματος. Επειδή είναι (m,η) 1 , όλα αυτά τα σημεία είναι διαφορετικά μεταξύ τους και χωρίζουν την AC σε m+η- 1 τμήματα .. Επειδή ο πρώτος όρος του αθροίσματος του πρώτου μέλους της ισότητας ( 1) έχει πρόσημο + και ο αριθμός m+η- 1 είναι περιτ­ τός, έπεται ότι το πλήθος του αθροίσματος με πρό­ =

σημο

+

υπερβαίνει κατά 1 το πλήθος των όρων με

πρόσημο -.

ΕΥΚΛΕΙΔΗΣ Β ' λζ' τ.2/26


Μ αθηματικοί Δ ιαγωνισμοί - Μ αθηματικές Ολυμπιάδες

Σημειώνουμε ότι αν το Αρ είναι σημείο τομής του AC με οριζόντια γραμμή του πλέγματος, τότε τα τμήματα Α Ρ_ 1 Α Ρ και ApAp + Ι είναι τύπου 2 και έχουν αντίθετα σημεία. Επομένως το πλήθος των τμημάτων τύπου 1 με πρόσημο + είναι κατά ένα μεγαλύτερο του πλήθους των τμημάτων τύπου 1 με πρόσημο -. Όλα τα τμήματα τύπου 1 είναι ίσα μεταξύ τους και έχουν μήκος ίσον προς �m2 + η 2 /m , οπότε τα τμήματα τύπου 1 συνεισφέρουν συνολικά στο άθροισμα τον αριθμό �m2 + η2 /m . Για σταθερό κ 1 ,2, ... ,η-1 γράφουμε tκη + rκ , με Ο < rκ < η . κm Τότε τα σημεία τομής της AC με την οριζό­ ντια γραμμή με αριθμό κ (δεν μετράμε την ΑΒ) έχει συντεταγμένες =

=

[

'·

+

Επιπλέον rκ και κ + tκ είναι ταυτόχρονα άρ­ τιοι ή περιττοί. Πράγματι, αν ο κ είναι άρτιος, τότε και ο tκη + rκ είναι άρτιος, οπότε οι rκ και tκ είναι ταυτόχρονα άρτιοι ή περιττοί. Ομοίως, αν κ είναι περιττός, τότε και ο tκη + rκ είναι περιττός, οπότε οι tκ και rκ είναι ένας άρτιος και ο άλλος περιττός. Έτσι σε κάθε περίπτωση οι rκ και κ+tκ είναι ταυτόχρονα άρτιοι ή περιττοί. Επομένως, όταν rκ είναι άρτιος, τότε -Α S- 1 ΑS + Α S Α S+ l - �m2 + η2 -

ενώ όταν rκ περιττός, τότε

.----

-ΑS- 1 Α S + Α S Α S+ l - �m 2 + η2 -

=

64 ο ς

m

m

[ rκ + (ηη - rκ) ] '

[ rκ + (ηη - rκ) ]

: ' κ] ' (2)

ως προς το σύστημα συντεταγμένων με αρχή το Α και θετικούς ημιάξονες ΑΒ και AD, ενώ το ση­ μείο (2) έχει αριθμό s κ + t κ - 1 . Αυτό έχει ως συνέπεια ότι το A 8_ 1 As έχει πρόσημο - και το Α 8 Αs+ Ι έχει πρόσημο + , αν κ+tκ περιττός.

-------

=

�m2 + η 2 m

[ι - η - 1 ] η

=

�m2 + η2 mn

ΠΑΝΕΛΛΗΝΙΟΣ ΜΑΘΗΤΙΚΟΣ ΔΙΑΓΩΝΙΣΜΟΣ ΣΤΑ ΜΑΘΗΜΑΤΙΚΆ ''Ο ΘΑΛΗΣ", ΣΑΒΒΑΤΟ, 1 ΝΟΕΜΒΡΙΟΥ 2003

Επιτροπή Διαγωνισμών Α ' ΛΥΚΕΙΟΥ 1.

Τ ο τετράγωνο ενός αριθμού ισούται με τον αριθμό αυξημένο κατά 72. Επιπλέον, αν από το 60 αφαιρέσουμε το δι­ πλάσιο του αριθμού λαμβάνουμε αριθμό μικρότερο του 52. Να βρεθεί ο αριθμός. Λύση

χ 2 = χ + 72 ( 1) 60 - 2χ < 52 (2) Από την ( 1) έχουμε: χ 2 - χ - 72 = Ο με διακρίνου σα Δ = (- 1)2 4 1 ( -72) = 1 + 288 = 289 = 1 72 , οπότε η εξίσωση έχει τις ρίζες χ = 1 ± 17 = Ι 9 2 \-8 Όμως από την (2) έχουμε ότι: -

·

·

--

Σύμφωνα με τις υποθέσεις του προβλήματος, αν χ είναι ο ζητούμενος ο αριθμός, θα έχουμε:

ΕΥΚΛΕΙΔΗΣ Β ' λζ' τ.2/27


Μαθηματικοί Διαγωνισμοί - Μ αθηματικές Ολυμπιάδες

60 - 2χ < 52 <:::::> -2χ < 52 - 60 <:::::> -2χ < -8 <:::::> 2χ > 8 <=:> χ > 4 Άρα ο ζητούμενος αριθμός είναι ο 9 . 2.

Αν

x,y,a,b είναι

-------

ΑΕ , οποτε , , θα ειναι: , ομοιοτητας ΕΓ (ΑΕΒ) = ΑΕ 2 <:::::> ΑΕ 2 = 72 <:::::> 50 ΕΓ (ΔΕΓ) ΕΓ

( ) ( )

<:::::>

θετικοί πραγματικοί α­

: = � = � = �.

ioc------7il '

ριθμοί τέτοιοι ώστε

χ :;ι!: y,χ :;ι!: 2y,y :;ι!: 2x,a:;ι!: ±3b 2x - y 2y - x και -- = -- = λ, a + 3b a - 3b

να αποδείξετε ότι : i) χ + y 2λa και χ - y = 2λb

Β

Α

=

χ2 + Υ 2 � 1 . χ 2 -y 2

Τα τρίγωνα ΑΕΔ και ΔΕΓ έχουν κοινό ύψος από την κορυφή Δ, οπότε (ΑΔΕ) = ΑΕ = � => Λύση 2x - y 2y - x (ΔΕΓ) ΕΓ 5 ι.) 'Εχουμε -- = -- = λ , a + 3b a - 3b => (ΑΕΔ) = � (ΔΕΓ) = � · 50m2 = 60m 2 . Τότε 5 5 2x - y = λa + 3λb (1) ΒΕ 6 , λαμβ ανουμε ' Ομοιως -=-χ + 2y = λa - 3λb (2) ΕΔ 5 2 , οπότε και (ΒΕΓ) = 60m από τις οποίες με πρόσθεση και αφαίρεση κατά (ΑΒΓΔ)=(ΑΕΒ)+(ΒΕΓ)+(ΓΕΔ)+(ΔΕΑ)= μέλη λαμβάνουμε: χ + y = 2λa χ + y = 2λa (3) +60+ 50+60=242m2 . 72 <:::::> 3χ - 3y = 6λb χ - y = 2λb (4) 4. Ν α βρεθούν οι ακέραιοι α, β για τους οποί­ ίί) Από τις (3) και (4) με πρόθεση και αφαίρεση ους ισχύει η ισότητα κατά μέλη λαμβάνουμε αβ 2 + 2αβ + α = 2β 2 + 4β + 3 2χ = 2λa + 2λb χ = λa + λb χ = λ(a+ b) <:::::> <:::::> , Λύση 2y = 2λa - 2λb y = λa � λb y = λ(a - b) Έχουμε οπότε θα είναι αβ 2 + 2αβ + α = 2β 2 + 4β + 3 x 2 + y2 = λ\ a + b) 2 + λ2 (a - b) 2 <:::::> α(β 2 + 2β + 1) = 2(β 2 + 2β) + 3 x 2 - y 2 λ2 (a + b) 2 - λ2 (a - b) 2 <:::::> α(β 2 + 2β + 1) = 2(β 2 + 2β + 1) + 1 2 2 2 a 2 + b2 >- λ · 2(a + b ) = --<=:> (α - 2)(β + 1) 2 = 1 2ab λ2 · 4ab α ,βεΖ οπότε αρκεί να είναι <:::::> α - 2 = 1 και (β + 1) 2 = 1 (αφού (β + 1) 2 � Ο) 2 2 a +b --� 1 <:::::> a 2 + b 2 - 2ab � Ο <:::::> (a - b) 2 � Ο, <:::::> (α- 2 = 1 και β+ 1 = 1) ή (α- 2 = 1 και β+ 1 = -1) 2ab <=:>(α = 3 και β = Ο) ή (α= 3 και β= -2) που ισχύει. ii)

}

=

}

}

--::---=--::-

}

----'-.,.----::-.:..._----'----:'--:-' _ -::-

a,b>O

3.

Σε τραπέζιο ΑΒΓΔ ( ΑΒ I ΓΔ ) οι διαγώνιες

τέμνονται στο Ε. Αν είναι (ABE)=72m 2 και (ΓΔ Ε)=50 m 2 , να υπολογίσετε το εμβαδόν

του τραπεζίου ΑΒΓ Δ. Λύση

Τα τρίγωνα ΑΕΒ και ΔΕΓ είναι όμοια με λόγο

Β ' ΛΥΚΕΙΟΥ Ι.

Δίνεται οξυγώνιο τρίγωνο ΑΒΓ και δύο τυ­ χαία σημεία Α 1 πάνω στη πλευρά ΒΓ και Β 1 πάνω στη πλευρά ΑΓ. Ονομάζουμε C 1 και C 2 τους περιγεγραμμένους κύκλους των τριγώνων ΒΑ 1 Β 1 , ΓΑ 1 Β 1 , αντίστοιχα.

ΕΥΚΛΕΙΔΗΣ Β' λζ' τ.2/28


Μ αθηματικοί Διαγωνισμοί - Μ αθηματικές Ολυμπιάδες

ευθεία ΑΒ τέμνει τον Cι στο Γι , ενώ η ευθεία ΓιΒι τέμνει τον C 2 στο Δ. Να απο­ δείξετε ότι τα ορθόκεντρα των τριγώνων ΑΒι Γι και ΔΒι Γ είναι σημεία συνευθειακά με το Βι .

Η

2Α2 + 2Β2 + 2r = 2ΑΒ+ 2ΒΓ+ 2ΓΑ (Α - Β)2 + (Β - Γ)2 + (Γ- Α)2 = Ο � Α = Β = Γ (1) Είναι 2 2 Α = Β <::} χ - kx + m = x + mx - k <::::} (m + k)x = m + k m+k::;tO χ=1 <::::} χ2 + rnx - k = m2 x2 + (k - 1)x + k Β=Γ

Λ ύση

Επειδή το τετράπλευρο Α 1 Β 1 ΔΓ είναι εγγεγραμμένο στον C 2 ισχύει: ΒιΔΓ = ΒιΑιΒ (1). Επειδή το τετράπλευρο Α 1 Β 1 Γ1 Β είναι εγγεγραμμένο στον C 1 ισχύει: ΒιΓιΑ = Β 1 Α 1 Β (2). Από τις (1), (2) συμπεραίνουμε ότι: (3).

--

{ !

--

Α

-------

και <::::}

και

χ=1

χ=1

2 1 + m-k = m +k - 1 +k 2 <::} m -m+ 3k - 2 = 0 <::::}

(2) Από την ισότητα Α = Γ και για χ = 1 , καταλήγου­ με και πάλι στη σχέση (2) . Επειδή m Ε R είναι: Δm ;:::: ο � (-1) 2 - 4 · 1 · (3k - 2) ;:::: ο � 1 - 12k + 8 ;:::: ο � 12k :ς 9 � k :ς �4

C2

c,

Άρα, η μέγιστη δυνατή τιμή του k Ε R είναι: k . Για την τιμή αυτή του k , η εξίσωση (2)

=%

Λόγω της (3) είναι ΑΒ // ΓΔ (αφού οι ευθείες αυτές σχηματίζουν με τέμνουσα την fΊΒ 1 Δ δύο γράφεται: 1 1 9 m 2 - m + - - 2 = 0 <=> m 2 - m + - = O <::::> m = εντός - εναλλάξ γωνίες ίσες.) Κατά συνέπεια, τα 2 4 4 ύψη των δύο τριγώνων που άγονται από την κοινή 3 . Σε αμβλυγώνιο τρίγωνο ΑΒΓ ( Α > 9 0· ) φέκορυφή τους Β1 , αποτελούν μία ευθεία. ρουμε ημιευθεία Αχ κάθετη προς την πλευΤο ζητούμενο έχει αποδειχθεί, αφού τα ορθόρά ΑΒ που τέμνει την πλευρά ΒΓ στο Δ. κεντρα των τριγώνων αυτών βρίσκονται στην ευ­ Από το Δ φέρουμε παράλληλη προς την ΑΓ θεία αυτή. 2.

Δίνονται οι παραστάσεις Α = χ 2 - kx + m, Β = χ2 + mx - k, Γ = m 2 x 2 + (k - l)x + k, με k,m Ε IR και k + m ::;t Ο . Αν είναι Α 2 + Β 2 + Γ 2 = ΑΒ + ΒΓ + ΓΑ, να βρείτε την τιμή του χ, την μέγιστη τιμή του k και την τιμή του m που αντιστοιχεί στη μέγιστη τιμή του k. i) Λ ύση

Έχουμε Α 2 + Β 2 + Γ 2 = ΑΒ + ΒΓ + ΓΑ �

που τέμνει την πλευρά ΑΒ στο Η και τη διάμεσο ΒΜ του τριγώνου ΑΒΓ στο Ε. Από το Ε φέρουμε παράλληλη προς την ΑΒ που τέμνει την ΒΓ στο Ζ. Να αποδείξετε ότι: (ί) η ΕΖ είναι διχοτόμος της γωνίας ΑΖΔ και (ίί) Γ = ΖΑΕ . �

Α

Λ ύση

Επειδή το Μ είναι μέσον της ΑΓ και ΔΗ/IΑΓ, θα είναι και το Ε μέσον της ΔΗ. Επειδή το Ε είναι το μέσον της ΔΗ και ΕΖ//ΒΗ, θα είναι και το Ζ μέσον της ΒΔ. Στο ορθογώνιο τρίγωνο

ΕΥΚΛΕΙΔΗΣ Β ' λζ' τ.2/29


Μαθηματικοί Διαγωνισμοί - Μ αθηματικές Ολυμπιάδες

α + b + c = χ 2 + y2 + z 2 - xy - yz - zx = = __!_ [(x - y) 2 + (y - z) 2 + (z - x)2 ] � 0 · 2

ΑΖ είναι διάμεσος προς την υποτείνου­ σα, οπότε θα είναι ΑΖ=ΖΒ=ΖΔ και Λ 1 = ΑΕΖ .

ΑΒΔ

-------

η

Α

Γ ' ΛΥΚΕΙΟΥ

1. Β

Γ

(i) Να προσδιορίσετε το σύνολο των σημείων Μ του επιπέδου που είναι εικόνες των μιγαδικών αριθμών z = χ + yi με χ, y IR και ικανοποιούν την ισότητα i(z + z α) + z z = O, όπου α e iR . (ii) Αν Α να προσδιορίσετε σημείο Β Cα τέτοιο ώστε η μεσοκά­ θετος του ΑΒ να διέρχεται από το κέ­ ντρο του κύκλου Κμ με εξίσωση lz + 1 - 3il = β, β > Ο. Για ποια τιμή του β , ο κύκλος Κ μ εφάπτεται του c α ;

Ε

- (-�,%)Εcα, Ε

Όμως είναι και Ζ 1 = Λ 1 (εντός εναλλάξ, ΕΖ//ΑΒ) Ζ2 = ΑΒΖ (εντός - εκτός και επί τα αυτά, ΕΖ//ΑΒ). Άρα θα είναι και Ζ 1 = Ζ 2 , οπότε η ΕΖ είναι διχοτόμος της γωνίας ΑΖΔ . ίί) Τα τρίγωνα ΑΕΖ και ΔΖΕ είναι ίσα γιατί έχουν τη πλευρά ΖΕ κοινή, Ζ 1 = Ζ2 και ΑΖ=ΖΔ. Άρα θα έχουν και ΖΛΕ = Δ Όμως Δ1 = Γ , ως εντός εκτός και επί τα αυτά Λύση στις παράλληλες ΔΗ και ΑΓ με τέμνουσα την ί) Έχουμε z + z = 2x και z - z = 2yi , οπότε η ΒΓ. Άρα θα είναι και ΖΛΕ = Γ . δεδομένη ισότητα γίνεται ί(2χ - α) + 2yi = Ο <:=:> ί(2χ + 2y - α) = Ο 4. Από τους αριθμούς x,y,z IR δύο είναι αρ­ <:=:> 2χ + 2y - α = Ο νητικοί και ένας είναι θετικός. <:=:> x + y = -α Να αποδείξετε ότι : 2 2 xy)2 (y-z)(Y yz)2 (x-y)(x ...:.__::_:....:. ._ ._ ..::....:._ + + οπότε το σύνολο Ca είναι μια ευθεία του επιπέ­ zx yz δου με εξίσωση χ + y = � . (z x)(z2 xz)2 �3(x-y)(y-z)(z-x) 2 + xy ίί) Επειδή Α - , Ε c α θα είναι Λ

-

• .

-

Ε

_

-

-

Λύση

Επειδή είναι xyz>O, αρκεί να αληθεύει η ανισό­ τητα: x\x - y)3 + y3 (y - z)3 + z3 (z - x)3 � 3xyz(x - y)(y - z)(z - χ) <:=:> (χ 2 xy)J + (y2 yz)J + (z2 zx)J � 3(χ 2 - xy)(y2 - yz)(z2 - zx). Αν θέσουμε α = χ 2 - xy, b = y 2 - yz και c = z 2 - zx , τότε (1) <:=:> α3 + b3 + c3 � 3αbc <:=:> α3 + b3 + c3 - 3αbc � Ο <:=> _!_ (α + b + c)[(α- b)2 + (b - c)2 + (c- αΥ ]� Ο; 2 που ισχύει γιατί _

_

_

( � %)

--1 + -3 = -α <:=:> α = 2 . 2 2 2 Έστω Β( κ, μ). Το Β είναι το συμμετρικό του Α ως προς την ευθεία δ που περνάει από το κέ­ ντρο του κύκλου Ι<ιJ, δηλαδή το σημείο Γ(- 1 ,3), και είναι κάθετη προς την ευθεία C α : χ + y = 1 . Η εξίσωση της ευθείας δ είναι y - 3 = 1(x + 1) <:=:> y = x + 4, αφού είναι λδ λc. = -1 => λδ = 1 . Η τομή των Ca και δ είναι· το σημείο ·

Ρ(- % , %) , οπότε θα είναι:

ΕΥΚΛΕΙΔΗΣ Β ' λζ ' τ.2/30


Μ αθηματικοί Δ ιαγωνισμοί - Μ αθηματ ικές Ολυμπιάδες

3

3 κ - -1 μ+- 5 5 7 2 = -- και --2 = - <=> κ = -- μ = - . __ 2 2 2 2 2' 2

-------

2ος Τρόπος

--Στα τρίγωνα ΡΓΒ, ΡΒΔ η ΓΡΒ κοινή, στα τρίγωνα ΡΓΑ, ΡΑΔ η ΑΡΓ κοινή άρα: Άρα είναι Β - , (ΡΒ Γ) ΡΓ . ΡΒ ΡΓ = = (ΡΔΒ) ΡΔ · ΡΒ ΡΔ � (ΡΒΓ) (ΡΑΓ) = Ο κύκλος � εφάπτεται της ευθείας Cα όταν ΡΓ (Ρ ΑΓ ) ΡΑ · ΡΓ _ (Ρ ΔΒ) (ΡΑΔ) -'--_� = = είναι (ΡΑΔ) ΡΑ · ΡΔ ΡΔ l-1 + 3 - 1 1 <=> 1 .J2 d ( Γ, ε ) = β <:::> = β - = β <:::> β = -. (ΡΑΓ) (ΡΒΓ) = � � 2 .J2 �

( % �).

-

(Ρ Δ_Β) - (Ρ Β Γ)

2.

I

,

Από σημείο Ρ εκτός κύκλου φέρουμε τις εφαπτόμενες ΡΑ, ΡΒ και τυχαία τέμνουσα προς τον κύκλο. Αν ισχύ ει ΡΓΔ 2(ΡΑΓ)=3(ΡΒΓ), να υπολογίσετε το λόγο (ΒΔΓ)

(ΑΔΓ) Λ ύ ση

Από την ομοιότητα των τριγώνων ΡΑΔ, ΡΑΓ παίρνουμε: 2 (ΡΑΓ) = ΡΑ (1) (ΡΑΓ) ΡΓ Από την ομοιότητα των τριγώνων ΡΒΔ , ΡΒΓ παίρνουμε: 2 (Ρ ΒΔ) = ΡΒ (2) ΡΓ (ΡΒΓ) Επειδή ΡΑ=ΡΒ, από τις (1), (2) έχουμε:

( )

(Ρ ΑΔ) - (ΡΑΓ)

(Ρ Β Γ) (Ρ ΑΓ) = � � -(ΒΔΓ)

3.

(ΑΓΔ)

(ΒΔΓ) (ΑΔΓ)

= (ΡΒΓ) = -23 (ΡΑΓ)

Δίνεται τρίγωνο ΑΒΓ με Β > 90° και πλευ­ ρά B C=R, όπου R είναι η ακτίνα του περι­ γεγραμμένου κύκλου του. Αν Ε είναι το εμβαδόν του τριγώνου και μα η διάμεσος που αντιστοιχεί στην πλευρά δείξετε ότι μ α2 = ! R 2 + 2EJ3 4

ΒΓ,

να απο­

Λ ύ ση

( )

(ΡΑΔ) (ΡΒΔ) (ΡΑΙ) (PBI)

-- =-- <=>

(ΡΑΔ)--(ΡΑΙ) (ΡΑΙ)

= (ΡΒΔ)- (ΡΒΓ) <=> (ΡΒΓ)

(ΑΔΓ) -(ΒΔΓ) (ΡΒΓ) (ΒΔΓ) 2 -= -- � -= (ΒΔΓ) <=> -=3 (ΡΑΓ)

(ΡΒΓ)

(ΑΔΓ)

(ΡΑΓ)

(ΑΔΓ)

-

Είναι a = R . Από τον νόμο των η μιτqνων παίρνουμε: 2R sin Α = R <:::> sin Α = _!_ και επειδή η γωνία 2 Α είναι κατ ανάγκην οξεία, έχουμε: Α = "6π , οπόΛ

J3 . τε cos A = 2 Από τον νόμο των συνη μιτόνων, παίρνουμε: a 2 = b 2 + c 2 - 2bc cos A = b 2 + c 2 - 2bc J3 2 2 2 = b + c - bc-/3 (1) ΕΥΚΛΕΙΔΗΣ Β' λζ' τ.2/3 1


Μαθηματικοί Διαγωνισμοί - Μαθηματικές Ολυμπιάδ ες

ι 1 1 MKΔ(x, y, z) = 1 και - + - = - , χ Υ z να αποδείξετε ότι ο αριθμός χ + y είναι τέ­ λειο τετράγωνο.

από το πρώτο θεώρ η μα των διαμέ σ ων έχουμε: a2 b 2 + c 2 = 2μ; + l (2)

Από τις (1), (2) προκύπτει: a2 a 2 = 2μ ; + - - bcJ3 � 2 .!.. a 2 = 2μ ; - bcJ3 � a 2 = 4μ; - 2bcJ3 (3) 2 Από τον τύπ ο του εμβαδού τριγώνου είναι: abc = 4ER � Rbc = 4ER � bc = 4E (4) Από τις (3), (4) έχουμε (είναι α=R): 4μ ; = R 2 + 8EJ3 (αφού α = R) � μ� = .!_ R 2 + 2EJ3 4

4.

Αν x, y, z είναι θετικοί ακέραιοι με

-------

Λύση

Αν d = (χ, y) , τότε θα αποδείξουμε ότι x + y = d2 • Πράγματι, αν χ = da , y = db , τότε ( a, b) = 1 και _!_ + _!_ = .!.. .!.. + = .!_ , οπότε χ y d a b z a + b = -d -1 + -1 = -d � (1 ) a b z ab z Όμως (d, z) = 1 και (a + b, ab) = 1, οπότε από την (1) έπεται ότι a + b = d και ab = z . Άρα θα είναι x + y = d(a + b) = d 2

( -.!.)

r Ολυμπ ιακές ΙΙροσεγγίσεις

Σωτήρης Ε. Λουρίδας

Συμβολισμοί Αθροισμάτων και οι Μαθη ματικές Συμπεριφορές τους.

Ο συμβολισμός των αθροισμάτων και των γινομένων αλλά και η μαθηματική τους συμπεριφορά είναι

μια αναγκαιότητα αφού διευκολύ νουν την διαδικασία μελέτης και επίλυσης προβλημάτων του είδους, όταν μάλιστα είμαστε αναγκασμένοι να « συμμαζέ ψουμε» αθροίσματα ή γιν όμενα με πολύ μεγάλο έως «άπε ιρο πλή θος» όρων. Θεωρού με, λοιπόν σημαντική την γνώση και τη μαθηματική συμπεριφορά των συμβολισμών αυτών αφού μας aπεγκλωβίζει από τεχνικές δυσκολίες που πιθανό ν να προέκυπταν. Τέτοιους συ μβολισμού ς έχουμε κατά καιρούς σε διεθνείς μαθηματικούς διαγωνισμού ς π.χ. πρόβλημα της 1 &ς Β.Μ Ο., πρόβλημα που τέθηκε στην τελευταία Ι. Μ Ο. στο Τόκιο το 2003 κ. α.

1.

Συμβολ ι σ μοί

1.1

Έστω το άθροισμα των η-πλήθους προσθε­ τέων α, + α2 + ... + αη τότε συμβολίζουμε η Σ α; = α, + α 2 + ... + αη .

Σ f, (α, , α 2 , ••• , αη ) για παράδειγμα αν α,β,γ πλευρές τριγώνων τότε Σ α(β - γ) = α(β - γ) + β(γ - α) + γ(α - β). Έστω το γινόμενο των η-πλήθους παραγό­ ντων α, · α 2 αη τότε συμβολίζουμε Π α; = α, · α2 αη .

i=l

• • •

I

1.2

Δεχόμαστε ότι Σ α; = α, .

I

• • •

i=l

1.3

Έστω το άθροισμα των απείρων όρων α, + α 2 + ... + αη + . .. τότε συμβολίζουμε 00

Σ α; = α, + α 2 + ... + αη + ... .

1.5

Δεχόμαστε ότι: η Π α; = α, (Άρα · Σ α; = Π α; = α, ) . i=l

i=l

1.4

i=l

Έστω ότι έχουμε f, (α, , αυ ... , αη ) + f2 (αυ α3 , . • . , αη , α, ) + ... + fη ( αη , α, , αυ ... , αη-ι ) , τότε συμβολίζουμε

I

I

i=l

i=l

1. 6 Έστω το γινόμενο των απείρων όρων

α, · α 2 •••αη ...

Τότε συμβολίζουμε Π α; = α, · α2 •••αη ... 00

i=l

ΕΥΚΛΕΙΔΗΣ Β ' λζ' τ.2/32


Μ αθηματικο ί Διαγωνισμοί - Μ αθηματικές Ολυμπιάδες

2.

2.ι

η η " η (ρ ι ···ρ 2 ...ρ3 ) · Π α; .π β; ...π Ύ;

Ιδιό τητ ε ς

η

Σc = η · c όπου c σταθερός αριθμός i=l

η

=ι. Ο για κάθε i Ε {1 , 2, . .. , η} τότε ισχύει " α α . = __!!:t!. ότι: Π ---!±!. . αι α;

2.ιο. Αν α ;

2.2.

i=l

η

η

i=l

i=l

i=ι

Σ <αί + β ; ) =Σ αί + Σβ ί � i=l

η

Π αρατη ρή σεις

η

i=l

2.3. (Ο μογένειας). Αν c σταθερός αριθμός τότε η

2 .4. Σ (c 1 · α; + c2 · β ; + ... + c, i=l

η

η

i=l

i=l

Ύ; )

·

i=l

i=

i=l

ρ

t. (t. )

Θέματα

" 2. 7 . Π c = c , όπου c σταθερός αριθμός, αφού i=l

η

α1 = α 2 = . .. = α" = c . Άρα Π 1 = 1 η

η

i=l

Να αποδειχθεί ότι:

i=l

i=l

i=l

i=l

Σ 1·3 = i=O

.

Π (α; · β ; ) = Π α; · Π β ; γενικά έχουμε η

η

η

π

i=l

i=l

Π (α; · β ; ···γ ; ) = Π α; ·Π β ; ···Π γ; · .

η

2.

ο3

-

2

+

oz

-.

4

η

2 η 2 (η + 1) 2 η 2 (η 2 + 2η + 1) = = Σ 1· 4 4 i=O

Αν α 1 ,α2 , . . . , «π θετικοί άνισοι αριθμοί. Να αποδείξετε ότι: η

Σ1=1 α . " Σ i=t Σ α . - α . i=l ι

η

ι

ι

i=l

"

+

Α πό δειξ η

( )

η α π α. Ορίζουμε επίσης Π _ι. = � . βί π βί η

4

= -

i=l

η

n

Σ ί 3 η4 i=O η

ι.

i=l

i=l

Γενικά έχουμε: Π [(ρ ι · α; ).(ρ 2 · β ; )... (ρ3 · γ; )] =

>

ι

0

2

ο-ι

--

.

Α πό δειξ η

i=l

Π ρ · α; = ρ Π α; , όπου ρ σταθερός αριθμός.

i=l

i=O

i=l

= α2 + α3 + . . . + α π - ι + α" ) -( α 1 + α2 + . . . + α π - ι ) = ... = α" - α1 2.6. Αν α; :::; β ; για κάθε i Ε {1, 2, .. . , η} τότε

η

i=l

η+ρ- 1

=

-

i=l

'

n-1

i=l

i=l

i=l

n=l

i

n-1

2.9.

η

i

t.

i=l

Σ (α;+ι - α; ) = α" - α 1 , αφού n- 1 n- 1 Σ <α Η α ; ) Σ α;- ι - Σα; =

η

λ= \

2. αι + α 2 + ... + α " = Σ α; = Σ α;+ι = Σ αi -ρ + Ι . i=l

η

n-1

2.8.

i=l

η

λ

η επιλογή του δείκτη είναι αυθαίρετη.

=

2.5. (τη λεσκοπική ιδιότητα)

η

η

) = η(η + 1) , > 2 = η(η + 1)(2η + 1) , 3• 6 2 2 2 + 2 i 3 η (η 1) . 'Ετσι έχουμε: i 3 = i , 4 >4 = η(η + 1)(2η + 1)(3η 2 + 3η - 1) . 30 4. Για την γενική μορφή πολυωνύμου η-βαθμού έχουμε:

= cι Σ α; + c 2 Σβ ; + ... + cκ Σγ; ,

i=l

η

ι. α1 + α2 + ... + αν = Σα; = Σ α = Σ α" δηλαδή

Σ (α; +β; ) = Σ (β; + α; ). i=l

·

i=l

i=l

i=l

( α1 = α2 = ... = α" = c ) . Άρα Σ 1 = η . η

-------

Από το θεώρημα Αριθμητικού - Γεωμετρικού Μέσου έχουμε:

t. (t. α; α; ) > η u(t. αj ) -

ΕΥΚΛΕΙΔΗΣ Β ' λζ' τ.2/33

π

- α;


Μαθηματικοί Διαγωνισμοί - Μαθηματικές Ολυμπιάδες

(1)

4.

Επίσης από το θεώρημα Αριθμητικού - Γεωμετρικού Μέσου έχουμε: 1 Σ Σα - α (2) η --

i=l

i

Να βρεθεί τύπος που να δίνει το άθροισμα Σπ αϊ αν ο τυχαίος - γενικός όρος δίνεται από τον τύπο: α π =α · f(η)+β · f(η+ 1)+γf(η+2) με α+β+γ=Ο, όπου f(η) ακολουθία του φυσι­ κού αριθμού η.

η

n

Λύση

i

η

η

Σ α " = Σ <α · f(η) + β · f(η + 1) + γ · f(η + 2)) i=l

i=l

i=l

i=l

α i . Αν ο τυχαίοςΣ i=ο γενικός όρος απ γράφεται σαν απ =f(η)--f(η+1) όπου f(η) ακολουθία του η τότε: Σπ α = f(1) - f(η + 1) . π i=O

Αφού απ = f(η) - f(η + 1) έχουμε Για η = 1 => α, = f(1) - f(2) Για η = 2 => α 2 = f(2) - f(3)

Λύση

i=l n-2

n=l

n=l

+f(η + 1)) + γ(Σf(η + 2) + f(η + 1) + f(η + 2)) n=l

= α(f(l) + (α + β)f(2) + +(β + γ)f(η + 1) + γf(η + 2) + (α + β + γ) n-2

η

n=l

n=l

Σ f(η + 2) . Άρα Σα" = α · f(l) - γ · f(2) -α . f(η + 1) + γ · f(η + 2) . και ταυτόχρονα

π

Π x i = α π . Να αποδειχθεί ότι: i=l

π

Π (1 + xi ) � (1 + αΥ . i=l

Απ όδε ιξη

η

Παρατηρούμε ότι: 1 1

i=l n-2

5. Έστω xi>O με ί=1,2, . . . ,η

Για η = η => α" = f(η) - f(η + 1) . Με πρόσθεση κατά μέλη των ισότητων αυτών έχουμε: Σ απ = f(l) - f(η + 1). Να βρεθεί τύποςη που να δίνει το Σ 2η + 3 i=O (η 2 + 3η + 2) 2 •

η

n-2

Απ ό δ ειξη

Π αράδ ειγμα

i=l n-1

= α ( f(l) + f(2)) + Σf(η + 2)+ β(f(2)+ Σ f(η+ 2)+

π

i=O

η

= αΣ f(η + 2) + βΣ f(η + 2) + rΣ f(η + 2) = i=l

i=O

Έστω το άθροισμα

η

n-2

η

3.

η

= αΣ f<η) + βΣ f<η + 1) + rΣ f<η + 2) =

Αν πολλαπλασιάσουμε κατά μέλη τις (1) και (2) έχουμε το ζητούμενο

Παρατήρηση: Το Σ α i είναι σταθερός αριθμός.

-------

Από την μορφή της γενίκευσης της ανισότητας του Hδlder για τους θετικούς α1 , α2 , β 1 , β2 , . . . , γ1 , γ2 και θετικούς h ψ, . . . ,z με χ+ψ+ . . . +z= 1 ισχύει: χ (αι + α2 ) · (βι + β2 Υ . . . (γι + γ 2 Υ > χ Υ χ Υ - αι · β ι · · ·γι + α2 · β 2 · · · γ2 , ) 'Εχουμε: -1 + -1 + . . . + -1 = 1 . (η-οροι η η η z

z

2η + 3 i=l

i=l

η

I

=> Π Ο + χ i ) � � ( 1 + α )

i=l

"

i=l

6.

Έστω τρίγωνο ΑΒΓ να αποδείξετε ότι:

ΕΥΚΛΕΙΔΗΣ Β ' λζ' τ.2/34

Σ βγ(β + γ) � �Π (β + γ - α) ·


Μ αθηματικοί Διαγωνισμοί - Μ αθηματικές Ολυμπιάδες

-------

Απόδειξη

Απόδειξη

αy

Παρατη ρού με ότι (αχ + β ) Ξ ( + β) mοdμ � Αφού α, β ,γ είναι πλευρές τριγώνου θα είναι α(χ - y) Ξ modμ � χ - y Ξ omodμ � χ Ξ ymodμ . θετικοί αριθμοί. Για τους τυχαίους έχουμε: Προφανώς έχουμε σχέση ισοδυναμίας που δη­ 2 μιουργεί διαμέριση. ( -Γχ - .[y) Άρα οι αριθμοί ακ + β , κ = Ο, 1, ... , μ - 1 διαιΆρα: ρούμενοι με το μ δίνουν διαφορετικά υπόλοιπα β = 6α γ . κ = 0,1, 2, ... , μ - 1 . ακ + β ακ + β Ό μως: α2 � α2 - (β - γ)2 = (α + β - γ)(α - β + γ). με μ μ μ Άρα: Ui τα αντίστοιχα υπόλοιπα που αναφέ ρ αμε. α2 β2 γ2 � Π <α + γ - α)2 � αβγ � Π (β + γ - α) ακ + β = ακ + β α = ρ (Αφού β + γ - α > Ο) μ μ μ Τελικά Σ βγ(β + γ) � 6Π (β + γ - α) .

x,y>O �0<=?x+y���xy(x+y)�2(fxY)3•

u",

Σβγ(β+γ) � 2Σ($r)3 � 2 · 3V�(αβγ)6

2,

Άρα Σ (--) = Σ [--] +Σ (u") -. ,

μ-Ι

μ-Ι

μ-Ι

Ά Σ- [-] Σ (-) Σ (u") ,

7.

Έστω

οι ακέραιοι α, μ που είναι πρώτοι μεταξύ τους. Να αποδειχθεί ότι:

Σ [ α · κ + β ] = (α - l)(μ - 1) + β . κ=Ο

μ

2

=

μ Ι

μ-Ι

μ-Ι

μ (α 1)(μ - 1)

2

μ

μ

+β .

Κατqλληλο β ο ή θημα για όσους διδάσκο υν καθώς και για τις ε ξε τάσε ις του ΑΣΕΠ. Θα βρε ίτε : •

Σχέδια διδασκαλίας

για όλ ε ς τι ς τάξεις

μ ε τη ν κατάλληλη Μέθοδο, Μορφή και Πορ ε ία διδασκαλ ίας. •

Τη ν αναλυτική βαθμολόγηση

θεμάτω ν στι ς προαγωγικές

και απολ υτήριες ε ξε τάσεις. •

Τη ν αξιολόγη ση το υ μαθη τή

και τη ς διδασκαλίας.

ΕΚΔΟΣΕΙΣ rΕΝΝΑΔΕΙΟΥ ΣΧΟΛΗΣ ι '

Λ. ΒΙΡΗΝΗΣ 7, 163 45 ΗΛΙΟΥΠΟΛΗ ΊΉΛ.: 010-9937645 - 6 FAX: 010-9937211

n

E-mail: mail@geuad.ios.edu.gr

Εmσχεφθεfτε μας: httyJ/www.geoadios.edu.gr

ΕΥΚΛΕΙΔΗΣ Β ' λζ' τ.2135


Α.

yια την Α ..

τάξη του Λυκείου

Τpιώνuμο

Β "'

Μ ετα βολές τ ων συντελεστ ών τ ριωνύ μ ου κα ι συνέπε ι ες στ ις ρ ίζες τ ου η + βχ

βαθμού

Κώστα Μαλλιάκα

Είναι β+ν'Δ -β + J'Δ - β + J'Δ � χ ι' = = Χι Ρ' · 2γ 2α γ 2α · 1 α

·

Έστω εξίσωση αχ1 + γ Ο (1) με α '# Ο, γ '# Ο με ρίζες χι,χ2. χ �"' ο ' α Χ ι' =χ ι . Ρ ' {:::> Χι' =χι . Χι' . Χ ' {:} α) Πολλαπλασιάζουμε όλους τους συντελεστές 1 = Χ ι . χ '2 . Άρ 2 ' ' ' χ · Χ ι' = 1 .Άρα χ ' ειναι της ( 1 ) με λ 7: Ο και έχουμε την εξίσωση βρισκουμε ο μοιως ' οτι 2 2 Ο 2 λαχ + λβχ + λγ = που προφανώς είναι ισο' ροφη της χ2 . ' ροφη της Χ ι και Χ ι' αντιστ αντιστ δύναμη με την ( 1) και έτσι οι ρίζες δεν θα αλ­ γ) Με αλλαγή στο πρόσημο του β , έχουμε: λάξουν. (Ε ') : αχ 2 - βχ + γ = Ο ξ ίσωση Με εναλλαγή του α με το γ έχουμε την ε β) Δ ' = (-β) 2 - 4αγ = β2 - 4αγ = Δ (Ε) : γχ 2 + βχ + α = Ο , με Δ ' = β 2 - 4αγ = Δ ' β ± ν'Δ , ν'Δ ± β Χι ' = 2 2α χ ι,' 2 = 2γ β ν'Δ - β - ν'Δ χ και χ 1' = + Παρατη ρούμε ότι 2α - 2 2α 2 2 ' - β + ν'Δ . - β - ν'Δ (- β) - (J'Δ ) ' β - ν'Δ - - β - ./Δ - - Χι Χι · Χ2 = Χ = 2 2α 2γ 4αγ 2α 2α β 2 - Δ β 2 - β 2 + 4αγ 4αγ Άρα οι νέες ρίζες είναι αντίθετες των προηγόυ­ = = ---1 . 4αγ 4αγ 4αγ μένων Π.χ. (Ε 1 ) : χ 2 + 4χ - 5 = 0 : Ρίζες 1 κα -5. Ομοίως β ρίσκουμε ότι χ 2 · χ � = 1 , δηλαδή 1 1 , , ι'ζες ειναι (Ε ' ) : χ 2 - 4χ - 5 = 0 : Ρίζες -1 και 5. χ ι, = και χ 2, = - . Άρα οι νεες ρ Χι Χ δ) Με εναλλαγή του α με το γ και αλλαγή πρ οσή­ 2 αvτίστροφες των προηγουμένων ριζών. μου του β, έχουμε: 2 Ο 1 Π. χ. (Ε ι ) : χ 5χ + 6 = : Ρίζες 2 και 3 (Ε ' ) : γχ 2 - βχ + α = Ο με Χι' = και Χι 1 1 ' - και - . (Ε 2 ) : 6χ 2 - 5χ + 1 = 0 : Ριζες 1 . 2 3 χ 2' = -χ2 Άλλη ε ξήγηση : Έστω (Ε) : αχ 2 + βχ + γ == Ο Π.χ. (Ε) : χ 2 + 6χ + 8 = Ο : Ρίζες -2 και -4 γ ' ' ' pμε ριζες χ ι , χ2 και γινομενο ριζων -χ ι · χ 2 . α (Ε ' ) : 8χ 2 - 6χ + 1 = Ο : Ρίζες _!_ και _!_ . 2 4 Έστω επίσης (Ε ') : γχ 2 + βχ + α = Ο με ρίζες β με ένα αριθμό ε) Όταν πολλαπλασιάζουμε το α , , χ ι' , χ 2' και γινομενο ριζων Ρ ' = Χι' · χ 2' = - . Ο λ 7: και το γ με το τετράγωνο του αριθμού γ

=

-

_

_

-

ΕΥΚΛΕΙΔΗΣ Β' λ.ζ' τ.2136


Μαθηματικά yια την Α ' Λυκείου

γιατί 5 + 2 + (-7) = 0 . αυτού, τότε οι ρίζες πολλαπλασιάζονται με τον β) Έστω η εξίσωση (Ε) : αχ 2 + βχ + γ = Ο, α 7: Ο αριθμό λ: (Ε ') : αχ 2 + λβχ + λ2 γ = Ο με α + γ = β τότε είναι: Δ = (α - γ) 2 με ρίζες Δ ' = λ2 β 2 - 4αλ2 γ = λ2 · (β 2 - 4αγ) = λ2 · Δ -1 και _ r _ α β± β± χ { 2 = -λ λΓΔ = λ · - ΓΔ = λ · χ \ ' 2 · 2α 2α Π αρατή ρηση : Π.χ. (Ε) : χ 2 + 5χ - 6 = Ο : με Ρίζες 1 και -6. Οι ρίζες τώρα είναι αντίθετες από την προη­ Για λ = 3 έχουμε: (Ε ') : χ 2 + 1 5χ - 54 = 0 : με γούμενη περίπτωση γιατί το β είναι αντίθετο από ρίζες 3 και -1 8. το προηγόύμενο σύμφωνα με το (Α) μέρος, περί­ στ)Ανάλογο για την (Ε ') : λ2 αχ 2 + λβχ + γ = Ο έ- πτωση (γ). χω τελικά χ {, 2 = ! · χ 1 ' 2 δηλ. οι ρίζες διαιρού- γ) Έστω η εξίσωση (Ε) : αχ 2 + βχ = Ο, α Ο λ (γ = Ο) (ελλιπές τριώνυμο). Η (Ε) γίνεται: νται με λ Π.χ. (Ε) : χ 2 - 6χ + 5 = 0 : Ρίζες 1 και 5. χ · (αχ + β ) = Ο {:} χ = Ο ή χ = - � . α Για λ = 2 (Ε ' ) : 4χ 2 - 12χ + 5 = 0 : 2 δ) Έ στω η εξίσωση (Ε):αχ + γ = Ο, α 7: Ο 1 5 ' Ριζες 2 και 2 . ( β=Ο). Η (Ε) γίνεται: χ 2 = _ l , οπότε: ζ) (Ε '):λαχ 2 +βχ+ l =ο {:} λ2 αχ 2 + λβχ + γ = Ο ο­ α λ i) Αν οι α,γ είναι ετερόσημοι, τότε 1 < Ο οπότε πότε καταλήγουμε στην περίπτωση (στ). α � 2 2 2 ' η ) (Ε ): χ +βχ+λγ=Ο {::} αχ + λβχ + λ γ = Ο , η (Ε) έχει δύο ρίζες χ 1 2 = ± [i λ . ν--; οπότε καταλήγουμε στην περίπτωση (ε). ii) Αν οι α, γ είναι ομόσημοι, τότε _ l < Ο ο­ α Β . Συ ν θή κες μ εταξύ τω ν α ,β,γ. 2 2 πότε η (Ε) είναι αδύνατη γιατί χ ?: Ο . α) Έστω η εξίσωση (Ε) : αχ + βχ + γ = Ο με α 7: Ο όταν α + β + γ = Ο , οπότε α + γ = - β ή β = -α - γ τότε Δ = β 2 - 4αγ = Γ. Β α σικές συνθήκες που ισχύ ουν για το εί­ 2 = (-α - γ) 2 - 4αγ = α2 + 2αγ + γ 2 - 4αγ = δ ος ρ ιζών της εξίσωσης {Ε): αχ +β χ+γ=Ο = α2 - 2αγ + γ 2 = (α - γ) 2 ?: Ο οπότε Η (Ε) έχει: 1) μοναδική ρίζα όταν α = Ο και β 7: Ο , δηλ. �ίναι - β ± �(α - γ) 2 - β ± (α - γ) χ \,2 πρωτο βάθμια. 2α 2α 2) μια διπλή ρίζα όταν α 7: Ο και Δ = Ο. -β + α - γ = α + f + α - f =1 3) πραγματικές ρίζες όταν α 7: Ο και Δ ?: Ο . 2α 2α -β - α + γ β + γ - β + γ = γ 4) δύο άνισες ρίζες όταν α 7: Ο και Δ>Ο. 2α 2α α 5) καμία πραγματική ρίζα όταν α 7: Ο και Δ<Ο. Άρα η (Ε) έχει πραγματικές ρίζες και αν α = γ 6) αντίθετες ρίζες όταν α 7: Ο, Δ ?: Ο και S = Ο. είναι ίσες. 7) αντίστροφες ρίζες όταν α 7: Ο, Δ ?: Ο και Ρ = 1 Η (Ε) έχει μία ρίζα το 1 και η άλλη είναι το 8) ετερόσημες ρίζες όταν α 7: Ο, Δ > Ο και Ρ<Ο. r=P 9) ομόσημες ρίζες όταν α Ο, Δ ?: Ο και Ρ > Ο α 1 0) θετικές ρίζες όταν α 7: Ο, Δ ?: Ο , Ρ>Ο, S>O π.χ. 5χ 2 + 2χ - 7 = 0 : Ρίζες το 1 και το 7 5 1 1) αρνητικές ρίζες όταν α 7: Ο, Δ ?: Ο , Ρ>Ο, S <O. ·

7:

7:

ΕΥΚΛΕΙΔΗΣ Β' λζ' τ.2!37


Μ αθηματικά για την Α ' Λυκείου

Δ.

Εξισ ώ σ ε ις π ου

ανάγ οντα ι σ ε

δ ευ τε ρο­

β ά θ μιες

α)

ΔΙΤΕΤΡΑΓΩΝΕΣ: Γενική μορφή: α · χ2κ + β · χκ + γ = Ο, α :;t: Ο (1)

Άρα l x + 2 1 = 1 ή l x + 2 1 = -7 (απορρίπτεται). Ισοδύναμα έχουμε: χ + 2 = 1 ή χ + 2 = - 1 οπότε

Χ = - 1 ή Χ = -3 .

ΜΕΘΟΔΟΣ: Θέτω χ κ = y οπότε Ε. Λυ μ έν ες Α σ κή σ ε ις 2 2 2 2 χ κ = (χ κ ) = y , άρα αy + βy + γ = 0 (2). 1 . Να λυθεί η εξίσωση ( x 2 - 3 1 xl-6>1 = 4 (1) i) Αν Δ>Ο η (2) έχει δύο ρίζες yι ,y2, οπότε: Λύση χ κ = yι η' χ κ = y2 ..... Η (1) είναι ισοδύναμη με: ii) Αν Δ = Ο η (2) έχει μία ρίζα y0, οπότε χ 2 - 3 l x l - 6 = -4 ή χ 2 - 3 l x l - 6 = 4 χ κ = yo .... Άρα χ 2 - 3 l x l - 10 = 0 ή χ 2 - 3 l x l - 2 = 0 iii) Aν Δ<Ο η (2) είναι αδύνατη οπότε και η (1 ) θα Θέτουμε l x l = y :::::: Ο οπότε είναι αδύνατη. (y2 - 3y - 10 = 0 ή y2 - 3y - 2 = 0) {:} Π α ρ α δ ε ίγ ματα : 5 ' ' y= 3 + Jl7 η' y = 3 - J17 y = η y = η 2 3 1 ) χ6 - 9χ + 8 = 0 2 2 3 5 Θέτω χ = y , οπότε η εξίσωση γίνεται Άρα { l x l = ή l x l = -2 } (απορρίπτεται) ή y 2 - 9y + 8 = Ο με ρίζες y1 = 1 ή y2 = 8 3 + Jl7 η' x = 3 - Jl7 Ο} (απορριπτεται) ' = x < l l l l 3 3 άρα χ = 1 ή χ = 8 2 2 οπότε χ = ifi ή χ = V8 δηλ. χ = 1 ή χ = 2. ' οποτε χ = ± 5 η' χ = ± 3 + Jl7 . . 4 2) χ8 + 6χ - 7 = 0 2 4 Θέτουμε χ = y :::::: Ο , οπότε η εξίσωση γίνεται 2. Να λυθεί η εξίσωση y2 +6y - 7=0 με ρίζες y 1 =1 ή y2 = -7 (απορ.) ιοχ2 + 2y2 + 2χ + 2y - 4xy + ι = ο . οπότε χ = ±� δηλ. χ = 1 ή χ = - 1 . Λύση 3) χ 1 0 + 7χ5 - 1 8 = 0 Θέτουμε χ5 = y , οπότε η εξίσωση γίνεται Η εξίσωση μπορεί να θεωρηθεί δευτεροβάθμια ως y 2 + 7 y - 1 8 = Ο με ρίζες yι = 2 ή Υι = -9 , προς χ οπότε ισοδύναμα γράφεται: 2 2 άρα χ5 = 2 ή χ5 = -9 , άρα και συνεπώς 1 0χ + (2 - 4y) · χ + (2y + 2y + 1) = ο Είναι Δ = (2 - 4y) 2 - 4 · 1 0 · (2y2 + 2y + 1) = χ = + ifi ή χ = - ifi . = 4 - 1 6y + 1 6y2 - 80y2 - 80y - 40 = = -64y2 - 96y - 36 = -4(1 6y2 + 24y + 9) = β) ΕΞΙΣΩΣΕΙΣ ΜΕ ΑΠΟΛΥΤΑ = -4 · (4y + 3) 2 ::ς ο . Γενική μορφή: Για να έχει λύσεις πρέπει Δ :::::: Ο οπότε αναγκαστια · (κχ + λ) 2 + β · lκχ + λl + γ = Ο, α � ο (1)

l

{

---}

ΜΕΘΟΔΟΣ: Θέτουμε l κχ + λl = y, . οπότε κά πρέπει Δ = Ο. Άρα 4y + 3 = 0 {::} y = -�4 (κχ + λ) 2 = l κχ + λl 2 = y2 και η ( 1) γίνεται Τότε είναι 2 - 4y = -4y - 2 = 2(2y - 1) = 2y - 1 αy2 + βy + γ = Ο ... Χ = --20 20 10 2· 10 Π α ρ ά δ ε ιγ μα 1 3 ' τελικα' χ = -' για y = -- εχω οποτε 3(χ + 2) 2 + 4 l x + 2l - 7 = 0 (1) 4 4 Θέτουμε l x + 2 1 = y οπότε η (1 ) γίνεται Άρα λύση (x, y) = - , 3y2 + 4y - 7 = Ο με ρίζες Υι = 1 ή Υ2 = -7 .

--

( � �)

ΕΥΚΛΕΙΔΗΣ Β ' λζ' τ.2138


Μ αθηματικά για την Α · Λυκείου

3.

β) Να δειχθεί ότι η f(x) έχει πάντα μια ρίζα ανεξάρτητη του λ. γ) Υπάρχει πραγματικά αριθμός λ ώστε η (1) να έχει: i) αντίθετες ρίζες, ii) αντίστρο φ ες ρίζες;

Ποιες τιμές μπορεί να πάρει η παράσταση Κ = 2χ 2 - 12χ + 5 ; Ποια η ελάχιστη τιμή που μπορεί να πάρει και για ποιο χ; Λύ ση

Η παράσταση μπορεί να πάρει μορφή εξίσωσης '? Λύ ση βαθμού ως προς χ: 2χ2 -Ι2χ+(5-Κ)=Ο . Προφανώς πρέπει να έχει λύσεις οπότε πρέπει Δ 2:: Ο δηλ. α) Αρκεί λ - 2 = Ο ή Δ = Ο και όχι όλοι οι συντε­ Ι44 - 4 · 2 ( 5 - Κ) 2:: Ο � Ι44 - 40 + 8Κ 2:: Ο � λεστές μηδέν. Πράγματι για λ = 2 η εξίσωση f(x) = Ο γίνεται Κ 2:: - Ι 3 . Άρα οι τιμές της παράστασης Κ βρίσκο­ 4χ+4=0 � χ = - 1 δηλ. μοναδική ρίζα το - Ι . νται στο διάστημα [ - 1 3, +οο) και η ελάχιστη τιμή β) Για λ 7: 2 έχουμε: Δ = 4λ2 - 4(λ - 2)(λ + 2) = της είναι το -Ι3 αν Δ = Ο δηλ. χ=� δηλ. χ = 3. 4 = 4λ2 - 4λ2 + Ι 6 = Ι 6 > ο . ±4 . δυο . 4. Να λυθεί η εξίσωση ' ανισες Άρα εχει ρι' ζες: χ 1 , = --2λ ---2 2 (λ - 2 ) [d(x 1 - 5)] 2 + 6 · d(x1 ,-5) - 7 = 0 . . χ = -2λ + 4 = Ι, οποτε 1 2λ - 4 Λύ ση Είναι d(x 1 , -5) = l x + 5 1 οπότε θέτω l x + 5 1 = d χ = -2λ - 4 = -λ - 2 2 2λ - 4 λ-2 Άρα d2 + 6d - 7 = Ο με ρίζες d = Ι ή d = -7 δηλ. έχει μία ριζα το -1 , ανεξάρτητη του λ. δηλ. l x + 5 1 = Ι (απορρίπτεται). γ ι ) Αν έχει ρίζες αντίθετες τότε Δ>Ο και S = Ο και Επομένως χ + 5 = Ι ή χ + 5 = - Ι οπότε χ = -4 . ' λ = ο που λ 7: 2 οποτε αφου. s = -2λ πρεπει ή Χ = -6 . λ-2 είναι δεκτή. 5. Να λυθούν οι εξισώσεις: ο γ2) Αν έχει ρίζες aντίστροφες τότε λ 7: 2, Δ 2:: Ο και (1) (χ 2 - 4) · (χ 2 + 3χ) · (-χ2 - 5χ + 6 ) = λ+2 =Ι� Ρ . λοιπον , -(2) l x 2 - 91 + x 2 - 3x + x 2 - 7 χ + 12 1 = Ο = Ι πρεπει λ-2 λ + 2 = λ - 2 � Ο · λ = -4 (αδύνατο). Λύ ση Άρα δεν μπορεί να έχει aντίστροφες ρίζες. Η (Ι) ισοδυναμεί με: χ2 - 4 = 0 ή χ2 + 3χ = 0 Ζ. Π ροτ ε ιν ό μενες Ασκή σε ις. ή -χ2 - 5χ + 6 = 0 ) � 1 . Να λυθούν οι εξισώσεις : ((χ = 2 ή χ = -2) ή (χ = Ο ή χ = -3) ή α) χ 6 + 5χ 3 - 24 = 0 ( χ = Ι ή χ = - 6)) β) χ 1 2 - 4 χ 6 - 2 Ι = Ο Σύνολο λύσεων: {2,-2,0,-3, Ι ,-6} γ) (3χ - 5)2 + l3x - 5 1 - 20 = ο Η (2) ισοδυναμεί με: χ2 - 9 = 0 και χ2 - 3χ = 0 και δ) < x 2 + 8 1 x l- 1 3)l = 7 χ2 - 7 χ + Ι2 = 0 ε) (3χ2 - 6) · (-2χ2 + 8χ ) · ( χ2 + 2 χ - 63) = 0 (( χ = 3 ή χ = -3) και (χ = Ο ή χ = 3) και (χ = 3 ή χ = 4)) στ) 1 3χ2 +4χ-71 + 1 2χ-2 1 + 1 6χ2 -5χ-Ι ΙΙ =ο Άρα κοινή λύση μόνο το 3 (δηλ. χ = 3). 2. Ποιο είναι το σύνολο τιμών της συνάρτησης 6. Έστω f(x) = (λ - 2)χ 2 + 2λχ + λ + 2 (1). y = -x2 + 7x + 2 . ·

-

--

l

l l

l

α) Να βρεθεί ο πραγματικός λ ώστε η εξί­ σωση f(x) = Ο να έχει μοναδική λύση η οποία να βρεθεί.

3.

Να λυθεί η εξίσωση: Ι 7 χ2 + 6χ + y2 - 8xy + 9 = Ο .

ΕΥΚ.ΛΕΙΔΗΣ Β ' λζ' τ.2139


Μ αθηματικά για την Α ' Λυκείου

-ι .

Έστω f(χ) = (α - 3)χ 2 - 2αχ + α + 3 . α) Να λυθεί η εξίσ�ση f(x) = Ο για α = 1 . β) Να δειχθεί ότι η f(x) έχει πάντα μια ρίζα ανεξάρτητη του λ.

γ) Είναι δυνατόν να έχει η f(x) μοναδική λύ­ ση και αν ναι ποια είναι; δ) Είναι δυνατόν η f(x) να έχει αντίθετες ρί­ ζες και αν ναι ποιες; ε) Να λυθεί η εξίσωση f(α) = Ο ως προς α.

a εχέuει� μεταξu και των

τω ν

αuντελειπών

pιζών εξίαωαης 2°υ βαθμού.

Για την εξίσωση αχ 2 + βχ + γ = Ο με α � Ο και ρίζες ρ ι , ρz θα πρέπει να ξέρουμε ότι: S = ρ ι + ρ z = - -β , Ρ = ρ ι · ρ z = -γ . α α Μία παράσταση των ριζών ρ 1 ,ρ2 μιας δευτεροβάθ­ μιας εξίσωσης λέγεται συμμετρική όταν παραμέ­ νει αμετάβλητη μετά από την αντιμετάθεση των ρ ι ,ρz . Για τις συμμετρικές παραστάσεις των ριζών ρ ι ,ρz θα ξέρουμε ότι: Κάθε ρητή παράσταση συμμετρική ως προς τις ρί­ ζες ρ ι ,ρz μιας δευτεροβάθμιας εξίσωσης μπορεί να εκφραστεί ως συνάρτηση των S και Ρ.

Α. Μπου ρνής

Α ΣΚΗΣΗ 2 .

θεωρούμε την εξίσωση (χ - 1) 2 - (2χ - 3)λ = Ο (1) λ ε ΊR . Αν χ ι ,χ2 οι πραγματικές ρίζες της (1), να δειχτεί ότι η παράσταση

( -�)( Χ2 -�)

Κ = Χ1

είναι ανεξάρτητη του λ. ΛΥΣΗ

(χ - 1) 2 - (2χ - 3) · λ = 0 � χ 2 - 2(1 + λ)χ + 1 + 3λ = ο Πρέπει Δ � 0 � 4(λ2 - λ) � Ο � λ � 1 ή λ � Ο . ΑΣΚΗΣΗ 1 : Να εκφραστούν με τη βοήθεια των S, Ρ οι πα­ Η παράσταση Κ γίνεται 3 (χ + χ 9. ραστάσεις: χ Κ = χ ) + ι 1 2 2 2 4 (ί) S 2 = ρ ; + ρ ; , S3 = ρ : + ρ� Όμως Χ ι + χ 2 = 2(1 + λ) και Χ ι · χ 2 = 1 + 3λ . Άρα 1 1 + -- . (ίί) Α = ρ: ρ 2 + ρ1 ρ ; , Β = -ρι + 3 ρz + 3 κ = 1 + 3λ - 23 · 2(1 + λ) + 49 = ΛΥΣΗ

(i) Sz = (ρ ι + ρ z ) 2 - 2ρ ι ρ z = S2 - 2Ρ S3 = (ρ ι + ρ z ) 3 - 3ρ ι ρ z (ρ ι + ρ z ) = S3 - 3PS (ii) Α = ρ ι ρ z (ρ ι + ρ z ) = p . S 6+S Β = ρ z + 3 + ρι + 3 = (ρ ι + 3)(ρ z + 3) ρ ι ρ 2 + 3(ρ ι + ρ z ) + 9 6+S P + 3S + 9

= 1 + 3λ - 3 - 3λ + 2. = .!. 4 4

ΑΣΚΗ Σ Η

3.

2 Στην εξίσωση αχ + βχ + γ Ο είναι α>β>γ>Ο. Αν ρ 1 ,ρ2 είναι οι ρίζες της να δειχτεί ότι: l ρ 1 1 < 1 και l ρ 2 1 < 1 . =

Λ ΥΣΗ

Αφού ρ ι ,ρz είναι οι ρίζες της αχ 2 + βχ + γ = Ο θα ισχu' ει ρ 1 + ρ 2 = - -β < ο και ρ 1 · ρ 2 = -γ > ο . α α

ΕΥΚΛΕΙΔΗΣ Β' λζ' τ.2/40


Μ αθηματικά για την Α ' Λυκείου

Άρα ρ ι < Ο και ρ2 < Ο οπότε l ρ 1 + ρ 2 Ι = l ρ ι l + l ρ 2 l ·

Α ΣΚΗΣΗ 6.

Είναι l ρ 1 + ρ 2 1 = - = < ι και συνεπώς

(i) Έστω ρ ι ,ρz οι ρίζες της εξίσωσης αχ 2 + βχ + γ = Ο όπου α, γ :;t: Ο . Να κατασκευ α-

� �� �

l ρ ι l + Ι ρ 2 1 < ι άρα l ρι l < ι και Ι ρ 2 1 < ι . ΑΣΚΗΣΗ 4.

Έστω ρ ι ,ρz οι ρίζες της εξίσωσης αχ 2 + βχ+γ=Ο α, β,γ ε :ΙR με α :;t: Ο και ισχύει l α i > Ι α + β + γ l . Να δειχτεί ότι μια τουλάχιστον από τις ρίζες α­ νοίκει στο ανοικτό διάστημα (0,2) . Λ Υ ΣΗ

Ι � �I ·

Είναι Ι α Ι > α + β + γΙ {::} ι > ι + + Ι Όμως Q_ = -(ρ ι + ρ 2 ) και Ί_ = ρ ι ρ 2 α α ι > Ι ι - ρ ι - ρ 2 + ρ ι ρ 2 Ι {::} άρα ι > I (1 - ρ ! ) - ρ 2 (1 - ρ ! )I {::} . ι > Ι ι - ρι i · Ι ι - ρ2 l Οπότε Ι ι - ρ 1 1 < ι ή l 1 - ρ 2 1 < 1 . Από την Ι ι - ρ 1 1 < ι {::} - ι < 1 - ρ 1 < 1 {::} -2 < -ρ 1 < Ο {::} Ο < ρ 1 < 2 ή Ι ι - ρ 2 1 < 1 {::} - 1 < ι - ρ 2 < ι {::} {::} -2 < -ρ < Ο {::} Ο < ρ < 2 ή Ι ι - ρ 1 < Ο <=> 2 2 2 ι <=> - ι < ι - ρ2 < <=> -2 < -ρ < ο <=> ο < ρ < 2 2 2

ι ι ' στει, εξισωση με ρι'ζες χ 1 = - και χ 2 = - . ρz ρι (ii) Αν ρ ι ,ρz οι ρίζες της χ 2 + ν · l λ l χ + λ = Ο όπου λ>

ο,

ν

Ε

Ν* .

Να αποδειχθεί ότι:

[ J

ΛΥΣ Η

(i) Η εξίσωση που ζητώ θα είναι 1 =ο. ι χ + _!_ . _ χ 2 - _!_ + _ ρι Ρ 2 ρι Ρ 2 _!_ + _ ι = ρι + Ρ2 ρι ρ2 Ρι ρ 2

Είναι

ι:. ι + ι:2 ι = ν .

β

γ α

_ Q. γ

και

. ι = -ι = -α . Οποτε -ι · ρι ρ2 l γ α χ 2 - - χ + = 0 {:} γχ 2 + βχ + α = Ο .

[ �]

(ii) Αφού ρ ι ,ρ2 οι ρίζες της χ 2 + ν l λl χ + λ = Ο , η εξίσωση λχ 2 + ν l λl χ + ι = Ο ( ι ) . ρι' ζες τις ι ,ι (λογω ' του ι. ) . θα εχει ρι ρ2 ΑΣ ΚΗΣ Η 5 . Έστω χ 1 ,χ2 οι ρίζες της ( ι ) θα είναι Έστω ρι,ρ z οι ρίζες της εξίσωσης κχ 2 +λχ+l =Ο ι με κ :;t: Ο και ισχύει λ 2 -2κ 2 < λ · l κ l . Να δειχτεί l x ι l + l x 2 1 = ι + ρ · 2 ότι: l ρ ι + ρ z l < 2 . 2 ( lx ι l + lx 2 1 ) = ( x f + χ � ) + 2 l x ι l lx 2 1 = Λ ΥΣΗ (χ 2 + Χ 2 ) 2 - 2χ ι χ 2 + 2 l χ ι χ 2 1 = Είναι λ2 - 2κ 2 < λ l κ l {::} l λl 2 - 2 l κ l 2 < l λl · l κl αφού ν λl ' - 2 + 2 = ν' αριθμός. που είναι θετικός λ ::; l λl διαιρώ με �, 1 1 Οπότε Άρα lx ι l + l x 2 1 = ν => + = ν . 2 λ 11 l ιl 1 21 � &2 _ 2 < {::} - - 2 < ο {::} κ κ κ l l l κl

l: l Ι 1

ΙΙ ΙΙ

(I �I- J-(1�1 ) Όμως � � � + ι > Ο άρα � �� - 2 < Ο {::} � �� < 2 {::} lρ1 + ρ2 1 < 2 . 2

+1 <ο

[- ι ]

i �

ΑΣΚΗΣΗ 7.

; :

Έστω ρ ι ,ρz οι ρίζες της εξίσωσης χ 2 + χ + λ = Ο . Να βρεθεί ο πραγματικός αριθμός λ, αν είναι γνωστό ότι ρ i + ρ 1 ρ 2 (2ρ 1 + ρ 2 ) + 2ρ 2 = Ο ( ι ) . Λ ΥΣΗ

Αφού ρ ι ,ρ2 οι ρίζες της εξίσωσης χ 2 + χ + λ = Ο θα έχω: ΕΥΚΛΕΙΔΗΣ Β ' λζ' τ.2/41


Μ αθηματικά γία την Α ' Λυκείου

ρ ι + ρ z = - 1 και ρι · ρ z = λ · Η σχέση (1) γράφεται: ρ i +λ(ρ 1 -1 )+2ρ 2 =Ο (2). Είναι ρ � + ρ 1 + λ = Ο =? ρi + ρ � + ρ 1 λ = 0 {:} {::} ρi = -ρ � - ρι λ {::} ρi = -(-ρ ι - λ) - λρ ι {::} {::} ρi = ρ 1 + λ - λρ 1 . Οπότε η σχέση (2) γράφεται ρ 1 + λ - λρ 1 + λ(ρ 1 - 1) + 2ρ 2 = Ο {::} ρ +ρ =-1 {::} ρ 1 + 2 ρ2 = Ο . Από το σύ στημα ι z ρ ι + 2ρ 2 = 0 προκύπτει ρι = -2 και ρ2 = 1 . Άρα λ = ρ 1 · ρ 2 = (-2) · 1 = -2 .

{

Αφού ρ ι ,ρz ρίζες. Άρα ρ � = 1 - ρ 1 Όμως ρ ι + ρ 2 = - 1 =? ρ ι = - 1 - ρ 2 · Δηλαδή ρ � = 1 - (- 1 - ρ 2 ) = 2 + ρ 2 . Ό μοια ρ � = 2 + ρ 1 •

Α ΣΚΗΣΗ 1 0.

Αν ρ 1 ,ρ2 οι ρίζες της εξίσωσης χ 2 - αχ + αβ = Ο , ρz ,ρ3 οι ρίζες της εξίσωσης χ 2 - β χ + βγ = Ο και ρ3,ρι οι ρίζες της χ 2 - γχ + γα = Ο . Να δειχτεί α+β+γ = Ο, αν είναι γνωστό ότι: αβγ :;t: Ο .

Α ΣΚΗΣΗ 8.

ΛΥΣΗ

Αφού ρ 1 ,ρ2 οι ρίζες της χ 2 - αχ + αβ = Ο , θα είναι ρ ι + ρz = α (1) και ρ 1 · ρ 2 = αβ (2). Αφού ρ2 ,ρ3 οι ρίζες της χ 2 - βχ + βγ Ο , θα είναι ρz + ρ3 = β (3) και ρ 2 · ρ 3 = βγ (4). ΛΥΣΗ Αφού ρ3,ρι οι ρίζες της χ 2 - γχ + γα = Ο , θα είναι Αφού Ι αΙ - Ι β l > 1 {::} Ι αl > 1 + l β l {::} α2 > Ι + β 2 +2 Ι β ι . ρ3 + ρ ι = γ (5) και ρ3 · ρ 1 = αγ (6). Πολλαπλασιάζω κατά μέλη τις (2), (4), (5) οπότε Όμως 1 + β 2 � 2 1 β 1 . ρ ι2 ρ 22 ρ 32 = α 2 β 2 γ 2 αρα ρ ι ρ 2 ρ 3 = ± αβγ . 1� 2:13 2 2 2 Άρα α > 41 βl α � 4β =? α -4β > 0 =? Δ > 0 . Αν ρ 1 ρ 2 ρ 3 = αβγ τότε λόγω (2), (4), (6) είναι Δηλαδή η εξίσωση χ 2 + αχ + β = Ο έχει ρίζες ρ ι = α, ρ 2 = β ' και ρ = γ · 3 πραγματικές και άνισες. Προσθέτω (1), (3), οπότε (5) Είναι τώρα ρ 1 + ρ 2 = -α και ρ 1 ρ 2 = β . Άρα 2(ρ ι + ρ 2 + ρ3 ) = α + β + γ οπότε l ρ ι + ρ 2 Ι - Ιρ ι l lρ 2 Ι > 1 και αφού 2( α + β + γ) = α + β + γ . lρ ι + ρ 2 Ι ::; Ι ρ ι l + lρ 2 1 θα έχω Άρα α+β+γ = Ο. l l Αν ρ ι ,ρ2 ,ρ3 = -αβγ τότε λόγω των (1), (3), (5) είναι lρ ι + ρ 2 1 - 1 ρ ι l lρ 2 1 - 1 > Ο {:} ρ ι = -α, ρz = -β και ρz = -γ οΠότε ( Ι ρ ι l - 1) - lρ 2 1 ( Ιρ ι l - 1) > Ο -2(α + β + γ) = α + β + γ δηλαδή α+ β+γ = Ο. {:} (l ρ ι l - 1) (1 - lρ 2 1 ) > 0 (1). Α ΣΚΗΣΗ 1 1 . Από την (1) παίρνουμε: Έστω ρ 1 ,ρ 2 οι ρίζες της εξίσωσης χ 2 + αχ + β = Ο l ρ 1 l - 1 > Ο και 1 - lρ2 1 > Ο ή με ρ ; :;t: ρ ; β :;t: Ο . Αν οι ρ1,ρz είναι ρίζες και της Cl ρ 1 l - 1 < Ο και lρ 2 l - 1 < Ο) . Οπότε: εξίσωσης χ 2 ν + αχ ν χ ν + 1 = Ο να δειχτεί ότι οι α1 > l ρ 2 1 > Ο ή Ο < lρ ι l < 1 και κ 2 = � είναι ρίζες της εξίΑπ ' τις σχέσεις αυτές συμπεραίνουμε ότι και οι ριθμοί κ 1 = .&. ρ ρι δύο ρίζες δεν μπορεί να είναι ακέραιοι αριθμοί. σωσης 1 + χ ν + (1 + χ) ν = Ο όπου ν άρτιος. Έστω α, β ε !R με Ι α 1 - Ι β l > 1 όπου β :;t: Ο . Να δειχτεί ότι η εξίσωση χ 2 + αχ + β = Ο έχει ρίζες πραγματικές και άνισες και ότι δεν μπορεί να είναι ακέραιοι αριθμοί και οι δύο ρίζες.

=

'

z

ΑΣΚΗΣΗ 9.

Έστω ρ ι ,ρz οι ρίζες της εξίσωση ς χ 2 + χ - 1 = Ο . Να δειχτεί ότι ρ: ρ 2 + 2 και ρ � = ρ 1 + 2 . =

ΛΥΣΗ

' ρ 12 + ρ1 - 1 = Ο και ρ 2 + ρ. - 1 = Ο . Ειναι 2 2

ΛΥΣΗ

Αφού ρ ι ,ρz οι ρίζες της χ 2 + αχ + β = Ο , θα είναι ρ ι + ρ 2 -α και ρ ι ρ 2 = β · Ακόμη ρ ι ,ρz ρίζες της χ 2 ν + αχ ν χ ν + 1 = Ο . Άρα ρ � ν + α ν ρ� + 1 = 0 (1) και ρ � ν + αν ρ� + 1 = 0 (2).

ΕΥΚΛΕΙΔΗΣ Β ' λζ' τ.2/42

=


Μ αθηματικά για την Α ' Λυκείου

Αφαιρώ (1) και (2) οπότε ρ � ν - ρ Ξν + α ν (ρ� - ρ� ) = Ο � (ρ� - ρ� )(ρ� + ρ� + αν ) = Ο . Άρα ρ� + ρ� + αν = Ο (3) αφού ρ� 7: ρ� . Διαιρώ την (3) με ρ� , ρ� διαδοχικά οπότε ν ν 1 + 12_ + α = Ο � 1 + ρ2 + (ρι +�2 ) ν ::::} ρι ρι ρι �

[]

:

'*

<<άρτιος>>)

[] ι+ [ :: J • + + ::J"=o

Π ρο τε ιν ό μ ες Α σκή σε ις

Ι.

2.

δηλαδή ο

αριθμός Κι = 12_ είναι ρίζα της 1+χν+(l+χ)ν=Ο . ρι Όμοια ο κ2 είναι ρίζα της ίδιας εξίσωσης.

3.

Αν α, β , γ ε JR με α · γ > Ο και j α + γ j < β να δειχτεί ότι η εξίσωση αχ 2 + βχ + γ = Ο έχει ρίζες άνισες και ότι αυτές οι ρίζες δεν μπο­ ρούν να είναι και οι δύο ακέραιοι αριθμοί. Έστω ρ ι ,ρ2 οι ρίζες της αχ 2 + βχ + γ = Ο με α · γ < Ο . Να σχηματισθεί εξίσωση 2ου β αθ­ μού με ρίζες j ρ ι j , j ρ 2 l · Να υπολογιστεί η παράσταση (3 - J5)4 + (3 + J5 ) 4 .

Ποpόλληλc( culcfcς �θpοιιιμα γωνιών τpιγώνοu e

Καραγιάννης Β. Ιωάννη ς Οι παράλληλες ευθείες και το άθροισμα των γωνιών ενός τρ ιγώνου αποτελο ύν σημαντικά εργαλεία της Ευκλείδειας Γεωμετρ ίας. Θα προσπαθήσουμε εδώ να δώσουμε κάποια υποδείγματα που «δε ίχ vουν» μια μe­ θοδολο γική προσέγγι ση. Στο τέλος υπάρχουν προτρνόμενες ασκήσεις για εξάσκηση και εμπέδωση. �

Δι = Ο 2 (ως εντός εναλλάξ γωνίες). Άρα από την Στο παρακάτω σχήμα η ΟΕ είναι διχοτόμος (1) έχουμε Γι = Δι , δηλαδή το τρίγωνο ΓΟΔ είναι της γωνίας ΑΟΒ και το σημείο Γ είναι τυ- ισοσκελές. χαίο σημείο στην προέκταση της πλευράς Σχό λιο : Μία ευθεία κάθετη ή παράλληλη στη δι­ ΑΟ. Αν ΓΔΙΙΟΕ να αποδειχθεί ότι το τρί­ χοτόμο μιας γωνίας δημιουργεί ισοσκελές τρίγωνο.

Υποδείγματα 1"

.

,..._

λ

-

2°.

γωνο ΓΟΔ είναι ισοσκελές.

Β

Γ

Στο παρακάτω σχήμα οι ευθείες ΑΒ και ΔΕ είναι παράλληλες. Αν ABr 120° και fΔΕ 110° ' να υπολογίσετε τη γωνία χ. =

Ε

=

Δ

Ε

Ο

Λύση

Γ

Υ

Αρκεί να αποδείξουμε ότι Γι = Δ ι . Λύση Έχουμε ότι: Οι = Ο 2 (1) (αφού η ΟΕ διχοτόμος Φέρνουμε από το Γ την ευθεία yy' 11 ΔΕ . Επειδή της γωνίας ΑΟΒ ). Επειδή όμως ΓΔΙΙΟΕ έχουμε ότι Γι = Οι (ως ε- Γy II ΔΕ και ΔΕ II ΑΒ προκύπτει ότι ΑΒ II Γy . ° ° ντός, εκτός και επί τα αυτά μέρη γωνίες) και Αφού Γy 11 ΔΕ θα είναι: Δry + 1 10 = 180 ή λ

,..._

,..._

-

λ

,..._

ΕΥΚΛΕΙΔΗΣ Β' λζ' τ.2/43


Μ αθηματικά για την Α · Λυκείου

Δr y = 1 80° - 1 1 0° = 70° . τέλος έχουμε ότι ABr = Br y (ως εντός εναλλάξ) άρα: 120° = χ + 70° ή χ = 120° - 70° ή χ = 50° . ' Σχόλι ο : Οι παράλληλες ευθείες σ ένα σχήμα, μας επιτρέπουν να «μεταφέρουμε» τις άγνωστες γωνίες δίπλα σε γνωστές γωνίες, και με τη βοήθεια των ι­ διοτήτων των παραλλήλων, να τις βρούμε. Για το λόγο αυτό πολλές φορές αν στο σχήμα δεν υπάρχουν παράλληλες τις φέρνουμε εμείς. (Βοηθητική παράλ­

Α, θα ταυτίζονται. Άρα τα Ε, Α, Δ είναι συν� θειακά. 4°. Δίνεται τρίγωνο ΑΒΓ και η διχοτόμος Αχ �

της γωνίας Α . Από το μέσο Μ της ΒΓ φέρνουμε κάθετη στην Αχ στο σημείο Ρ που τέμνει την ΑΓ στο Κ και την προέκταση της ΑΒ στο Σ. Να αποδείξετε ότι 2ΑΣ = ΑΒ+ΑΓ. Λύση

Α

ληλη) .

3".

Σε ένα τρίγωνο ΑΒΓ οι ΒΜ και ΓΝ είναι διάμεσοι οι οποίες προεκτείνονται έτσι ώ­ στε, ΜΒ = ΜΔ και ΓΝ = ΝΕ. Να αποδείξετε ότι: i. ΑΔ//ΒΓ ii. ΕΑ//ΒΓ iii. Τα σημεία Ε, Α, Δ είναι συνευθειακά.

Από το Γ φέρνουμε παράλληλη στην ΑΒ που τέ­ μνει την προέκταση της ΣΚ στο Ν. Το τρίγωνο ΑΣΚ είναι ισοσκελές (αφού η ΑΡ εί­ Λύση ναι ύψος και διχοτόμος). i. Για να αποδείξουμε ότι ΑΔ//Β Γ αρκεί να απο­ Άρα ΑΣ = ΑΚ (1) και ΑΣΚ = ΑΚΣ = θ . δείξουμε ότι οι εντός-εναλλάξ γωνίες που σχη­ Συγκρίνουμε τα τρίγωνα ΒΜΣ και ΜΓΝ. Έχουμε: ματίζονται είναι ίσες. ΒΜ = ΜΓ (διότι Μ μέσο της ΒΓ) Συγκρίνουμε τα τρίγωνα ΑΔΜ και ΒΓΜ. fJ3M: = Mrn (εντός εναλλάξ των ΑΣ//ΓΝ) ΜΔ = ΒΜ (υπόθεση) ΒΜΣ = ΝΜΓ (κατακορυφήν) ΑΜ = ΜΓ (Μ μέσο της ΒΓ) Άρα τα τρίγωνα ΒΜΣ και ΜΓΝ είναι ίσα. Μ 1 Μ 2 (κατακορυφήν γωνίες). Άρα ΒΣ = ΓΝ (2), ΚΝΓ = ΑΣΚ = θ . Ακόμα ΑΚΣ=ΝΚΓ=θ (κατακορυφήν). Άρα το τρίγωνο ΓΚΝ είναι ισοσκελές, και έτσι ΓΝ = ΓΚ (3). -

-

-

-

=

-

-

-

-

(I)

Άρα ΑΒ + ΑΓ = ΑΣ - ΒΣ + ΑΚ + ΚΓ = (2) ( 2) = ΑΣ - ΒΣ + ΑΣ + ΓΝ = 2ΑΣ .

Άρα τα τρίγωνα ΑΔΜ και ΒΓΜ είναι ίσα οπότε και ΑΔΒ = ΔΒΓ . Δηλαδή οι ΑΔ και ΒΓ που τέμνονται από τη ΒΔ σχηματίζουν τις εντός ε­ ναλλάξ γωνίες τους ίσες. ii. Τα τρίγωνα ΕΑΝ και ΒΝΓ είναι ίσα (αφού ΑΝ = ΝΒ, ΓΝ = ΕΝ και Ν1 = Ν2 ). Άρα ΑΕΝ = ΕΓΒ . Δηλαδή οι ΕΑ και ΒΓ που τέμνονται από την ΕΓ σχηματίζουν τις εντός­ εναλλάξ γωνίες ίσες. Άρα ΑΕ//ΒΓ. iii. Από τα ερωτήματα i) και. ii) προκύπτει ότι ΑΕ//ΑΔ και αφού έχουν ένα κοινό σημείο, το -

-

-

5". Ένας μηχανικός θέλει να μετρήσει την α­

πόσταση του σημείου Β το οποίο θέλει να βρίσκεται στο δρ όμο χχ ', από το σημείο Α που βρίσκεται στην απέναντι όχθη ενός π� ταμού. Μέτρησε τη γωνία ΑΒχ = 60° , την απόσταση ΒΓ = 60m και τη γωνία ΑΓχ για την οποία βρίσκει ότι ισούται με το μισό της γωνίας ΑΒχ ' . Το συμπέρασμα του είναι ότι τα σημεία Α και Β απέχουν 50m. Έχει δίκαιο;

ΕΥΚΛΕΙΔΗΣ Β' λζ' τ.2/44

--

--


Μ αθηματικά για την Α' Λυκείου

7.

Λύση Α

'

'

'

'

'

=

'

'

'

'

'

'

'

'

ΑΒχ = 60°

και Arx = 30° έτσι θα είναι .ABf = 120° και BAr = 30° . Δηλαδή το τρίγωνο ΑΒΓ που σχηματίζεται είναι ισοσκελές με Α = Γ και άρα ΑΒ = ΒΓ = 60m. και όχι 50m όπως ισχt}­ ρίζεται ο μηχανικός.

Όχι, διότι

Α

6 ".

Έστω ΑΒΓ ένα ισόπλευ ρο τρίγωνο και σtt μείο Δ στην προέκταση της ΒΓ (προς το ΒΓ ' ' ' ' ΒΔ Τ . Φ ερμερος του Β) , τετοιο ωστε νουμε το ύψος ΓΕ και έστω Ζ το σημείο τομής της ΔΕ με την ΑΓ. Από το Ζ φέρνο&­ με ευθεία παράλληλη στη ΒΓ η οποία τέ­ μνει την ΑΒ στο Θ. Να αποδείξετε ότι: i. Το τρίγωνο ΔΒΕ είναι ισοσκελές ii. ΔΖ .l ΑΓ iii. AE = 2ΖΘ iv. 4ΓΖ = 3ΑΒ. •

Λύ ση

Δίνεται τρίγωνο ΑΒΓ τέτοιο, ώστε ΑΒ<ΒΓ. Πάνω στη ΒΓ παίρνουμε σημείο Κ ώστε να ισχύει ΒΚ = ΑΒ. Αν ΑΛ είναι η διχοτόμος της Α , να αποδείξετε ότι Γ 2ΛΑΚ �

----

=

Α

Α

λύση

Β

i. Αφού το τρίγωνο ΑΒΓ είναι ισόπλευρο και

ΓΕ .l ΑΒ , το Ε είναι μέσο του ΑΒ . Άρα ΕΒ = ΔΒ = � . Δηλαδή το τρίγωνο ΔΒΕ 2 Γ Λ Κ είναι ισοσκελές με κορυφή το Β. Έχουμε: Γ = ΒΚΑ - Α3 (διότι ΒΚΑ εξωτερική ii. Στο τρίγωνο ΔΒΕ έχουμε: ΕΒΓ εξωτερική γωτου τριγώνου ΚΓΑ) οπότε Γ = ΒΑΚ - Α3 (διότι νία του τριγώνου ΔΒΕ. ΑΒ = ΒΚ, το τρίγωνο ΚΒΑ είναι ισοσκελές, Στο τρίγωνο ΔΖΓ έχουμε ( ΑΖΔ εξωτερική γωΒΚΑ = ΒΑΚ ). νία του τριγώνου ΔΖΓ). Έτσι: Γ = Αι + Az - Α3 (διότι ΒΑΚ = Αι + Az ) m=x+60° =30° +60° =90° , άρα ΔΖ .l ΑΓ . άρα iii. Το τρίγωνο ΘΕΖ είναι ισοσκελές Γ = (Αι - A 3 ) + Az . Αλλά Αι = Az + Α 3 αφού η ( ΘΕΖ = ΘzΕ = 30° ) γιατί ΘΕΖ = ΔΕΗ = 30° ΑΛ διχοτόμος. και eZE = ΑΖΕ - AZe = 90° - 60° = 30° άρα Άρα: Γ = Az + Az = 2Α 2 = 2 · ΛΑΚ ΘΖ = ΘΕ = ΑΘ (γιατί το τρίγωνο ΑΘΖ ισό­ πλευρο) και έτσι ΑΕ = 2ΘΖ. Σχόλιο: Σε κάθε τρίγωνο ΑΒΓ έχουμε: ΑΒ Α Γ Β A + B + f = 1 80° (1) ή + + = 90° (2) ή ΑΒ (1). ΑΕ 2 2 2 iv. 'Εχουμε ΘΕ = = 2 = 4 2 2 Α + Β = 90ο _ Γ (3) ή Α = 90ο _ Β + Γ (4). ΑΒ + ΑΒ = -3ΑΒ *? 2 2 2 2 2 Επίσης ΘΒ = ΒΕ + ΘΕ (Ι) =4 4 2 Ειδικά τις σχέσεις (2) και (4) τις χρησιμοποιούμε 3ΑΒ = 4ΘΒ (2) όταν στο σχήμα υπάρχουν διχοτόμοι γωνιών τρι­ Επίσης ΑΒ = ΑΓ και ΑΘ = ΑΖ, οπότε γώνου. Σε ορθογώνια τρίγωνα ΑΒΓ με Α = 90° ΑΒ - ΑΘ = ΑΓ - ΑΖ άρα ΘΒ = ΓΖ (3). Από είναι Β + f = 90° . (2),(3) προκύπτει 3ΑΒ = 4ΓΖ. -

Α

-

Α

-

--

-

......

......

-

Λ

-

λ

.-...

-

..-,

.-...

-..

-

.......

.-_

-

-

-

.-.

-

-..

-..

-.

......

ΕΥΚΛΕΙΔΗΣ Β ' λζ' τ.2/45


Μ αθηματικά για την Α' Λυκείου Δ

8°. Δίνεται ορθογώνιο τρί­

γωνο ΑΒΓ με Α = 90° . Στο ημιεπίπεδο τη ς ΒΓ που δεν περιέχει το Α φέρνουμε κάθετη ημιευ­ θεία προς την ΒΓ στο σημείο Β και σ ' αυτήν παίρνουμε τμήμα ΒΔ = ΒΑ. Να αποδειχθεί ότι η διχοτόμος ΓΕ της γωνίας Γ είναι κάθετη στην ΑΔ.

λ

λ

1 0°. Στο παρακάτω σχήμα είναι Β = 2Γ και

ΑΔ ..l ΒΓ και ΒΜ = ΒΔ.

Α

Γ

λ

Λύ ση

Έχουμε Δ = ΒΑΔ = φ . Στο τρίγωνο ΒΑΔ είναι 2φ+ΑΒr=90° <==> 2φ+(90° -2χ) = 90° <==> φ=χ (1). (Ι) Άρα: ΔΑΓ + ΕΓΑ = (90° - φ) + χ = 90° άρα λ

Να αποδείξετε ότι: ί. ΒΜΔ = Γ , ... ιιι. θ + y = 9 0° , --

λ

ίί. ΔΡ = ΡΓ, iv. ΑΡ = ΡΓ. Λ ύση

Αφού ΒΜ ΒΔ το τρίγωνο ΜΒΔ είναι ισοσκελές και έτσι Β = 2ΒΜΔ (αφού ΒΜΔ = ΒΔΜ ), και αφού Β = 2Γ από τα ΑΔ ..l ΓΕ . δεδομένα θα έχουμε ΒΜΔ = Γ . 9 °. Στο διπλανό σχήμα η ΑΒ ίί. ΒΜΔ = ΒΔΜ = y (ως κατακορυφήν γωνίες) και είναι εφαπτομένη του αφού από το i) ΒΜΔ = Γ θα είναι y = Γ άρα κύκλου με κέντρο Ο και το τρίγωνο ΡΔΓ είναι ισοσκελές, άρα ΔΡ = ΡΓ. οι ΑΒ και ΒΓ σχηματί­ ίίί. θ + f = 90° [ αφού το τρίγωνο ΑΔΓ είναι ζουν ίσες γωνίες με την ορθογώνιο (Mr = 90° ) και αφού y = Γ θα ΑΟΔ. Ας είναι ΒΑΟ=χ . είναι y + θ = 90° . α) Να εκφράσετε ως iv. Έχουμε χ + y = 90 ° και y + θ = 90° άρα συνάρτηση της χ τη γωνία ΟΒΓ . χ + y = y + θ <==> χ = θ και έτσι το τρίγωνο β) Να αποδείξετε ότι: ΓΟΑ = 3ΑΟΒ ΑΔΡ είναι ισοσκελές, άρα ΑΡ ΡΔ και επειδή Λύση ΡΔ ΡΓ θα είναι ΑΡ ΑΓ. α) Οι ΑΒ, ΒΓ σχηματίζουν ίσες γωνίες με την ΑΟΔ =? ΒΑΟ = ΑΕΒ = χ . Πρ οτ ε ιν όμ ενες Ασκή σεις Στο τρίγωνο ΑΟΒ είναι ΑοΒ = 90° - χ . Έστω ΟΒΓ = φ , τότε και ΟΓΒ = φ . Στο τρίγωνο 1 . Σε ένα τρίγωνο ΑΒΓ είναι Γ = 3Β . Η μεσοκάθετος της ΒΓ τέμνει την ΑΒ στο Δ. ΟΕΒ έχουμε ότι η γωνία ΑΟΒ είναι εξωτερική Να αποδείξετε ότι: γωνία του, άρα i) Το τρίγωνο ΔΒΓ είναι ισοσκελές. ΑΟΒ = ΟΕΒ + ΟΒΕ <=:> 90° - χ = χ + φ ii) Το τρίγωνο ΑΓΔ είναι ισοσκελές. <=:> φ = 90° - 2χ <=:> ΟΒΓ = 90° - 2χ 2. Δίνεται ορθογώνιο τρίγωνο ΑΒΓ ( Α = 90 ° ) β) Έχουμε ΑοΒ = 90° - χ (1) (από το τρίγωνο με ΑΒ > ΑΓ και η διχοτόμος ΑΔ. Φέρνουμε ΑΟΒ). Δχ ..l ΒΓ που τέμνει την ΑΒ στο Ε και την Επίσης ΓΟΑ = φ + ΟΕΓ προέκταση της ΓΑ στο Ζ. Να αποδείξετε ότι: (ως εξωτερική στο τρίγωνο ΓΟΕ) = ii) ΒΕ ΓΖ. i) ΔΕ ΔΓ (α) ° ° = φ + 1 80 - χ = 90 - 2χ + 1 80 - Χ = 3. Αν η βάση ΒΓ ενός ισοσκελούς τρίγωνου είναι -

ί.

=

λ

-

-

λ

-

--

-

Λ

Λ

-

-

Λ

Λ

·

-

=

=

-

=

-

Λ

λ

-

-

-

-

-

-

-

=

(I)

= 270° - 3χ = 3(90° - χ) = 3ΑΟΒ -

=

μεγαλύτερη ή ίση ή μικρότερη των ίσων πλευ-

ΕΥΚΛΕΙΔΗΣ Β' λζ' τ.2/46


Μαθηματικά για την Α' Λυκείου

4.

5.

ρών του, να δείξετε ότι η γωνία Α είναι αντί­ στοιχα μεγαλύτερη ή ίση ή μικρότερη των 60° . Δύο κύκλοι εφάπτονται εσωτερικά ή εξωτερι­ κά στο σημείο Α. Μία ευθεία ε διέρχεται από το Α και τέμνει τους κύκλους στα σημεία Β και Γ. Να δείξετε ότι οι εφαπτόμενες των κύ­ κλων στα Β και Γ είναι παράλληλες. Δίνεται τρίγωνο ΑΒΓ με ΑΒ<ΑΓ. και ένα σημείο Μ της πλευράς ΒΓ . Από το Μ φέρ­ νουμε παράλληλη προς την διχοτόμο της γωνίας Α που τέμνει τις ΑΒ και ΑΓ στα σημεία Ε και Ζ αντίστοιχα. Να αποδείξετε ότι: i) Το τρίγωνο ΕΑΖ είναι ισοσκελές. ii) Το άθροισμα ΒΕ+ΓΖ είναι σταθερό. /'.

6.

Στο παρακάτω σχήμα οι ευθείες χχ· και yy· είναι παράλληλες. Έστω ΒΓ η διχοτόμος της γωνίας ΑΒΔ , ΓΔ η διχοτόμος της γωνίας ΒΓ y , ΔΕ η διχοτόμος της γωνίας fΔχ ' . --

χ

Β

\

Υ

\

\

\

\

\

I

I

I

I

I

I

I

Δ

I \

εναλλάξ γωνίες στα σημεία Ε και Η. Όμως η διάταξη των κτιρίων δεν του επέτρεψε να φέ­ ρει την ευθεία ΕΗ. Έτσι λοιπόν κατασκεύασε τη γωνία ΑΕΖ = 86° και τη γωνία ΕΖΗ = 1 65° . Πόσες μοίρες πρέπει να είναι η γωνία ΖΗΔ ώστε να σχεδιάσει την ΓΔ παράλληλη στην ΑΒ. -

1 0.

Στο διπλανό σχήμα το τρίγωνο ΑΒΓ είναι ορ­ θογώνιο με Α = 90° . Αν ΑΔ το ύψος του και ΒΜ ΒΔ, να αποδείξετε ότι: =

Μ '

, ,

Β•

ι

' '

\

\Δ ' ' '

Χ\ Ρ '

Α

i) ΒΜΔ = 30° ii) χ = 60° iii) AM ΔΓ.

Γ

=

\

\

\

\

\

\

Ε

i) Να εξετάσετε αν ΓΔ//ΑΒ. ii) Αν οι ΓΔ και ΑΒ δεν είναι παράλληλες, πόσες μοίρες πρέπει να είναι η γωνία Α για να είναι ΓΔ//ΑΒ; Δίνεται ισοσκελές τρίγωνο ΑΒΓ(ΑΒ ΑΓ) 7. και η διχοτόμος ΒΖ. Ο κύκλος (Γ, ΓΒ) τέμνει την πλευρά ΑΒ στο Δ και την πλευρά ΑΓ στο Ε. Να αποδείξετε ότι ΒΖ//ΔΕ. 8. Έστω ένα ορθογώνιο τρίγωνο ΑΒΓ με Α = 90° , ΑΔ το ύψος του και ΒΕ η διχοτόμος του. Να αποδείξετε ότι η διχοτόμος της γωνίας ΔΑΓ είναι κάθετη στη ΒΕ. Γ 9. Ένας μηχανικός θέλο­ Α ντας να σχεδιάσει τον Η δρόμο ΓΔ παράλληλο z στο δρόμο ΑΒ, σκέ­ Ε φτηκε να κατασκευάΔ σει ίσες τις εντός- R

ΚΥΚΛΟΦΟΡΕΙ Γ.Μ. ΣΠΗΛΙΩΊΉ

/'.

=

>

ΜΑθΗΜΑτΙΚΑ + ΑΣΕΠ

Για τους διαγωνισμούς του ΑΣΕΠ Περιέχει:

>

Πρακτική Αριθμητική Άλγεβρα Γυμνασίου-Λυκείου Όλα τα τεθέντα θέματα

ΔΙΑΓΩΝΙΣΜΑΤΑ Γ ΛΥΚΕΙΟΥ • •

-

>

Γενικής Παιδείας Κατεύθυνσης

ΜΑΘΗΜΑΤΙΚΆ Β' ΛΥΚΕΙΟΥ Κεντρική διάθεση:

Εκδόσεις Σπηλιώτη Σολωμού 3 & Θεμιστοκλέους, Αθήνα τηλ. : 210 3834533 fax: 210 3847710

ΕΥΚΛΕΙΔΗΣ Β ' λ.ζ' τ.2/47


Δpαατηpιότητα για το 5ο Κεφάλαιο {Παpαλληλ όγpαμμα} Κουνάδης Φώτης

Μια σημαία έ­ χει διαφορετικά χρώματα στις δύο όψεις της, για παράδειγμα μπλε και κίτρι­ (Σχ. Ι ) νο. (σχ. l). Θέλουμε να κόψουμε από την μπλε όψη της σημαί­ ας μια τριγωνική περιοχή, να την αντιστρέψουμε ώστε να φαίνεται το κίτρινο στρώμα και να την ε­ φαρμόσουμε πλήρως στη θέση του κενού τριγώνου. Αν η τριγωνική περιοχή που κό­ βουμε ορίζεται α­ πό ισοσκελές τρί­ γωνο ΑΒΓ ΑΓ), τότε (ΑΒ (Σχ. 2) είναι προφανές ότι κατά την αντιστροφή εφαρμόζεται πλήρως η κίτρινη τριγωνική περιοχή στο κενό τρίγωνο. (σχ. 2) Το ίδιο φυσικά συμβαίνει αν η τριγωνική περιοχή που κόβουμε ορίζεται από ισόπλευρο τρίγωνο.

Δ

=

Τότε τα τρίγωνα ΑΟΒ, ΑΟΓ, Β ΟΓ είναι ισοσκελή με βάση ΑΒ, ΑΓ και ΒΓ αντίστοιχα. Έτσι μπορούμε για καθένα από αυτά, αφού τα κ� ψουμε, να εφαρμόσουμε την προηγούμενη διαδι. κασία, όχι αμέσως ολόκληρο το τρίγωνο ΑΒΓ αλ­ λά τμηματικά με τα τρία ισοσκελή τρίγωνα, τα ο­ ποία είναι «αντιστρέψιμω>. Α 2'1 ) Το τρίγωνο ΑΒΓ είναι ορθογώνιο (Α = 90 ο ) , (ΣΧ. 4). Τότε χαράζουμε την διάμεσο ΑΟ που α­ Β (Σr. 4) ντιστοιχεί στην υποτείνουσα Β. Επειδή ΑΟ ΒΟ ΓΟ, προκύπτουν τα ισοσκελή τρίγωνα ΑΟΒ με βάση ΑΒ και ΑΟΓ με βάση ΑΓ τα οποία είναι «aντιστρέψιμα». _,..._

=

=

3'1)

Β

Το τρί­ γωνο ΑΒΓ είναι αμβλυ­ Μπορεί να πραγματοποιηθεί το παραπάνω αν η γώνιο ° τριγωνική περιοχή που κόβουμε ορίζεται από (Α = 90 ) , (σχ. 5). σκαληνό τρίγωνο; Τότε αν φέρουμε το ύψος ΑΗ που αντιστοιχεί πως είναι γίνει. απάντηση Η ναι, αυτό μπορεί να Ας το δούμε αυτό αναλυτικά, διακρίνοντας τρεις στην πλευρά ΒΓ δημιουργούνται δύο ορθογώνια τρίγωνα τα: ΑΗΒ και ΑΗΓ στα οποία αν χαράξου­ περιπτώσεις. με τις διάμεσους ΗΚ και ΗΛ στις υποτείνουσες lη) Το τρίγωνο ΑΒΓ και ΑΓ προκύπτουν τα ισοσκελή τρίγωνα ΑΒ είναι οξυγώνιο (σχ. 3). ΑΚΗ (με βάση ΑΗ), ΒΚΗ (με βάση ΒΗ), ΑΛΗ (Φέρνουμε τις μεσο­ (με βάση ΑΗ) και ΓΛΗ (με βάση ΗΓ) που αντικάθετες μι , μz , μ3 των στρέφονται. πλευρών ΑΒ , ΑΓ, ΒΓ Τελικά σε όλες τις περιπτώσεις και όχι μόνο για το αντίστοιχα που τέ­ ισοσκελές τρίγωνο μπορούμε να επιτύχουμε την μνονται στο σημείο πλήρη «αντιστροφή» αρκεί να χωρίσουμε την τρι­ (Σχ. 3 ) Ο (περίκεντρο) που γωνική περιοχή που κόβουμε σε ισοσκελή τρίγω­ είναι εσωτερικό σημείο του κύκλου. να. ΕΥΚΛΕΙΔΗΣ Β' λζ' τ.2/48


για την Β '

τάξη

του

Λυ«είου

Πολ υώνυμα

Κ. Μαλλιάκας, Βασίλης Σέϊτης, Μιχάλης Καραβόλιας

Ακολουθώντας τη σειρά με την οποία αναφέρεται το σχολικό εγχειρίδιο στα πολυώνυμα μιας μετα­ βλητής, προσπαθήσαμε να ομαδοποιήσουμε τις α­ σκήσεις σ' αυτά. Πριν παρουσιάσουμε το αποτέλεσμα, αναφέρουμε μερικά διλήμματα, που εμφανίζονται μόλις κά­ ποιος ασχοληθεί με την εφαρμογή των εννοιών και των προτάσεων στη λύση προβλημάτων.

(iv) το Ρ είναι το μηδενικό πολυώνυμο αν:

{ κ + 4 = Ο, ν + 2 = Ο, λ + 6 = Ο, μ - 1 = Ο } {::} { κ = -4, ν = -2, λ = -6, μ = 1 }

ΔΙΛΗΜΜΑ 2

Μια άσκηση - 4 εκφωνήσεις

Αρκεί να παρατηρήσουμε ότι οι εκφράσεις: " Ρ(ρ) = Ο ", "το πολυώνυμο Ρ(χ) έχει ρίζα Ρ", "το πολυώνυμο Ρ(χ) έχει παράγοντα χ-ρ", "το Ρ(χ) ΔΙΛΗΜΜΑ 1 . διαιρείται (ακριβώς) δια του χ-ρ", "η αριθμητική Βαθμός του πολυωνύμου τιμή του Ρ( χ) για χ ρ είναι Ο", "η διαίρεση (Βρίσκεται το πολυώνυμο στη λεγόμενη κανονική Ρ( χ) : (χ - ρ) είναι τέλεια", "το υπόλοιπο της του μορφή;) διαίρεσης Ρ( χ) : (χ - ρ) είναι Ο" είναι ισοδύναμες! =

ΑΣΚΗΣΗ

ΑΣΚΗΣΗ

Να βρείτε τους αριθμούς κ,λ,μ,ν αν ο βαθμός του πολυωνύμου Ρ(χ) = [(κ + 1)χ3 + 2χ 2 + λχ + μ 2] +

1 Εκφώνηση :

-

+[3χ 3 + νχ 2 + 6χ + 1]

είναι (i) 3, (ii) 1, (iii) Ο (iv) τι συμβαίνει όταν το Ρ(χ) είναι το μηδενικό πολυώνυμο; λ ύση

Το Ρ(χ) γράφεται: Ρ( χ) = (κ + 4)χ 3 + (χ + 2)χ 2 + (λ + 6)χ + μ - 1 (i) το Ρ είναι τρίτου βαθμού αν

κ + 4 :;: ο {::} κ :;: -4 (ii) το Ρ είναι πρώτου βαθμού αν { κ + 4 = Ο, ν + 2 = Ο, λ + 6 Ο } {::} { κ = -4, ν = -2, λ .= -6 } (ili) το Ρ είναι μηδενικού βαθμού αν { κ + 4 = Ο, ν + 2 = Ο, λ + 6 = Ο, μ - 1 Ο } {::} { κ = -4, ν = -2, λ = -6, μ - 1 } :ο=

:ο=

:ο=

Αν το πολυώνυμο Ρ( χ) = χ3 + αχ 2 + βχ + 4 έχει ρίζα το 2 και η αριθμητική του τιμή για χ = 1 είναι 8, να προσδιορίσετε τους αριθμούς α, β. 2 Εκφώνηση :

Για το πολυώνυμο Ρ( χ) = χ 3 + αχ 2 + βχ + 4 ισχύουν Ρ(2) = Ο και Ρ(1) = 8. Να υπολογίσετε τους αριθμούς α, β. 3 Εκφώ νηση :

Αν το πολυώνυμο Ρ(χ)=χ 3 +αχ2 +βχ+4 έχει παράγοντα το χ - 2 και το υπόλοιπο της διαίρεσής του με χ - 1 είναι 8, να βρείτε τους αριθμούς α, β. 4 Εκφώ νηση :

Αν το πολυώνυμο Ρ( χ) = χ 3 + αχ 2 + βχ + 4 διαιρείται ακριβώς με το χ - 2, ενώ το υπόλοιπο της διαίρεσής του με το χ - 1 είναι 8, να βρείτε τους αριθμούς α, β.

ΕΥΚΛΕΙΔΗΣ Β' λζ' τ.2/49


Μαθηματικά Β ' Λυκείου

Λύση

!

Άσκη ση

Όλες οι εκφωνήσεις οδηγούν στο σύστημα Ρ(2) = Ο 2 3 + α · 2 2 + β · 2 + 4 = Ο {::} {::} Ρ(1) = 8 13 + α · 1 + β · 1 + 4 = 8 4α + 2β = - 12 2α + β = -6 α : -9 {::} {::} {::} α+β=3 α+β= 3 β - 12

!

!

!

!

ΔΙΛΗΜΜΑ 3 .

Σχήμα Horner ή αλγοριθμική διαίρεση; ΑΣΚΗΣΗ 1 .

Ν α β ρείτε το υπόλοιπο της διαίρεσης Ρ( χ) : (χ - 1) όπου 003 3χ 2οο2 l χ 2οοι 2οοο χ ι999 χ + Ρ( χ) = 4χ 2 + _

_

Λύση

Η λύση προκύπτει εύκολα με τη χρήση των θεω­ ρημάτων της αλγοριθμικής διαίρεσης. Υ = P(l) = 4 . 12οο3 3 . 12002 + 2 . 1200 1 _ 12οοο + 11 999 = = 4-3+2-1+1=3 . Το σχήμα του Horner όμως, δεν είναι εύκολο να εφαρμοστεί (?). _

ΑΣΚΗΣΗ 2.

Να υπολογίσετε το α ώστε το πολυώνυμο 2 3 Ρ( χ) = 2χ - 4χ + α να έχει ρίζα -1. Ι '�

t 2

--6

6

2

-

6

α

--6

I

α--6

Ρ( χ) = 14χ 3 - 25χ 2 + 7χ - 12 ! Αλλαγή μεταβλητής

Θέτω χ 2 + χ = ψ(1) και η εξίσωση ισοδυναμεί με την (ψ + 1) ψ - = (ψ - 1)

( %) % {::} 2( ψ + 1) ( ψ -%) = 2 . % (ψ - 1) {::}

{::} (ψ + 1)(2ψ - 3) = 3(ψ - 1) {::} {::} 2ψ 2 - 3ψ + 2ψ - 3 = 3ψ - 3 {::} {::} 2ψ 2 - 4ψ = ο {::} {::} 2ψ(ψ - 2) = ο {::} {:} ψ = Ο ή ψ - 2 = 0 {::} {:} ψ = Ο ή ψ = 2 Αντικαθιστώντας στην (1) έχουμε: χ2 + χ = 0 ή χ2 + χ = 2 {:} χ 2 + χ = Ο ή χ 2 + χ - 2 = 0 {:} Χ = Ο ή Χ = - 1 ή χ = 1 ή χ = -2

Ν α λυθεί η εξίσωση 7(3χ + 2) 2 (1 - χ) 2 - (3χ + 2)(1 - χ) 3 = ο . Λ ύση

Πρέπει α - 6 = Ο {::} α = 6 . Εδώ οι δύο λύσεις είναι εξίσου προσιτές. Δείτε όμως τι συμφέρει για το Ρ

ΔΙΛΗΜΜΑ 4.

Λ ύση

ΑΣΚΗΣΗ :

2 '� Λύση

ο

%) %

Παραγοντοποίηση ή σχήμα Horn er;

Λύση .

-4

(

ΔΙΛ Η Μ Μ Α 5 .

Είναι Ρ( - 1) = Ο {::} 2( - 1) 3 - 4(- 1) 2 + α = Ο {::} {:} -2 - 4 + α = 0 {:} -6 + α = 0 {:} α = 6 . 2

Να λύσετε την εξίσωση z (x + χ + 1> χ 2 + χ - = (χ 2 + χ - 1)

(�)

όπου

Μετασχηματίζοντας την παράσταση του α' μέ­ λους σε γινόμενο παραγόντων, έχουμε: 7(3χ + 2) 2 (1 - χ) 2 - (3χ + 2)(1 - χ) 3 = ο {::} {::} (3χ + 2)(1 - χ) 2 [7(3χ + 2) - (1 - χ)] = ο {::} {::} (3χ + 2)(1 - χ) 2 (22χ + 1 3) = ο {::} {::} 3χ + 2 = ο ή (1 - χ)2 = ο ή 22χ + 13 = ο {::} 3 η' χ = 1 (δ ιπλη' ) ' η χ = - 13 {::} χ = -2 22 ΑΣΚΗΣΗ :

Να λιιθεί η aνίσωση: 2χ 3 - l lx 2 + 12χ + 9 < 0 ΕΥΚΛΕΙΔΗΣ Β ' λζ' τ.2/50


Μαθηματικά Β ' Λυκείου

Λύση

Παραγοντοποιούμε το πολυώνυμο του πρώτου μέλους με τη βοήθεια του σχήματος Horner. 2

-1 1

2

-5

• Η ανίσωση

6

12

-1

5

-3

9

-9

3

Λύση

J

Είναι χ 2 - χ - 6 = (χ + 2)(χ - 3) Αρκεί το Ρ(χ) να διαιρείται με χ + 2, χ - 3 ή ι­ σοδύναμα να έχει ρίζες -2, 3. Άρα α = -4 Ρ(-2) = Ο 2α + β = 2 {:} {:} 3α - β = -22 β = 10 Ρ(3) = Ο

ο

ισοδυναμεί με την 2 (χ - 3)(2χ - 5χ - 3) < 0 ή (χ - 3) 2 (2χ + 1) < 0 {:} 2χ + 1 < 0 και χ � 3 1 {:} χ < -2

!

5)

ΑΣΚΗΣΕΙΣ 1) Να βρεθούν οι αριθμοί α, β, γ ώστε το πο­ λυώνυμο Ρ( χ) = χ4 + αχ3 + γχ + 9 να είναι τετράγωνο του χ 2 + χ + 3 .

Αν 2 χ2 -5χ+5=Α(χ-2)(χ-3)+Β(χ+ι)(χ-2 )+ +Γ(χ+ι)(χ-3) να προσδιοριστούν τα Α, Β, Γ. Λύση

3)

Αν για το πολυώνυμο Ρ(χ) ισχύει Ρ(χ)=Ρ(3χ) για κάθε χ, τότε είναι σταθερό.

6)

Λ ύση

Θεωρούμε το Ρ( χ) = αν χ ν + αν _ 1 χ ν-1 + ... + α 1 + α0 Από την ισότητα Ρ(χ) = Ρ(3χ) έχουμε αν χ ν + αν- Ι Χ ν- 1 + ... + α ι Χ + αο αν 3 ν Χ ν + αν- Ι 3 ν- Ι Χ ν - 1 + . .. + α ι 3 Χ + αο αν = 3 ν αν α ν - Ι = 3 ν-1 α ν - Ι ν-1 {:} αν- Ι (3 - 1) = 0

{ αν αν-Ι . αι 0 Άρα Ρ( χ) = α0 (σταθερό). =

= ..

=

Να βρείτε ένα πολυώνυμο Ρ(χ), που αν διαιρεθεί με το χ 2 + χ + ι αφήνει πηλίκο 2χ-1 και υπόλοιπο 3χ+5. Λύση

Από την ταυτότητα της διαίρεσης θα είναι: Ρ( χ) = (χ 2 + χ + 1)(2χ - 1) + 3χ + 5 = 2χ 3 - χ 2 + 2χ 2 - χ + 2χ - 1 + 3χ + 5 = = 2χ 3 + χ 2 + 4χ + 4 .

_

{:}

Το υπόλοιπο της διαίρεσης ενός πολυωνύμου Ρ(χ) δια χ+2 είναι 5, ενώ το υπόλοιπο της διαίρεσής του με χ - ι είναι 2. Να βρείτε το υπόλοιπο της διαίρεσης Ρ(χ): (χ 2 +χ-2) .

Είναι χ 2 + χ - 2 = (χ + 2)(χ - 1) και Ρ( -2) = 5 και P(l) = 2 . Το υπόλοιπο της παραπάνω διαίρεσης θα εί­ ναι το πολύ 1 ου βαθμού, δηλαδή της μορφής αχ + β , οπότε Ρ( χ) = π(χ)(χ + 2)(χ - 1) + αχ + β . Για χ = -2 παίρνουμε Ρ(-2) = -2α + β = 5 . Για χ = 1 παίρνουμε Ρ(1) = α + β = 2 και από την επίλυση του συστήματος α = - 1 και β = 3. Άρα Υ(χ) = -χ + 3 .

Λύση

για χ = 2 παίρνουμε 3 = Γ · (-3) {:} Γ = - 1 για χ = 3 παίρνουμε 8 = Β · 4 {:} Β = 2 για χ = -1 παίρνουμε 12 = Α · 12 {:} Α = 1

ι

!

Λύση

Είναι χ 4 + αχ 3 + βχ 2 + γχ + 9 = (χ 2 + χ + 3) 2 {:} χ 4 + 2χ 3 + 7χ 2 + 6χ + 9 {:} α = 2 και β = 7 και γ = 6 . 2)

Αν το Ρ( χ) = 2χ 3 + αχ 2 - βχ + ι2 διαιρείται με το χ 2 - χ - 6 , να προσδιορίσετε τα α, β.

4)

7)

Ένα πολυώνυμο Ρ(χ) διαιρείται δια χ - 2. Δείξτε ότι το πολυώνυμο Ρ(2χ - 6) διαιρείται δια χ - 4. Λύση

Για το υπόλοιπο τη διαίρεσης Ρ(2χ - 6) : (χ - 4) έχουμε Υ = Ρ(2 · 4 - 6) = Ρ(2) = Ο αφού το Ρ(χ) διαιρείται δια χ - 2. ΕΥΚΛΕΙΔΗΣ Β ' λζ ' τ.2/51


Μαθηματικά Β' Λυκείου "

το (χ - 1 ) 2 είναι παράγοντας του πολυω­ ''ύμου Ρ( χ) = αχ• + βχ ν - ι + 1 , να βρείτε τους Α,.

·

αριθμούς α, β. Λύση

Πρέπει P(l) = 0 <=? α · 1ν + β · 1ν-1 +1=0 {::} α+β+1=0 (1) και αν Π(χ) το πηλίκο της διαίρεσης Ρ( χ) : (χ - 1) θα είναι και Π(1) = Ο . Κάνοντας τη διαίρεση αυτή βρίσκουμε ότι Ι Π(Χ)=αχ ν- +(α+β)χ ν-2 +( α+β)χ ν-3 + ... + +(α + β)χ + (α + β) οπότε Ι Π(l) = α· 1ν- + (α + β)1ν-2 + .... + (α+ β) = 0 <=? <=? να + (ν - 1)β = 0 (2) Επιλύοντας το σύστημα των (1) (2), α+β = -1 <=? {α = ν - 1 ' β = -ν} να + (ν - 1)β = Ο 9)

!

}

Αν το πολυώνυμο Ρ( χ) = χ3 + κχ 2 - λχ + 6 έχει ρίζα το 1 και το υπόλοιπο της διαίρε­ σής του με χ+ 1 είναι 8, να βρείτε τους α­ ριθμούς κ, λ και τις ρίζες του πολυωνύμου. Λ ύση

Εκτελούμε τη διαίρεση του Ρ(χ) με το χ + 1 : Πα­ ρόμοια εκτελούμε τη διαίρεση του πηλίκου με το: 6 κ 1 -λ -1 κ + 1 κ-λ+ 1 1 • 1 κ + 1 κ-λ + 1 κ-λ+7

I

Πρέπει κ - λ + 7 = Ο (1) κ -λ 6 -1 1 • -1 -κ + 1 κ+λ-1 1 κ + 1 -κ-λ+ 1 κ+λ+5

I

Πρέπει κ + λ + 5 = 8 (2) Θεωρούμε το σύστημα των (1 ), (2): κ - λ = -7 κ = -2 <=? λ= 5 κ+λ=3 Άρα Ρ( χ) = χ 3 - 2χ2 - 5χ + 6 το Ρ(χ) έχει ρίζα 1 , οπότε παράγοντα το χ-1 δηλα­ δή Ρ(χ) = (χ - 1)(χ2 - χ - 6)

!

!

Άρα Ρ(χ) = 0 <=? (χ - 1)(χ2 - χ - 6) = 0 <=? <=? χ - 1 = 0 ή χ2 - χ - 6 = 0 <=? <=? χ = 1 ή χ = -2 ή χ = 3 1 0) Έστω πολυώνυμο Ρ(χ) 3ου βαθμού με ρίζες

ρ ρz, ρ3 . Τότε Ρ( αχ + β) έχει ρίζες ρι - β ρ2 - β ρ3 - β , με α * 0 , α α α ι,

Α ΠΟΔΕΙΞΗ

Πράγματι: Ρ α ΡΙ β + β

(

:

)

=

Ρ(ρ 1 ) = Ο

Αντίστοιχα: Ρ α Ρ2 : β + β = Ρ α Ρ3 : γ + γ = Ο .

(

) (

)

Ι I ) Τα υπόλοιπα της διαίρεσης ενός πολυωνύ­

μου Ρ(χ) με τα χ+2 και χ-1 είναι 1 1 και 2 α­ ντίστοιχα. α) Ποιο το υπόλοιπο της διαίρεσης Ρ( χ) : (χ 2 + χ - 2)

β) Αν υ(χ) το προηγούμενο υπόλοιπο να λυ­ θεί η εξίσωση �υ( χ) + 1 = ..)3 - χ + 1 . γ) Αν Q(x) = Ρ( χ + 2003) ποιο το υπόλοιπο της διαίρεσης του Q(x) με το (χ + 2002) .. δ) Να δείξετε ότι το υπόλοιπο της διαίρε­ σης του Ρ(χ) με το χ-2004 είναι ίσο με το άθροισμα των συντελεστών του Q(x) . Λύση

α) Έστω αχ + β το ζητούμενο υπόλοιπο ( αφού ο

διαιρέτης είναι 2ου βαθμού). Σύμφωνα με την ταυτότητα της Ευκλείδειας Διαίρεσης πρέπει Ρ(χ) = (χ2 + χ - 2) · Π(χ) + (αχ + β) . Επίσης Ρ(-2) = 1 1 και P( l) = 2 . Άρα έχουμε: Ρ(-2) = [(-2)2 + (-2) - 2} Π(-2) + α· (-2) + β Ρ(1) = (12 + 1 - 2) · Π(l) + α· 1 + β 1 1 = -2α + β α = -3 <=> 2 = α+β β=5 Οπότε υ( χ) = -3χ + 5 . β) Είναι .J-3x + 5 + 1 = .J3 - x + 1 <=? . .J6 - 3x = .J3 - x + 1

ΕΥΚΛΕΙΔΗΣ Β ' λζ' τ.2/52

)

} { }


Μαθηματικά Β ' Λυκείου

Πρέπει καταρχήν 6 - 3χ 2': Ο και 3 - χ 2': Ο . Επειδή και τα δύο μέλη είναι μη αρνητικά ψώνω στο τετράγωνο και έχω: ( .)6 - 3χ )2 = ( .)3 - χ + 1 )2 {:}

Η(χ) = Ρ 2 (χ) + αΡ(χ) + β .

υ­

Αν ρ ι , ρ2 ρίζες του Ρ(χ) - χ να δείξετε ότι είναι ρίζες και του Η( χ) - χ . Λ ύση

Πρέπει Ρ(ρ 1 ) - ρ 1 = Ο και Ρ(ρ 2 ) - ρ 2 = Ο οπότε Ρ(ρ 1 ) ρ 1 και Ρ(ρ 2 ) = ρ 2 . Έχω: Η(ρι ) - ρ ι = p 2 (ρ ι ) + αΡ(ρ ι ) + β = ρ� + α · ρ ι + β - ρ ι Ρ(ρι ) - ρ ι Ο . Παρόμοια Η(ρ 2 ) - ρ 2 = Ο .

6 - 3χ = 3 - χ + 2)3 - χ + 1 *>

{:} 2 - 2χ = 2.)3 - χ {:} 1 - χ = .)3 - χ . Πρέπει επίσης 1 - χ 2': Ο οπότε υψώνοντας πάλι στο τετράγωνο κι έχω: 2 (1 - χ) 2 ( .J3 - χ ) {:} 1 - 2χ + χ 2 = 3 - χ {:} x 2 = 0 � ( x + l )( x 2 ) = 0 {:} Χ = - 1 ή Χ = 2 Οι περιορισμοί μας όμως είναι 6 - 3χ 2': Ο και 3 - χ 2': Ο , 1 - χ 2': Ο οπότε δεκτή είναι μόνο η λύση Χ = - 1 . γ) Το ζητούμενο υπόλοιπο είναι το Q( -2002) = Ρ( -2002 + 2003) = P(l) = 2 . δ) Ζητώ το Ρ(2004) = Ρ(1 + 2003) = Q(l) . Όμως το Q( 1 ) εκφράζει το άθροισμα των συντελε­ στών του πολυώνυμου Q(x).

=

=

=

χ2

-

-

-

1 2) Έστω

Π Ρ Ο Τ Ε Ι Ν Ο Μ Ε Ν Ε Σ ΑΣ ΚΗΣ Ε Ι Σ

1)

2)

3)

Ρ(χ) = (λ2 - 4) · χ4 + χ 3 - 5χ 2 + 6 χ + 4λ + 6 . α) Αν το Ρ(χ) ρίζα το 1 να βρεθεί ο βαθμός του. β) Ποιες οι άλλες ρίζες του Ρ(χ).

4)

α) Πρέπει P(l) = Ο οπότε έχουμε: .

5)

.

'

1 3)

'

Έστω Ρ( χ) = χ 2 + αχ + β και

Θεωρούμε το πολυώνυμο Ρ( χ) = χ 3 - 2αχ 2 + βχ + 6 . Αν το Ρ(χ) διαιρείται με το χ 2 - 5χ + 6 να υπολογίσετε τους αριθμούς α, β. Πολυώνυμο Ρ(χ) διαιρούμενο με χ+ 1 δίνει υ­ πόλοιπο 2, ενώ διαιρούμενο με χ-2 δίνει υπ&. λοιπο 1 1 και διαιρούμενο με χ+ 3 δίνει υπ&. λοιπο 6. Ποιο το υπόλοιπο της διαίρεσης του Ρ(χ) με το (χ + 1)(χ - 2)(χ + 3) . Ποια τα α, β ώστε το πολυώνυμο Ρ(χ) = 2χ 3 + αχ 2 - 1 3χ + β να διαιρείται με το (χ - 3)(χ + 2) Αν το Ρ(χ)=χ ν +ξψ ν +η · z ν (ν ε Ν, ν 2': 2) διαιρείται με το Q(x)=x 2 -(αψ+ βz)χ+αβψz .

.\ ύση

(λ2 - 4) . 1 4 + 1 3 - 5 1 2 + 6 . 1 + 4λ + 6 = ο {:} ι: -4+1-5+6+4λ+ 6= 0{:} JC + 4λ+ 4= 0{:} (λ + 2) 2 = ο {:} λ = -2 Άρα Ρ( χ) = χ 3 - 5χ 2 + 6χ - 2 , επομένως έχει βαθμό 3. Με την βοήθεια του σχήματος Homer: Άρα Ρ( χ) = Ο {:} (χ - 1)(χ 2 - 4χ + 2) = Ο {:} χ = 1 η χ 2 - 4χ + 2 = 0 ' τελικα χ = 1 η χ = 4 ± .J8 δη λ. οι ριζες οποτε 2 του Ρ(χ) είναι: 1, 2 + J2,2 - .fi .

=

6)

'

7)

τότε θα ισχύει ότι __5__ν + �ν + 1 = Ο . α β Αν ν ε Ν* και Ρ(χ)=(χ+ 1) 2ν -χ 2ν -2χ-1 με ν 2': 2 να δείξετε ότι το Ρ(χ) διαιρείται με το 2χ 3 + 3χ 2 + χ . Τα υπόλοιπα των διαιρέσεων Ρ( χ) : (χ + 1) και Ρ( χ) : (χ - 3) είναι -4 και -164 αντίστοιχα. α) Ποιο το υπόλοιπο της διαίρεσης Ρ(χ) : (χ 2 - 2χ - 3) . β) Αν το Ρ(χ) είναι 4ου βαθμού με ρίζες 0,2,2. Ποια η τέταρτη ρίζα του; α) Πολυώνυμο Π(χ) έχει την ιδιότητα Π( χ) = Π(l - χ) για κάθε χ ε IR . Να δείξετε ότι το Π(χ) - Π(Ο) διαιρείται από το Χ · (χ - 1) .

ΕΥΚΛΕΙΔΗΣ Β' λζ ' τ.2/53


Μαθηματικά Β' Λυκείου

β) Έστω πολυώνυμο Ρ(χ) ώστε Ρ(χ)=Ρ(χ-1) για κάθε χ. Να δείξετε ότι το Ρ(χ) είναι σταθερό πολυώνυμο. 8) Έστω Ρ( χ) = χ 3 - λχ 2 + 1 9χ - λ - 4 και Η(χ) = χ 3 - (λ + 1)χ 2 + 23χ - λ - 7 . Ποιο το λ ώστε το κλάσμα Ρ( χ) να απλο­ Η(χ) ποιείται και να το γράψετε στην απλούστερη δυνατή μορφή.

a

Ο

λόγος

των

9)

1 Ο)

ν

εμβαδών δύο τριγώνων

Βασίλης Σείτης

κορυφών χωρίζει το τετράπλευρο που σχηματί­ ζουν οι υπόλοιπες κορυφές τους σε δύο ισοδ� ναμα σχήματα.

lo Θ ε ώ ρη μα

Αν μια γωνία ενός τριγώνου είναι ίση ή παρ/κή με μία γωνία ενός άλλου τριγώνου, τότε ο λόγος των εμβαδών των δύο τριγώνων είναι ίσος με το λόγο των γινομένων των πλευρών που περιέχουν τη γωνία αυτή. Π ρ όβ λη ματα με π ρ ο εκτάσε ις στα ο π οία ε­ φ α ρ μόζετα ι τ ο π ρ οη γ ού μεν ο θ ε ώ ρ η μα .

i.

Αν υ 1 (χ) και υ 2 (χ) τα υπόλοιπα των διαιρέ­ σεων f1 (χ) : δ( χ) και f2 (x) : δ( χ) όπου f1 (χ), f2 (χ), δ(χ) :;: Ο πολυώνυμα τότε ισχόει: το δ(χ) διαιρεί την διαφορά fι (χ) - f2 (x) {::} υ ι (χ) = υ 2 (χ) . Αν υ 1 (χ), υ 2 (χ), .... , υ (χ) είναι αντίστοιχα τα υπόλοιπα των διαιρέσεων fi (χ) : δ( χ) για i = 1 ,2, ... ,ν τότε η διαίρεση [fι (x) + f2 (x) + ... + fv (x)] : δ(x) έχει υπόλοι­ πο υ( χ) = υ 1 (χ) + υ2 (χ) + ... + υΛ χ) .

Αν Δ,Ε,Ζ είναι τα μέσα των πλευρών τριγώνου ΑΒΓ, τότε να δείξετε ότι: 1 (ΔΕΖ) = -(ΑΒΓ) και 4

Απ αντή σ εις

i.

Το τετράπλευρο ΔΕΓΖ είναι παραλληλόγραμμο. Άρα Γ = Δι , άρα από το θεώρημα (ΑΒΓ) = ΑΓ · ΒΓ = 2ΔΖ · 2ΔΕ = 4 ( ΔΕΖ) ΔΕ · ΔΖ ΔΕ · ΔΖ �

Α

{::} (ΔΕΖ) = _!_ (ΑΒΓ) 4

Αφού το τετράπλευρο ΔΕΓΖ παρ/μο, τότε (ΔΕΖ) = (ΕΖΓ) και όμοια (ΑΔΕ) = (ΔΕΖ) = (ΒΔΖ) .

Α

(ΕΖΓ) = (ΒΔΖ) = (ΑΔΕ) = .!_ (ΑΒΓ) . 4

ii.

Αν Κ, Λ, Μ, Ν είναι τα μέσα των πλευρών τε­ τραπλεύρου ΑΒΓ Δ, να δείξετε ότι

Δ

(ΚΛΜΝ) = .!_ (ΑΒΓΔ) .

2 iii. Αν ω η γωνία των διαγωνίων δ ι ,δ2 του τετρα­ πλεύρου ΑΒΓΔ τότε (ΑΒΓΔ) = .!_ δ ι · δ 2 · ημω 2 και ακόμα ένα: iv. Καθένα από δύο ισοσκελή και ορθογώνια τρί­ γωνα έχουν το ένα την κορυφή της ορθής γωνί­ ας στην υποτείνουσα του άλλου και κάθε κο­ ρυφή είναι στο εσωτερικό της άλλης ορθής γω­ νίας. Να δείξετε ότι το ευθύγραμμο τμήμα των

Β

ii. (ΚΛΜΝ) =

π �

-

',

'

- - - - -

'

,

I ' I

I

I

,'

Ε

z

(ΑΒΓ Δ) - (ΑΚΛ) -

Γ

-(ΒΛΜΗΓΜΝ}-(ΚΔΝ)=(ΑΒΓΔ) - _!_ (ΑΒΔ) 4

_ .!_ (ΑΒΓ) _ .!_ (ΒΓΔ) _ _!_ (ΑΔΓ) = 4

4

4

= (ΑΒΓΔ) - _!_ [(ΑΒΔ) + (ΒΓΔ)] 4

_ .!_ [(ΑΒΔ) + (ΑΔΓ)] = 4

ΕΥΚΛΕΙΔΗΣ Β' λζ' τ.2/54


Μαθηματικά Β ' Λυκείου

Να δείξετε ότι: (ΟΚΒΛ)+ (ΟΜΔΝ) (ΟΝΑΚ)+ (ΟΛΓΜ) . =

Α

Α Δ

Β

Γ

Γ

Απάντηση

(Θυμηθείτε ότι το ΚΛΜΝ είναι παραλληλό­ Στο τρίγωνο (ΟΑΒ) η ΟΚ είνα� διάμεσός του. Άρα γραμμο). (ΟΑΚ) (ΟΚΒ) (από Ι (α)). iii. και Όμοια =

Αν προσθέσουμε αυτές τις ισότητες κατά μέλη έ­

iv.

χουμε:

δηλ. Ε

Α

είναι αφού

z

και

και (παρ/κές)

άρα (ΑΕΓΖ) = (ΕΒΔΓ).

β) Σε κυρτό τετράπλευρο ΑΒΓΔ χωρίζουμε κάθε πλευρά του σε τέσσερα ίσια τμήματα και σχη­ ματίζεται το πλέγμα, όπως στο σχήμα. Να δεί­ ξετε ότι το άθροισμα των εμβαδών των λευκών τετράπλευρων είναι ίσο με το άθροισμα των εμβαδών των μαύρων τετράπλευρων. εφαρμόστε την 2(β) στα κυρτά τετράπλευρα ΕΒΖΚ, ΖΚΗΓ , ΚΗΔΘ και ΕΚΘΑ Α

λ σ κηση γ ια λύση :

Δίνεται το τρίγωνο ΑΒΓ . Στο εξωτερικό του κατα­ σκευάζουμε τα τετράγωνα ΑΒΔΕ , ΑΓΖΗ και ΒΓΘΙ. Να δείξετε ότι ΣΧΕΊΙΚΑ Μ Ε Ί Η ΔΙΆΜΕΣΟ Ί Ρ Ι ΓΩΝΟΥ 2ο Θ εώρη μα

3 ο Θ εώρη μα

Δίνεται ένα τρίγωνο ΑΒΓ και η παράλληλη (ε) από Η διάμεσος ενός τριγώνου χωρίζει το τρίγωνο το Α προς τη ΒΓ . Για κάθε σημείο Μ της (ε) είναι σε δύο ισεμβαδικά τρίγωνα.

(ΜΒΓ) (ΑΒΓ) . =

(εφαρμογή (3) σχολικού σελ. 2 1 6). α) Έστω τετράπλευρο ΑΒΓΔ, τα μέσα Κ, Λ, Μ, Ν των πλευρών ΑΒ, ΒΓ, ΔΓ, ΔΑ αντίστοιχα και Ο το σημείο τομής των ΚΜ και ΛΝ .

ΕΥΚΛΕΙΔΗΣ Β' ΊJ:.; τ.2/55

Απ άντηση

και


Μαθηματικά Β' Λυκείου

Αλλά ΑΑ' = ΜΜ ' (απόσταση των παρ/λων ΒΓ και ε) άρα (ΑΒΓ) (ΜΒΓ). =

Α

Μ

ε

., , "" ,' ι Ι ι ι Ι 1 ι 1 ι

,__�--�

- -Μ.-; - - -

{::?

(ΕΒΖ) = (ΜΒΓ) (l). Οπότε αρκεί να δείξουμε ότι (ΑΔΖ) = (ΒΜΓ) = (ΑΜΓ) [αφού ΓΜ διάμε­ σος του ΒΓ άρα (ΒΜΓ) = (ΑΜΓ) ]. Έχουμε: (ΑΔΖ) = (ΑΔΜ) + (ΔΜΖ) και (ΑΜΓ) = (ΑΜΔ) + (ΜΔΓ) . Επίσης ΖΓ//ΜΔ άρα (ΜΔΖ) = (ΜΔΓ) οπότε (ΑΔΖ) = (ΑΜΓ) .

α) Επί της πλευράς ΒΓ ενός τριγώνου ΑΒΓ θεω­ 4ο Θ εώρη μα ρούμε τα σημεία Δ, Ε τέτοια ώστε Αν δύο τρίγωνα είναι όμοια τότε ο λόγος των ΒΔ = ΓΕ < � . Η παράλληλη της ΑΒ από το Δ εμβαδών τους είναι ίσος με το τετράγωνο του 2

τέμνει την ΑΓ στο Ζ ενώ οι ΒΖ, ΑΕ τέμνονται λόγου ομοιότητάς τους. α) Δίνεται τρίγωνο ΑΒΓ με εμβαδόν Ε. Από ένα στο Θ. Να δείξετε ότι (ΘΑΒ) = (ΘΕΓΖ). σημείο Δ της ΑΒ φέρνουμε ευθεία παράλληλη Α στη ΒΓ που τέμνει την ΑΓ στο Η, έτσι ώστε να είναι ΑΔ = . Να δείξετε ότι 1 6Ε 25(ΑΔΗ). ΑΒ 5 ' '

'

'

'

Ε

=

Α

Γ

Απ άντηση

(ΑΒΔ) (ΑΕΓ) ( 1 ) αφού έχουν ίσες πλευρές (ΒΔ =

=

ΕΓ) και κοινή κορυφή την Α (δηλ. κοινό ύψος).

(ΑΖΒ) = (ΑΒΘ) + (ΑΘΖ) και (ΑΕΓ) = (ΑΘΖ) + (ΘΖΓΕ) οπότε από ( 1 ) και (2)

Απ άντη ση

Επειδή ΔΗ // ΒΓ τότε τα τρίγωνα ΑΒΓ, ΑΔΗ είναι όμοια, άρα από 4(α)

(�)2

(ΑΔ)2

έχουμε ότι {::? (ΑΔΗ) (ΑΔΗ) (ΑΒΘ) + (ΑΘΖ) = (ΑΘΖ) + (ΘΖΓΕ) {::? {::? = = (ΑΒΓ) ΑΒ (ΑΒΓ) 5 {::? (ΑΒΘ) = (ΘΕΓΖ) β) Από το μέσο Μ της πλευράς ΑΒ ενός τριγώνου {::? 1 6(ΑΒΓ) = 25(ΑΔΗ) ΑΒΓ φέρουμε ευθεία xy που τέμνει την πλευρά β) Θεωρούμε κύκλο (O,R) και μια χορδή του ΑΓ στο Δ και την προέκταση της ΒΓ στο Ε. Αν ΑΒ = l R . Στο Β φέρνουμε εφαπτομένη του 2 η παράλληλη της xy από το Γ τέμνει την ΑΒ κύκλου και από το Α την ΑΓ κάθετη σ' αυτήν. στο Ζ, τότε (ΑΔΖ) = (ΒΕΖ). Αν ΑΔ είναι η διάμετρος του κύκλου να αποΑ Υ Δ δείξετε ότι (ΑΒΔ) = � . (ΑΒΓ) . 9

I I I \ I \

z

I

I

I

""

I

I

Ι

Ι �

;

,

·

Δ

''

' '

Απ άντη ση

Είναι ΜΕ/!ΓΖ άρα (ΕΓΖ) = (ΜΓΖ) {::? (ΕΒΖ) + (ΒΖΓ) = = (ΜΒΓ) + (ΒΖΓ) {::?

Απ άντηση

90 °

Είναι ΑΒΔ (εγγεγραμμένη που βαίνει σε ημικύκλιο). Επίσης ΑΒΓ = Δ (γωνία που σχηματί=

ΕΥΚΛΕΙΔΗΣ Β ' λζ' τ.2/56

-


Μ αθηματικά Β ' Λυκείου

ζεται από χορδή και εφαπτόμενη). Άρα τα ορθ� γώνια τρίγωνα ΑΒ Δ και ΑΒΓ είναι όμοια. Οπότε από 4( α) είναι 2 2 2R = � δηλ. (ΑΒΔ) = ΑΔ (ΑΒΓ) ΑΒ 9 iR 2

Απάντηση Α

( )

I I I I

z

, ,

��e

""' .... .J *"' "" .,. , .... ....

_.

.. ..

Ε

I I

(ΑΒΔ) = � (ΑΒΓ) 9

Αν Ε το εμβαδόν του τριγώνου με πλευρές ίσες με γ) Έστω τρίγωνο ΑΒΓ με εμβαδόν Ε και ύψος τις διάμεσους του ΑΒΓ, τότε το τρίγωνο αυτό θα ΑΔ. Φέρουμε τη διάμετρο ΑΖ του περιγεγραμ­ είναι όμοιο με το ΘΑ 'Γ (ΑΘ = ΘΑ ' ) γιατί μένου κύκλου του τριγώνου ΑΒΓ . Να δείξετε 2 2 ΘΓ = } μr , ΘΑ = 3 μ α και ΓΑ = 2ΘΕ (γιατί;) 4Ε 2 (ΑΓΖ) ότι (ΑΔΒ) = ---τ-τ . 2 αβ = άρα = = μβ ), άρα , ' Γ) (Θ Απύ.ντ η σ η ι

Ι

� (;: ) i

j

Είναι ArZ = 90° (εγγεγραμμένη σε ημικύκλιο) Ε = 2_(ΘΑ'Γ) = 2_ 2(ΘΓΔ) . (ΓΔ διάμεσος του 4 4 και Β = Ζ (εγγεγραμμένη που βαίνουν στο ίδιο δηλ. Ε = 2_ _!_(ΑΒΓ) . τόξο ΑΓ ). Άρα τα ορθογώνια τρίγωνα ΑΒΔ και ΓΘΑ') άρα Ε = 2_(ΘΓΔ) 2 6 2 ΑΓΖ όμοια από 4(α) (γιατί; θυμηθείτε τι ισχύει για τη διάμεσο τριγώνου) Άρα Ε = �(ΑΒΓ) . 4 �

--

_

5 ο Θ ε ώ ρ η μα

( ) ( )

z

(ΑΒΔ) ΑΒ 2 = γ 2 = γ 2 (1). Είναι όμως = ΑΖ 2R (ΑΓΖ) 4R 2 αβγ (ΑΒΓ) = 4R 4Ε 2 = γ {:} -γ2 αρα αβγ 2 Ε = , η (1) Ε = - {:} 2 2 2 4R αβ 2R α β 4R γίνεται 4Ε 2 (ΑΔΒ) 4Ε 2 {::} = 2 2 (ΑΔΒ) = 2 2 (ΑΓΖ) (ΑΓΖ) α β αβ

Έστω τρίγωνο ΑΒΓ με πλευρές ΑΒ = 12, ΒΓ= 5, ΑΓ = 13. Να βρεθεί το εμβαδόν του τριγώνου που έχει κορυφές τα σημεία επαφής των πλε\}­ ρών του τριγώνου ΑΒΓ με τον εγγεγραμμένο του κύκλο. Απ ά ντη ση

1°ς τρόπος

.

Α

Υ�

Δ

,

, '

I

I

ω

δ) Να δείξετε ότι το εμβαδόν ενός τριγώνου που έχει πλευρές ίσες με τις διαμέσους ενός τριγώ- (Παρατηρήστε ότι το ΑΒΓ είναι ορθογώνιο αφού νου ΑΒΓ είναι ίσο με τα � του εμβαδού του οι αριθμοί 13, 12,5 είναι ορθογώνια τριάδα). 4 Για το τρίγωνο ΑΒΓ είναι 2τ = ΑΒ + ΒΓ + ΓΑ ΑΒΓ. {::} τ = 1 5 και από τον τύπου του Ήρωνα ΕΥΚΛΕΙΔΗΣ Β' λζ' τ.2/57


Μαθηματικά Β ' Λυκείου

(ΑΒΓ) = �τ(τ - α)(τ - β)(τ - γ) = 3 0 (1). Επίσης είναι χ + y = 12, y + ω = 1 3 και χ + ω = 5 οπότε λύνοντας το σύστημα έχουμε χ = 2, y = 1 0, ω = 3. Τα τρίγωνα ΑΒΓ και ΕΖΓ έχουν τη γωνία Γ κοινή οπότε (ΕΖΓ) = � {::} (ΕΖΓ) = _2._ (ΑΒ Γ) . (ΑΒΓ) 5 · 1 3 65 Παρόμοια η γωνία Β είναι κοινή στα τρίγωνα ΔΒΖ και ΑΒΓ οπότε (ΔΒΖ) 2 · 2 {:} (ΔΒΖ)= _i_ (ΑΒΓ) 60 (ΑΒΓ) 5 · 12 και η γωνία Α κοινή στα τρίγωνα ΔΒΖ και ΑΒΓ οπότε (ΑΔΕ) = 1 Ο · 1 Ο {::} (ΑΔΕ) = 1 00 (ΑΒΓ) (ΑΒΓ) 12 · 1 3 1 56 άρα (ΔΕΖ) = (ΑΒΓ) - (ΕΖΓ) - (ΔΒΖ) - (ΑΔΕ) δηλ. (ΔΕΖ) = 30 - 4, 2 - 2 - 19, 2 = 4, 6 τ.μ.

r

2°ς τρόπος: Διαφέρει από τον 1 ο τρόπο ως προς τον υπολογισμό των εμβαδών των τριγώνων ΔΒΖ, ΕΖΓ, ΑΔΕ. Δηλαδή:

1 1 1 '2 '2 '2 .αρα ημΑ = 2 · 3 0 = 12 5 · 1 3 13 2 · 30 = 1 και ημΑ = 2 · 30 = 5 ημΒ = -1 2 · 13 13 5 · 12 . (ΕΖΓ) = 1 ω · ω · ημΓ = 1 3 · 3 · 12 = 4, 2 τ.μ. αρα 13 '2 '2 1 1 (ΔΒΖ) = - χ · χ · ημΒ = - 2 · 2 · 1 = 2 τ.μ. 2 2 1 1 5 (ΑΔΕ) = '2 y · y · ημΑ = 1 0 · 10 · 13 = 1 9, 2 τ. μ. και '2 συνεχίζουμε όμοια με τον 1 ο τρόπο. (ΑΒΓ) = 5 · 13ημΓ = 5 · 12 · ημΒ = 1 2 · 1 3ημΑ

--

Εuθείες

ΜΕ ΡΟΣ 1 ° : Θε ω ρ ία

Κώστας Μαλλιάκας

ΣΧΟΛΙΑ

1) Ο συντελεστής διεύθυνσης λ της ευθείας λέγε­ ται και κλίση της ευθείας και είναι λ = εφω ό­ που ω η γωνία που σχηματίζει η ευθεία με τον θέματα ένα τρ όπο σκέ ψης για τη λύση ασκήσεων άξονα χ ' χ. Αν όμως ω = 90° τότε δεν ορίζεται λ και η ευθεία είναι κάθετη στον χ ' χ με εξίσωση πάνω στις ευθείες. της μορφή χ = χσ. Μ ΟΡΦΕΣ 2) Η γενική μορφή μπορεί να μας δώσει τις άλλες • Η εξίσωση της ευθείας, που περνάει απ' την περιπτώσεις με κατάλληλες τιμές των Α, Β, Γ. αρχή των αξόνων, είναι y = λχ. Πράγματι για Α = Ο είναι παράλληλη στον άξο­ • Η εξίσωση της ευθείας, που τέμνει τους άξονες να χ ' χ, για Β = Ο είναι παράλληλη στον άξονα σε δύο σημεία, είναι y = λχ + β. y ' y ενώ για Γ = Ο περνάει από αρχή ξένων. • Η εξίσωση της ευθείας, που είναι παράλληλη 3) Στην γενική μορφή για Β += Ο είναι λ = Α προς τον άξονα χ ' χ, είναι y = β. Β • Η εξίσωση της ευθείας, που είναι παράλληλη ενώ για Β = Ο δεν ορίζεται λ. προς τον άξονα y ' y, είναι χ = χ0• ΜΕ ΡΟΣ 2 ° : ΒΑΣΙΚΑ Θ Ε Μ Α Τ Α • Η εξίσωση της ευθείας, που διέρχεται απ' το σημείο Ρ(χο, Υο) κι έχει συντελεστή διεύθυνσης Σχετικά με μια ευ θεία τη ς μορ φής: y-yo = λ(χ-χσ). Συνήθως ζητείται να βρεθεί το λ ή ένα σημείο της λ, είναι y - Υο = λ(χ - χσ). Ρ(χο, Υο). • Η γενική μορφή της εξίσωσης ευθείας είναι: α) Για το λ: Αχ + By + Γ = Ο με I A I + IBI ;c Ο

Σ' αυτό το άρθρο μετά από ορισμένες επισημάνσεις

της θεωρίας των ευθειών θα αναφερθούμε σε απλές εφαρμογές τους και θα δείξουμε με κάποια βασικά

- ­

ΕΥΚΛΕΙΔΗΣ Β ' λζ' τ.2/58


Μαθηματικά Β ' Λυκείου

Αν ξέρω δύο σημεία: Α(χι ,ψ ι ) και Β(χ2 ,ψ2) με χ 1 7: χ τότε λ = ψ2 Ψ ι 2 χ - χ, 2 ii. Αν η ευθεία ε είναι 1 1 σε ευθεία ζ τότε λε = λς iii.Aν η ευθεία ε είναι ..l σε ευθεία ζ τότε λε · λς = - 1 iν. Αν η ευθεία ε σχηματίζει γωνία ω με τον χ ' χ τότε λε = εφω (ω 7: 90° ) ν. Ανάλογα αν η ευθεία ε είναι 1 1 ή ..l σε διά­ νυσμα. ι.

·

3.

Να βρεθεί η μεσοπαράλληλη ε των ευθειών 2χ + ψ - 4 = 0 και 2χ + ψ + 2 = 0 καθώς και η απόστασή τους. Απ άντ ηση

ε θα έχει τον ίδιο συντελεστή διεύθυνσης με τις δύο ευθείες οπότε λ -2. Ένα σημείο της ε θα εί­ ναι το μέσο Μ οποιουδήποτε τμήματος ΑΒ με ά­ κρα στις δύο ευθείες. Για Α(Ο,4) και Β(Ο,-2) είναι Μ(0, 1 ) και έτσι έχουμε: ψ - 1 = - 2(χ - 0) <=> 2χ + ψ - 1 = Ο η ζητούμενη μεσοπαράλληλη ε. Η απόσταση των δύο ευθειών είναι επίσης η απ� σταση του Α από την δεύτερη ευθεία δηλαδή 1 2 · 0 + 4 + 2 1 � 6J5 = = d= J5 5 22 + 12 � Η

=

β) Για το σημείο Ρ(Χο,ψο) : i. Αν είναι μέσο ευθ. τμήματος ΑΒ με Α(χι ,ψ ι ) Χ και Β(χ2 ,ψ2) τότε χ 0 = ι + χ 2 και 2 Γενίκευ ση : Έστω δύο παράλληλες ευθείες ψ _ Ψι + Ψ 2 Αχ + Βψ + Γ1 = 0 και Αχ + Βψ + Γ = 0 . Η μεο 2 2 ii. Αν είναι σημείο τομής δύο άλλων ευθειών ή σοπαράλληλή τους είναι η Αχ + Βψ + Γ = Ο με άλλων «γραμμών)) τότε λύνω το σύστημα Γ = Γι + Γ2 . 2 των εξισώσεών τους. 2 ) Σχετικ ά μ ε τη γεν ική μ ορφή Αχ+Αψ+Γ=Ο ΘΕΜΑ: Έστω η

εξίσωση (Ε): (λ2 + λ - 2)χ + (λ2 + 2λ)ψ + (i - 3λ - 1 0) = ο 1 . Ποια είναι η εξίσωση ευθείας ε που περνάει a) Πότε παριστάνει ευθεία; από το μέσο Μ του τμήματος ΑΒ με Α(3,8) b) Πότε είναι παράλληλη στον άξονα χ ' χ; και Β(-7,2 ) και είναι 1 1 στην ευθεία c) Πότε είναι παράλλη�η στον άξονα y ' y; ζ : 2χ + ψ - 2003 = ο . d) Πότε περνάει από την αρχή των αξόνων; Α π άν τ ηση e) Είναι δυνατόν να σχηματίζει γωνία 1 35° με τον Είναι λε = λς = -2 και Μ(-2,5) οπότε η εξίσωση χ ' χ; της ευθεία ε είναι ψ - 5 = -2(χ + 2) δηλαδή f) Να δείξετε ότι όλες οι παραπάνω ευθείες πε� 2χ + ψ - 1 = 0 . νούν από σταθερό σημείο δηλαδή αποτελούν δέσμη ευθειών. 2. Ποια είναι η εξίσωση ευθείας ε που περνάει Π α ρ αδ ε ίγ μ ατα

από

το

3χ + ψ = 7

σημείο τομής των ευθειών και 4χ - 3 ψ = 5 και σχηματίζει

γωνία 45° με τον χ 'χ. Απ άντηση

Απ άντη ση

a) Πρέπει λ2 + λ - 2 7: Ο ή λ2 + 2λ 7: Ο δηλαδή να μην μηδενίζονται συγχρόνως τα Α και Β. Αν λ2 + λ - 2 7= 0 τότε λ 1 ή λ -2. Αν λ2 + 2λ 7: Ο τότε λ Ο ή λ -2. Άρα μόνο για λ -2 μηδενίζονται συγχρόνως οπότε για να παριστάνει ευθεία η (Ε) πρέπει λ 7: -2 . Πράγματι για λ -2 η (Ε) γίνεται Οχ + Οψ = Ο που επαληθεύεται από κάθε σημείο (χ,ψ) δηλαδή η (Ε) παριστάνει τότε όλο το επίπεδο . =

Είναι λ = εφ45° = 1 και για το σημείο Ρ επιλύου­ με το σύστημα 3χ + ψ = 7 και βρίσκουμε Ρ(2, 1) 4χ - 3ψ = 5 οπότε η εξίσωση της ευθείας ε είναι ψ - 1 = 1(χ - 2) ή αλλιώς χ - ψ - 1 = 0 .

=

=

ΕΥΚΛΕΙΔΗΣ Β ' λζ' τ.2/59

=

=

=


Μαθηματικά Β ' Λυκείου

b) Για να είναι παράλληλη στον άξονα χ· χ πρέπει '>! + λ - 2 = Ο και λ 7: -2 οπότε λ = 1 . Πράγματι για λ = 1 έχω 3ψ - 12 = Ο # ψ = 4 . c) Για να είναι παράλληλη στον άξονα Υ Ύ πρέπει λ2 + 2λ = 0 και λ 7= -2 οπότε λ = Ο. Πράγματι για λ = Ο έχω -2χ - 1 0 = 0 # Χ = -5 . Για να περνάει από την αρχή των αξόνων πρέ­ πει λ2 - 3λ - 1 0 = Ο και λ 7: -2 . Οπότε λ=5. Πράγματι για λ = 5 έχω 28χ + 35ψ = Ο # ψ = -0, 8χ . Αν σχηματίζει γωνία 135 ° με τον χ ' χ πρέπει λ2 + λ - 2 3 φ 135 ° και λ 7: ο και λ 7: - 2 ')! + 2λ . εχω . - λ2 + λ - 2 = - 1 δη λαδη' οποτε ')! + 2λ 2 2 λ + λ - 2 = λ + 2λ # λ = -2 απορρίπτεται. f) Μετασχηματίζω την εξίσωση (Ε) σαν πολυω­ νυμική ως προς λ και μετά από πράξεις έχω: (χ + ψ + 1) · λ2 + (χ + 2ψ - 3) · λ + (-2χ - 1 0) = . Αν περνάει από σταθερό σημείο θα πρέπει οι συντεταγμένες του (χ, ψ) να επαληθεύουν την παραπάνω εξίσωση για κάθε λ οπότε το πολυώ­ νυμο θα είναι το μηδενικό. Άρα χ + ψ + 1 = Ο και χ + 2ψ - 3 = 0 και -2χ - 1 0 = 0 δηλαδή χ = -5 και ψ = 4. Επομένως όλες οι παραπάνω ευθείες διέρχονται από το σημείο Κ(-5,4) γιατί οι συντεταγμένες του ικανοποιούν την (Ε) για κάθε λ.

d) e)

Διαφορετικά πρέπει - 1 # λ = -2 . λ ΑΒ = λΑΓ # -λ = 6 3 Άρα b) Είναι (ΑΒΓ) =

�l det(AB,AΓI .

� l -6 - 3λl = 12

c)

{:::}

1 6 + 3λl = 24

{:::}

λ = 6 ή λ = -10. Πρέπει ΑΒ ..l ΑΓ . - Άρα ΑΒ · ΑΓ = Ο # 1 8 - λ = Ο # λ = 1 8

2° ΘΕΜΑ: Δίνονται τα σημεία: Α(3,5), Β(5,-9),

Γ(-3,7) a) Να αποδειχθεί ότι σχηματίζουν τρίγωνο. b) Να βρεθούν οι εξισώσεις των πλευρών, διαμέ­ σων, μεσοκαθέτων και υψών του καθώς και οι συντεταγμένες του βαρύκεντρου, περίκεντρου και ορθόκεντρου του. c) Ποια τα μήκη πλευρών, διαμέσων και υψών; Ποιο το εμβαδόν του τριγώνου ΑΒΓ; Ποια η ακτίνα του περιγεγραμμένου κύκλου του τριγώνου ΑΒΓ;

d) e)

ΥΠΌΔΕΙΞΗ :

5+9 - 9 - 7 = -2. Οπότε: a) λ ΑΒ = -- = -7, λ 8r = -3-5 5+3 λ ΑΒ :F- λ ΒΓ " b) Για την πλευρά· ΒΓ έχω: λ 8r = 7 + 9 = -2 . -3 - 5 Άρα είναι ψ - 7 = -2(χ + 3) # 2χ + ψ - 1= Ο . Παρόμοια για τις πλευρές ΑΒ και ΑΓ. Για την διάμεσο ΑΜ έχω: μέσο της ΒΓ το - 1 - 5 = 3 οπότε είναι M(l ,-1) και λΑΜ = -3 ) Ευθ ε ία π ου διέρ χεται από τ ρία ση μ εί α 1-3 του Ε πιπέδου ψ + 1 = 3(χ - 1) # 3χ - ψ - 4 = Ο . l0 ΘΕΜΑ: Έστω Α(λ, λ + 1), Β(λ + 6, 2λ + 1), Παρόμοια και για τις άλλες διαμέσους. Γ(λ + 3, λ) Για την μεσοκάθετο ε της ΒΓ έχω ότι περνάει a) Πότε τα Α,Β,Γ είναι συνευθειακά; από το M(l ,- 1) και λε λ8r = - 1 . b) Αν ορίζουν τρίγωνο ΑΒΓ ποιο είναι το λ ώστε Άρα λε = Ο, 5 οπότε έχω: (ΑΒΓ) = 12 τ.μ.; ψ + 1 = 0, 5(χ - 1) # χ - 2ψ - 3 = Ο . c) Ποιο είναι το λ ώστε το τρίγωνο ΑΒΓ να είναι Ομοίως για τις άλλες μεσοκαθέτους. ορθογώνιο στο Α; Για το ύψος ΑΔ έχω: περνάει από το Α(3,5) και λΑΔ · λ8r = - 1 οπότε λΑΔ = 0, 5 . πάντ Α ηση Άρα ψ - 5 = 0, 5 (χ - 3) # χ - 2ψ + 7 = Ο . a) Είναι ΑΒ = (6, λ) και ΑΓ = (3, - 1) οπότε Ομοίως για τα άλλα ύψη. -- 6 λ = -6 - 3λ . Για το βαρύκεντρο θα πρέπει να λύσουμε το 3 _1 ·

det(AB,AΓ)= ι ι

ΕΥΚΛΕΙΔΗΣ Β' λζ' τ.2/60


Μαθηματικά Β' Λυκείου

νείται το Δ και προφανώς ε 1 11 ε 2 . Επίση .; ισ-f.ύει σύστημα των εξισώσεων δύο διαμέσων. 2 = - 1 + 3 οπότε Α ε ( ε 1 ) . Αν Γ ε (ε 2 ) πρέΓια το περίκεντρο θα πρέπει να λύσουμε το σύ­ στημα των εξισώσεων δύο μεσοκαθέτων. πει 2κ = κ - 1 {::} κ = - 1 και τότε Γ(- 1 ,-2) . Για το ορθόκεντρο θα πρέπει να λύσουμε το b) Το τετράπλευρο ΑΒΓΔ είναι παραλληλόγραμ­ σύστημα των εξισώσεων δύο υψών. μο αν ΑΒ 1 1 ΓΔ και ΒΓ 1 1 ΑΔ οπότε πρέπει λΑΒ = λ rΔ και λ 8r = λΑΔ . c) Για τα μήκη τμημάτων χρησιμοποιούμε τον � Άρα αφού ε 1 1 1 ε 2 , λΑΒ = λrΔ οπότε αρκεί πο απόστασης σημείων. Άρα έχουμε: λ 8r = λ ΑΔ δηλαδή για μ "# - 1, λ "# - 1 πρέπει: (ΒΓ) = J( 5 + 3) 2 + (- 9 - 7) 2 = mσ = gJS μ + 3 + 2 = -λ - 3 {::} μ + λ = -2 . Αν λ = -1 το-' Ομοίως εργαζόμαστε για τα τμήματα που ξέ­ μ+1 λ+1 ρουμε τις συντεταγμένες των άκρων τους. τε η ευθεία ΑΔ έχει εξίσωση χ = - 1 και τότε για Για τα ύψη μπορώ να χρησιμοποιήσω τον τύπο να ισχύει ΒΓ 1 1 ΑΔ πρέπει μ = κ = - 1 αλλά τότε απόστασης σημείου από ευθεία. Άρα η ευθεία ΒΓ είναι πάλι χ = -1 και τα σημεία 2 · 3 + 5 - 11 r; 1 Α,Β,Γ,Δ συνευθειακά. Οπότε δεν ορίζεται τε­ υα = d(Α, ΒΓ) = � = 2-ν 5 κ.τ.λ. 22 + 12 τράπλευρο. 1 c) Για να είναι ρόμβος πρέπει μ+λ = -2 με μ � - 1 d) (ΑΒΓ) = 2 det(ΑΒ, ΑΓ) = .... ή ΑΓ ..L ΒΔ οπότε και - - λ � - 1 - και 1 - · ΒΔ Ο και έτσι έχω λ-μ = 4 οΑΓ ΑΔ = ΟΑΓ (ΑΒΓ) = - (ΒΓ) · (υα ) = .... = 40 τ.μ. 2 πότε λ = 1 και μ = -3 και τότε Α(- 1 ,2), Β(-3,0), e) Για την ακτίνα R του περιγεγραμμένου κύκλου Γ(- 1 ,-2), Δ(1 ,0). έχω: R : (ΟΑ) = (ΟΒ) = (ΟΓ) όπου Ο το περίΓια να είναι ορθογώνιο πρέπει μ+λ = -2 και κεντρο του τριγώνου. Αλλιώς από τύπο γεωμε­ μ � - 1 λ � - 1 και ΑΒ ..L ΒΓ . Οπότε - τρίας λΑΒλΒ r = - 1 ή ΑΒ · ΒΓ = Ο και έτσι έχω (ΑΒΓ) = αβγ {::} R = αβγ μ 2 + 4μ + 3 = Ο {::} μ = -3 ή μ = - 1 που 4R 4(ΑΒΓ) απορρίπτεται. Άρα τότε και λ = 1 και έχω την και όλοι βρίσκονται από προηγούμενα. προηγούμενη περίπτωση και επομένως είναι ρόμβος και ορθογώνιο δηλαδή τετράγωνο. 4) Τέ σσερ α ση με ία του Επιπέδ ου ΘΕΜΑ: Δίνονται τα σημεία: Α(-1 ,2), Β(μ,μ+ 3), d) Για να είναι τραπέζιο αφού ήδη έχω ΑΒ 1 1 ΓΔ αρκεί ΒΓ να τέμνει την ΑΔ δηλαδή μ + λ � - 2 Γ(κ,2κ), Δ(λ,λ-1) και για να είναι και ισοσκελές πρέπει επιπλέον a) Δείξτε ότι τα σημεία Β και Δ κινούνται σε δύο παράλληλες ευθείες ε 1 και ε2 με Α ε ε 1 κατόπιν (ΑΔ) = (ΒΓ) και συνεχίζω με τον τύπο απόστα­ να βρεθεί το κ, αν είναι γνωστό ότι Γ ε ε 2 . σης σημείων ..... b) Ποια είναι η συνθήκη ώστε το τετράπλευρο 5) ΣΥ Μ Μ ΕΤ ΡΙΑ ΑΒΓΔ να είναι παραλληλόγραμμο; ΒΑΣΙΚΟ ΣΧΗΜΑ c) Είναι δυνατό να είναι ρόμβος, ορθογώνιο, τε­ a) Να βρεθεί το συμμετρικό του σημείου Ρ(4,2) τράγωνο; ως προς το Κ(2,-3). d) τι θα πρέπει να ισχύει ώστε να είναι τραπέζιο Απάντηση και τι για ισοσκελές τραπέζιο; Πρέπει Κ μέσο του ΡΡ ' όπου Ρ ' (Χο,ψο) το ζητούμε­ ΥΠΌΔ Ε Ι Ξ Η : νο συμμετρικό. a) Έστω Β(χ,ψ) οπότε χ = μ και ψ = μ+3. 4 + χο 2+ , Ρ'(Ο ,- 8 ) . Άρα 2 = -και - 3 = Ψο οποτε Άρα ψ = χ+3 η ευθεία που κινείται στο Β. 2 2 Παρόμοια ψ = χ-1 η εξίσωση ευθείας που κι.:..._ ._ _

1

,

·

ΕΥΚΛΕΙΔΗΣ Β ' λζ' τ.2/61

=


Μαθηματικά Β' Λυκείου

b.

και λ3 = 04+ 4 -3 . Επομένως -- - 0 3

Να βρεθεί το συμμετρικό του σημείο Ρ(4,2) ως προς την ευθεία ε : 3χ + ψ - 4 Ο =

.

( x-i)

Απ άντ ηση

ε 3 : y = -3 {::} - 3χ + y - 4 = Ο Έστω Κ το ίχνος της καθέτου από το Ρ προς την ε . Ισχύει λρκ λε = - 1 οπότε λpκ = ..!. και Δ) Τα συμμερικά τους ως προς την ψ = χ είναι τα 3 -± _ ο 1 ΡΚ : ψ - 2 = -(χ - 4) {::} χ - 3ψ + 2 = Ο . �(4,0) και Β4 ο , -± και λ4 - -3- - - ..!. . 3 3 0-4 3 Το Κ θα βρεθεί από τη λύση του συστήματος των Επομένως ευθειών ε και ΡΚ: 1 - 4) χ + 3 - 4 = Ο . ε 4 : y = --(χ {::} y 3χ + ψ - 4 = Ο και χ - 3ψ + 2 = Ο 3 και έτσι έχουμε Κ(1 , 1 ). Άρα αν Ρ'(Χο,ψο) το ζη­ ΓΕ Ν ΙΚΕ Υ ΣΗ : Μπορώ με την ίδια λογική να α­ τούμενο συμμετρικό τότε Κ μέσο του ΡΡ ' και ποδείξω ότι η συμμετρική ευθεία της 4+ ε : Αχ + Βψ + Γ = Ο με Α 7: Ο και Β 7: Ο είναι: 1 = 4 + Χο και 1 = Ψο οπότε Ρ ' (-2,0). 2 2 Α) ως προς χ ' χ ε 1 : Αχ - Βψ + Γ = Ο c. Να βρεθούν οι συμμετρικές ευθείες της Β) ως προς ψ ' ψ ε 2 : -Αχ + Βψ + Γ = Ο ευθείας: 2χ + ψ - 8 Ο ως προς Γ) ως προς 0(0,0) ε 3 : -Αχ - Βψ + Γ = Ο Β) ψ'ψ Γ) 0(0,0) Α) χ'χ Δ) ως προς ψ = χ ε 4 : Βχ + Αψ + Γ = Ο

( )

=

Δ) ψ = χ (διχοτόμος lης _ 3ης γωνίας). Απάντ ηση

d. Ποια η συμμετρική της ευθείας ε : 2χ + ψ - 10 Ο ως προς την ζ : 3χ + ψ - 4 = 0 =

Αρκεί κάθε φορά να βρίσκω δύο σημεία της ε, τα συμμετρικά τους ως προς αυτό που μου ζητείται Π ΕΡ Ι Γ ΡΑ Φ Η Τ Η Σ Μ Ε Θ ΟΔΟ Υ και μετά την εξίσωση της ευθείας που ορίζουν αυ­ 1 ΒΗΜΑ: Βρίσκω δύο σημεία της ε, έστω Α(4,2) τά. και Β(5,0). 'Εστω Α(Ο,4) και B , o δύο σημεία της ε 2° ΒΗΜΑ: Βρίσκω τα συμμετρικά τους ως προς την ζ με βάση προηγούμενο παράδειγμα και Α) Τα συμμετρικά τους ως προς χ 'χ είναι τα 4 ο και λ = 0 - (-4) = 3 . είναι Α'(-2,0) και Β ' - ,- 1 1 . Α 1 (0,-4) και Β 1 3, 1 5 -4 - 0 3 3° ΒΗΜΑ: Βρίσκω λ ' = _ .!...!_ . Επομένως ε 1 : y+4=3(χ - Ο) {::} 3χ - y - 4 = Ο . 2 Β) Τα συμμετρικά τους ως προς ψ'ψ είναι τα 4° ΒΗΜΑ: Η εξίσωση της ε ' είναι: 1 1χ + 2ψ + 22 = Ο Az (0,4) και Β 2 - , o και λ2 = 4 = 3 . --0 3 Επομένως ΠΡΟΤΕΙΝΟΜΕΝΕΣ ΑΣΚΗΣΕΙΣ ε 2 : y = 3 · χ - {::} - 3χ + y - 4 = Ο . 1 ) Δίνονται τα σημεία Α(4,0), Β(8,3), Γ(5,7). α) Να δειχθεί ότι σχηματίζουν τρίγωνο. Γ) Τα συμμετρικά τους ως προς την αρχή των β) Να δειχθεί ότι το τρίγωνο ΑΒΓ είναι ορθο­ αξόνων 0(0,0) είναι τα Α3 (0,-4) και γώνιο ισοσκελές. B3 = - , o γ) Ποιο είναι το σύνολο των σημείων Μ(χ,ψ) - ώστε ΑΒ 2 + 2ΑΜ · ΒΜ = 2 · ΜΓ 2 ο

(i )

(� )

( )

(i)

�-

( i)

(i)

ΕΥΚΛΕΙΔΗΣ Β ' λζ' τ.2/62


Μαθη ματικά Β ' Λυκείου

γ) να βρεθεί η απόσταση του σημείου Α(3 , 9) Δίνονται τα σημεία Α(1 ,3), Β(μ,2μ+ 1 ), από την Ε για λ =1 Γ(2,- 1 ), Δ(λ,2λ-5). δ) ποιο το λ ώστε η απόσταση του Β(2, 1) από α) Να δειχθεί ότι τα Β και Δ κινούνται σε πα­ την Ε να είναι 1 . ράλληλες ευθείες ει και ε2 με Α στην ε 1 και Β στην εz . 6) α) Δείξτε ότι η εξίσωση χ 2 + ψ 2 + 2χψ - 3χ - 3ψ + 2 = ο β) Ποια η συνθήκη ώστε το τετράπλευρο ΑΒΓΔ να είναι παραλληλόγραμμο, ρόμ­ παριστάνει δύο παράλληλες ευθείες. βος, ορθογώνιο, τετράγωνο, τραπέζιο και β) Ποια η μεταξύ τους απόσταση; ισοσκελές τραπέζιο. γ) Ποιο το εμβαδόν του τραπεζίου που σχη­ ματίζουν αυτές με τους άξονες χ ' χ και y ' y; 3 ) Σε τρίγωνο ΑΒΓ η πλευρά ΑΒ έχει εξίσωση 2χ - ψ = 1, , ενώ η ΒΓ έχει εξίσωση 7) Έστω η εξίσωση χ - 2ψ + 1 = Ο . Αν η ΑΓ είναι παράλληλη Ε : (μ 2 - μ)χ + (μ 2 - 1)ψ +(μ + 3) = 0 . στην ευθεία 2χ + 1 1ψ + 2004 = Ο και Α(2,3 ) α) Πότε είναι ευθεία; τότε β) Πότε είναι παράλληλη στον χ ' χ. α) Ποια είναι η εξίσωση της ΑΓ. γ) Πότε είναι παράλληλη στον y ' y; β) Ποια τα Β και Γ. δ) Πότε περνάει από την αρχή των αξόνων; γ) Να δείξετε ότι το τρίγωνο ΑΒΓ είναι ισο­ 8) Να βρεθεί η εξίσωση της ευθείας ε η οποία σκελές. είναι παράλληλη στην ευθεία δ) Να βρεθούν οι εξισώσεις των f.Lr και υα. ζ : 3χ - 4ψ + 2003 = Ο και η απόσταση του 4 ) Σε τρίγωνο ΑΒΓ είναι Α(2, 1) και δύο ύψη του σημείου Ρ(2,- 1) από την ε να είναι 3 έχουν εξισώσεις 3χ + ψ - 1 1 = Ο και (ΥΠΌΔΕΙΞΗ: Ορίζω ε : ψ = λχ + β , βρίσκω χ-ψ+3=0. λ, παίρνω τύπο d(Ρ, ε) = ... κ.τ.λ.). α) Ποιο το ορθόκεντρο Η. β) Ποιες οι κορυφές Β και Γ 9) Να βρεθεί η εξίσωση τη ευθείας ε η οποία γ) Ποιες οι εξισώσεις των πλευρών του περνάει από το Μ(Ο, 1 ) και απέχει από το δ) Ποιο το εμβαδόν του. Ρ(2,0) απόσταση 2. (ΥΠΌΔΕΙΞΗ: Ορίζω ε : ψ = λχ + β , βρίσκω 5) Έστω η εξίσωση Ε : (2λ + 1)χ - (2 + λ)ψ + 1 - λ = Ο β σε σχέση με το λ, παίρνω τύπο d(P, ε) = .... α) να δείξετε ότι παριστάνει ευθεία για κάθε κ.τ.λ. λΕR β) να δείξετε ότι διέρχεται από σταθερό σημείο 2)

Η

Συντακτική επιτροπή του Ευκλείδη

Β'

σε συνεργασία με το

Παράρτημα της Δωδεκανήσου εξέδωσε το τεύχος αυτό.

Ευχαριστούμε το Παράρτημα Δωδεκανήσου για την Συνεργασία.

ΕΥΚΛΕΙΔΗΣ Β ' λζ' τ.2/63


Στατιι1τική Εpμηνεία και καται1κεuή 6ιαγpαμμ6των

Συντυχάκης Χαράλαμπος

Ένα από τα βασικά ζητούμενα στη μελέτη της Στατιστικής από τους μαθητές, είναι η «Ερμηνεία» των διαγραμμάτων και η σωστή αξιοποίησή τους στην απάντηση ερωτημάτων. Το διάγραμμα λοιπόν είναι η «lk φετηρία» (και όχι το ζητούμενο) των αντίστοιχων ασκήσεων και η «ανάγνωσή» του προϋποθέτει καλή γν� ση τόσο των ορισμών όσο και των σχέσεων μεταξύ εννοιών της θεωρίας. Παραθέτουμε παρακάτω μερικά προβλήματα Στατιστικής αυτής της μορφής, καθώς επίσης και άλλες lk σκήσεις λυμένες και προτεινόμενες.

ΠΡΟΒ Λ Η ΜΑ 1 Το διπλανό ιστόγραμμα συχνοτήτων περιγράφει το χρόνο (σε min) που χρειάζονται

Οδηγοί 30

οι οδηγοί για να διανύσουν μια απόσταση

20

lO Κm. i. Να εξεταστεί αν το δείγμα είναι ομοιο­

10

γενές. ii. Ένας οδηγός παρατήρησε ότι το πλή­

χρόνος

θος των αυτοκινήτων που προσπέρασε,

t(min)

ισούται με το πλήθος των αυτοκινήτων που τον προσπέρασαν. Ποια είναι η ταχύτητα του οδηγού αυτού;

Λύση

Μεταφέρουμε τα δεδομένα σε πίνακα και συμπλη ρώνουμε με τις απαιτούμενες στήλες. Χ ρ όν ος

(min)

[8, 12) [12, 1 6) [ 1 6, 20) [20, 24) [24, 28) [28, 32) Σύνολο

Υι

Χι

ΧιΥι

ΧΙ2

5 10 20 30 15 10 90

10 14 18 22 26 30

50 140 360 660 390 300 1 900

1 00 1 96 324 484 676 900

ΕΥΚΛΕΙΔΗΣ Β' λζ' τ.2/64

χ� νι

500 1 960 6480 14520 10140 9000 42.600

Νι

5 15 35 65 80 90


Μαθηματικά Γ' Λυκείου

ί.

Για τον υπολογισμό του συντελεστή μεταβολής τυπική απόκλιση s. , -χ = Σ χ ι ν Ι = 1 .900 2 1, 1 mιn. . Ειναι 90 ν

CV,

-

χρειάζεται να βρούμε τη μέση τιμή χ και την

-- ==

-(

]

2 χ νί Επίσης s 2 = .!_ Σ χ 12 ν 1 - ( Σ Ι / = 1 · 42.600 - 1 · 900 == 27 ' 65 ' ν ν 90 90 επομένως s = ..}27, 65 5, 26 min . Ο συντελεστής μεταβολής είναι ==

=

� = 5 ' 26 = 0, 249 χ 2 1, 1

)

δηλ. CV 24, 9% άρα το δείγμα δεν είναι ομοιογενές. Ο χρόνος που χρειάστηκε ο οδηγός για να διανύσει Ni την απόσταση είναι προφανώς η διάμεσος δ της 90 80 κατανομής. Για να βρούμε τη διάμεσο, χρειάζεται να 70 κατασκευάσουμε το ιστόγραμμα αθροιστικών 60 50 40 συχνοτήτων. 30 Τα τρίγωνα Α Β Δ και Γ Δ Ε είναι όμοια οπότε 20 10 ισχύει: χρόνος 10 χ ={::} 20χ = 10(4 - χ) {::} 30χ = 40 {::} χ ::: 1, 33 t(sec) 4 - χ 20 άρα η διάμεσος είναι δ = 2 1 ,33 min. Εφόσον ο οδηγός κάλυψε την απόσταση των 10 Κm σε χρόνο 2 1 ,33 min συμπεραίνουμε ότι η Κm ταχύτητα του είναι υ = 1Ο = 28,5Κm / h περίπου. 0, 35h ==

ίί.

(cv

Δ

Δ

--

ΠΡΟΒ Λ ΗΜ Α 2

Δίνεται το διπλανό ιστόγραμμα σχετικών α­ θροιστικών συχνοτήτων, το οποίο περιγράφει το χρόνο (σε min) που χρειάζονται οι οδηγοί για να διανύσουν μια συγκεκριμένη απόσταση 8 Km. ί. Να βρεθεί η μέση τιμή της ταχύτητας των

οδηγών.

Fi%

1 00 90 80 70 60 50 40 30 20 10

ίί. Να βρεθεί η ταχύτητα που πρέπει να έχει

ένας οδηγός, ώστε να περιλαμβάνει στο 15% των πιο «γρήγορων>>.

χρόνο ς

t(min)

ίίί. Αν το όριο ταχύτητας για τη συγκεκριμένη διαδρομή είναι 50 Κm/h , να βρεθεί η

πιθανότητα να δεχθεί κλήση ένας οδηγός.

Λύση

Αρχικά φτιάχνουμε πίνακα και τον συμπληρώνουμε με τις απαραίτητες στήλες.

ΕΥΚΛΕΙΔΗΣ Β' λζ' τ.2/65


ί.

Από τον τύπο χ = Σχί fi έχουμε ;z = ( 5 · 0, 1 + 7 · 0, 1 + 9 · 0, 1 + 1 1 · 0, 4 + 1 3 · 0, 3) min = 1 0, 4 min 8Κm 46 2Κm / h . ' ταχυτητα ' ' -υ = 8Κm ειναι Άρ α η μεση 1 0, 4 min 0, 1 73h Από το πολύγωνο των σχετικών αθροιστικών Fi% συχνοτήτων βρίσκουμε αρχικά το χρόνο που 1 00 πρέπει να διανύσει ένας οδηγός την απόσταση 90 80 για να ανήκει στο 1 5% των πιο «γρήγορων)). 70 Υπολογίζουμε ότι θα πρέπει να είναι 7 min. 60 50 Οπότε η ταχύτητα θα πρέπει να είναι 40 8Κm = 8Κm ' 30 χ τουλαχιστον -68, 6Κm I h . 7 min 0, 1 1 66h 20 ==

ίί.

==

,

==

_ _ _ _ _ _ _ _ _ _ _

10 8 9.6 1 0

12

χρόνος

14

t(min)

ίίί. Αν η ταχύτητα είναι 50 Κmlh, τότε ο χρόνος στον οποίο θα διανυθεί η απόσταση των 8 Κm είναι _!_ h = 0, 1 6h = 9, 6 min . Επομένως ένας οδηγός θα δεχτεί κλήση: αν καλύψει την απόσταση σε χρόνο . 50 μικρότερο των 9,6 min. Πάλι από το πολύγωνο σχετ. αθρ. συχνοτήτων, παρατηρούμε ότι τα τρίγωνα ΑΒΓ και ΓΔΕ είναι όμοια οπότε: 1 • 6 = χ - 20 {::} 0, 4χ - 8 = 48 - 1, 6χ {::} 2χ = 56 {::} χ = 28 . 0, 4 30 - χ Άρα η πιθανότητα να δεχτεί κλήση ένας οδηγός είναι 28%.

ΠΡΟΒ Λ ΗΜΑ 3

Το διπλανό κυκλικό διάγραμμα παρουσιάζει βαθμολογία ενός τμήματος σε test στο μάθημα της Στατιστικής. ί. Να βρεθεί η μέση τιμή της βαθμολογίας του τμήματος. ίί. Να βρεθεί το ποσοστό των μαθητών που πήραν από 14 έως 18.

iii. Αν

επιλέξουμε έναν μαθητή στην τύχη, να βρεθεί η πιθανότητα να έχει πάρει τουλάχιστον βάση. iv. Να εξεταστεί αν το τμήμα έχει ομοιόμορφη απόδοση. ΕΥΚΛΕΙΔΗΣ Β ' λζ' τ.2/66

[8,12]

1 44 ° [12,16] [4,8]


Μ αθηματικά Γ Λυκείου

Λύση

Χρησιμοποιώντας τον τύπο αι = fι 360° βρίσκουμε τις σχετικές συχνότητες και φnάχνουμε πίνακα. ·

Χι

Β α θμ ο ί

0, 1

2 6 10 14 18

ΓΟ,4) Γ4,8) l8,12) [12,16) Γ16,201

Χ;

4 36 1 00 1 96 324

02 1,2 2,5 5,6 09 10 4

0_,2

0,25 0,4 0 05 1

x�f; 04 72 25 78,4 1 6,2 127 2

2

X;f;

t•;

ί. ίί.

Η μέση τιμή είναι � = Σ χίfί = 10, 4 . Επειδή θεωρούμε όn σε κάθε κλάση οι παρατηρήσεις κατανέμονται ομοιόμορφα, συμπεραίνουμε ότι το ποσοστό των μαθητών που πήραν από 14 έως 1 8 είναι 40% + 5% = 20% + 2, 5% = 22, 5% . 2 2 ' 25% + 40°/ + 5 °/ = 57 , 5 °/ ' • .. Όπως φαινεται απο' την κατανομη' το ποσοστο' αυτο' ειναι ιιι. 2 Άρα η πιθανότητα να έχει περάσει ένας μαθητής τη βάση είναι 57 ,5%. iv. Χρειάζεται να υπολογίσουμε πρώτα την τυπική απόκλιση s. Γνωρίζουμε ότι η διακύμανση είναι Σ χί νί 2 s2 = .!..ν Σ χ� νί - ( ν ) .

--

/ο

/ο

/ο .

Επειδή όμως Σχί νί = χν ο παραπάνω τύπος γράφεται ( νΧ)2 = .!.. χ � - �2 ν = .!.. χ �ν - �2 = χ2 νί - �2 s2 = _!_ Σ \ ν ν Σ χ2 ν. ν Σ ν Σ ν Επομένως καταλήγουμε στον τύπο s2 = Σ x�fi - �2 . Βρίσκουμε s2 = 127, 2 - 1 0, 42 = 127, 2 - 1 08, 1 6 = 19, 04 οπότε η τυπική απόκλιση είναι s = J1 9, 04 4, 36 . Άρα ο συντελεστής μεταβολής είναι CV = � = 4• 36 41, 9% δηλ. το τμήμα δεν έχει ομοιόμορφη χ 10, 4 απόδοση. 1

(

1

ι

·)

·

1

.

1

==

==

Π Ρ Ο ΒΛΗ:\ΙΑ 4

Δίνεται η παρακάτω κατανομή της μισθοδοσίας (σε εκατοντάδες ευρώ) των 80 υπαλλήλων μιας επιχείρησης. Αν είναι γνωστό ότι η διάμεσος είναι 640 €, να βρεθούν οι συχνότητες ν2 και ν3. Λύση

Αρκεί να βρούμε την αθροιστική συχνότητα Ν2, διότι γνω­ ρίζουμε ότι ν2 Ν2 - Νι και ν3 Ν3 - Ν2, όπου οι αθροι­ στικές συχνότητες Νι και Ν3 είναι γνωστές. =

=

ΕΥΚΛΕΙΔΗΣ Β' λζ' τ.2/67

Μισ θός (ε κ ατοντάδες €)

[2 4) Γ4 6) Γ6 8) Γ8 10) [10 12] Σύνολο

Υ πάλληλοι

10 15 5 80

!


Μαθηματικά Γ' Λυκείου V;

Μ ισ θ ό ς

10

Γ2 4) Γ4,6) 16,8) [8, 1 0) [10,12]

Κατασκευάζουμε το ιστόγραμμα αθροιστικών συχνοτήτων.

Ν;

10

Νί

80 70 60 50 40 30 20 10

60 75 80

15 5 80

- - - - - - - - - - - - -

- -

Γ7

1

-

Γ,_.., 20 [ �, 1 ,6 r::::;::;-' .� 4 6 6, 4 8 10

- - - - - - - - - - - -

2

12

Μισθο δ οσία

(xlOO€)

-Ν Από τον ορισμό της διαμέσου, παρατηρούμε ότι: Ο, 4 40 2 9 8 64 - 1, 6Ν 2 9 Ν2 35 . 20 1, 6 Επομένως είναι: ν 2 Ν 2 - Ν1 35 - 1 0 25 υπάλληλοι και ν 3 Ν3 - Ν 2 60 - 35 25 υπάλληλοι. =

=

=

=

=

=

=

=

=

ΠΡΟΒ Λ Η Μ Α S

Σε μαθητικούς αγώνες σφαιροβολίας, η χαμηλότερη επίδοση ήταν 3m και η ψηλότερη 11m. Ομαδοποιήσαμε τα αποτελέσματα σε 4 ισοπλατείς κλάσεις και προέκυψε ότι: Το 12% των μαθητών έριξε μέχρι 5m και το 80% έριξε μέχρι 9m. Αν οι μισές ρίψεις ξεπέρασαν τα 7,Sm, να βρεθεί η τυπική απόκλιση. Λ ύση τ

1 1 4--3 48 2 m ' ' ' των κλασεων ο πλατος θ α ειναι =

Fi %

=

1 00

επομένως οι κλάσεις θα είναι [3,5), [5,7), [7,9) και 90 80 [9, 1 1]. Από τα δεδομένα, προκύπτει ότι Fι = 12%, F3 70 = 80% και F4 = 1 00%. Για να βρούμε την τυπική α­ 60 50 πόκλιση, χρειάζεται να υπολογίσουμε τις σχετικές 40 συχνότητες :t;, ώστε να χρησιμοποιήσουμε τον τύπο 30 2 20 2 s Σχ�fί - � που αποδείξαμε στο πρόβλημα 3. 10 Χρησιμοποιώντας το δεδομένο ότι η διάμεσος δ = m 11 7 ,5m, θα υπολογίσουμε πρώτα τη σχετική αθροιστι­ κή συχνότητα F2 . Από το ιστόγραμμα σχετικών αθροιστικών συχνοτήτων έχουμε: O, S 50 ' F2 9 = 1 5 75 - 1, 5F2 9 F2 40 . 1, 5 30 Κατασκευάζουμε τώρα πίνακα, συμπληρωμένο με τις απαραίτητες στήλες. Βρίσκουμε τις σχετικές συ­ χνότητες από τον τύπο fκ Fk - Fκ ι . =

=

=

=

Ρ ίψεις (m )

Γ3;5) [5,7) [7,9) Γ9, 1 11

Χ;

4 6 8 10

-

F;

12 40 80 100

f;

12 0,28 0,4 0,2 1

ο

ΕΥΚΛΕΙΔΗΣ Β ' λζ' τ.2/68

X;f;

0 48 1 ,68 3,2 2 7,36

χ�

x � f;

16 36 64 1 00

1 92 1 0,08 25,6 20 57,6

I


Μαθηματικά Γ Λυκείου

Η μέση τιμή είναι χ = Σ χ ί fί = 7, 36m και η διακύμανση s 2 = Σ χ;fί - χ 2 = 57, 6 - 7, 362 = 3, 43 . Οπότε η τυπική απόκλιση είναι s = .J3, 43 ::::: 1, 85m . Π ΡΟ Β Λ Η Μ Α 6

Σε ένα δείγμα μεγέθους V = 20, η μέση τιμή είναι

-

χ=

20

25 και ισχύει Σ χ : = 12.820 . Να βρεθεί i=l

πόσο τουλάχιστον θα πρέπει να αυξηθεί η κάθε παρατήρηση, ώστε το δείγμα να γίνει ομοιογενές. Λ ύσ η

Αρχικά θα υπολογίσουμε την τυπική απόκλιση S του δείγματος. Είναι s 2 = .!.. Σ χ; - (Σ

[

5002 ]

ν

1 ι 2.820 1 =(12.820 - 12.500) = 16 . Σ x i = � ν = 500 επομένως s 2 = 20 20 20 ·

χί

ν

)2 , όπου

Άρα η τυπική απόκλιση είναι s = 4. Έστω ότι κάθε παρατήρηση πρέπει να αυξηθεί κατά c δηλ. χ ί = χ ί + c . Τότε η νέα μέση τιμή θα είναι: χ = χ + c = 25 + c ενώ η νέα τυπική απόκλιση s ' = s δηλ. s ' = 4 . Αφού το νέο δείγμα είναι ομοιογενές, s ' < 0, 1 {::} -4 < 0, 1 {::} 2, 5 + O, lc > 4 {::} θα ισχυ' ει: c> 15 . 25 + cχ -

ι

-

Επομένως θα πρέπει η κάθε παρατήρηση να αυξηθεί τουλάχιστον κατά 15. ΠΡΟΒ Λ Η Μ Α 7

Σε ένα διαγώνισμα Μαθηματικών, ομαδοποιήσαμε τα αποτελέσματα σε 5 ισοπλατείς κλάσεις. Να γίνει η κατανομή συχνοτήτων I σχετικών συχνοτήτων και αθροιστικών I σχετικών αθροιστικών συχνοτήτων, αν είναι γνωστό ότι: •

υπάρχουν 2 γραπτά από 16 και πάνω,

η διάμεσος είναί 1 2 ,

η 3η κλάση έχει αθροιστική συχνότητα 10,

η 4η κλάση έχει σχετική συχνότητα 40%

η μέση τιμή είναι 1 1,

Ισχύει ν 3

= ν 1 + ν 2 , όπου νι είναι η συχνότητα της κλάσης. Λύση

20 Το πλάτος είναι - 0 = 4 , οπότε οι κλάσεις είναι [0,4), [4,8), [8,12), [12, 1 6) και [ 16,20]. 5 Αφού η διάμεσος δ = 12, τότε F3 = 50%. Επειδή Ν3 = 10 συμπεραίνουμε ότι το μέγεθος δείγματος είναι 40 20 = 8 μαθητές. ν = ..!.Q_ = 20 μαθητές οπότε ν 4 = �5 1 00 ·

ΕΥΚΛΕΙΔΗΣ Β' λζ' τ.2/69


Μαθηματικά Γ Λυκείου Χ;

Β αθμός

Ν;

2 5 10 18 20

10 15 25 40 10 100

2 3 5 8 2 20

2 6 10 14 18

[0, 4) [4, 8) [8, 12) [12, 1 6) [ 1 6, 20)

f;%

ν;

F;%

10 25 50 90 100

2ν ι + 6ν 2 + 10ν3 + 14 · 8 + 1 8 · 2 _ 1 1 {::} 2ν + 6ν + 1 0(ν + ν ) 220 - 1 12 36 1 2 1 2 20 {::} 12ν1 + 1 6ν 2 72 {::} 3ν 1 + 4ν 2 1 8 (1). Επίσης είναι ν 1 + ν 2 + ν 3 Ν3 οπότε 2ν3 10 δηλ. ν3 = 5. Άρα ν 1 + ν 2 5 (2). Από το σύστημα ( 1 ) και (2) β ρίσκουμε Vι = 2 και Vz = 3.

, Ειναι

-

-

=

=

=

=

=

Π ροτεινόμεν α Προβλή ματα 1.

2.

3.

ντίστοιχες αθ ροιστικές συχνότητες του Fi%. Να β ρείτε ί. τη διάμεσο δ ίί. τη μέση τιμή χ

Σε ένα υπουργείο εργάζονται 1 50 υπάλληλοι, από τους οποίους οι 90 είναι άντρες με μέση τιμή ηλικίας 30 έτη και συντελεστή μεταβο­ λής 10%. Αντίστοιχα για τις γυναίκες, η μέση τιμή είναι 25 έτη και ο συντελεστής μεταβολής 20%. Να β ρεθεί ο συντελεστής μεταβολής όλων των εργαζομένων στο υπουργείο. Σε μια ομαδοποιημένη κατανομή, με ισοπλα­ τείς κλάσεις, το πολύγωνο συχνοτήτων έχει κο ρυφές με συντεταγμένες (6, 10), (1 0, 12), (14, 1 5), (1 8,20) και (22, 16). ί. Να β ρεθούν οι κλάσεις και να γίνει ο πίνα­ κας συχνοτήτων - σχετικών συχνοτήτων. ίί. Να υπολογίσετε το εμβαδόν που σχηματί­ ζεται μεταξύ του πολυγώνου συχνοτήτων και του ο ριζόντιου άξονα. ίίί. Να υπολογίσετε τη διάμεσο. Δίνεται ο διπλανός πίνακας ομαδοποιημένων παρατη ρήσεων μιας μεταβλητής Χ και οι α-

4.

Κλά σ εις [ - , )-

Fi%

1-5

7

5-9

42

9 - 13

82

13 - 17

97

17 - 21

1 00

Σε ένα δείγμα μεγέθους ν = 30, η μέση τιμή 20 είναι � 20 και ισχύει Σ χ � 1 1 .220 . Να =

i=l

=

β ρεθεί πόσο τουλάχιστον θα πρέπει να αυξη­ θεί η κάθε παρατή ρηση, ώστε το δείγμα να εί­ ναι ομοιογενές.

ΕΥΚΛΕΙΔΗΣ Β' λζ' τ.2ΠΟ


�� �

Αιτκήσεις ατα όpια

Καραβόλιας Μιχάλης, Μπου ρνής Απόστολος, Σείτης Βασίλης, Λιγνός Ηλίας

Α σ κή σ εις στα όρια

Αν lim[(f(x) - 3x2 + χ - 4] = 19 και ισχύει Χ--+1 ότι f(x) + 1979 = f(1 - χ) για κάθε χ ε R να

1.

·

(�) · συν-α

πu

υπολογίσετε τα όρια: α) lim f(x) Χ--+1

αν είναι

β) limf(x) . Χ--+0 Λύση

f(x) - 3χ 2 + χ - 4 = g(x) τότε 2 f(x) = g(x) + 3χ - χ + 4 και limg(x) = 1 9 χ->1 2 άρα limf(x) = lim(g(x) + 3χ - χ + 4)

α) Έστω

χ -> 1

=

χ -> 1

= 19 + 3 - 1 + 4 = 25 . β) 1im f(χ)(θέτω χ = 1 - u) = limf(l - u) = χ->0 u->1 1 979] = 1imf(u) + 1 979 = 1im[f(u) + u->1

u->1

(από (α)) = 25 + 1079 = 2004. 2.

πχ Να βρεθεί το ΙiΠJ (2χ - α) · εφ . α χ-+2 Λύση

Θέτω χ - � = u άρα 2χ - α = 2u και u Ο α2 α φου, χ ---+ - . 2 Επίσης χ - = u � · χ = u� ---+

-�)

�(

π �πχ = -π + πu -π x - -π = -u . α 2 α α 2 α Οπότε το όριο γίνεται lim 2u . εφ u_,o

[

πu

α

U->0

= lim -2u - -1- - συν πu πu u->0 α ημ α

ΠU ημ-

α

=

α

έχουμε:

=

---+

Ο αφού u

---+

ι.

Ο οπότε

1 · συνt = -2α · -1 2α . -2 α · lιm · 1 = -. t->0 π π π ημt t Να βρεθεί το όριο: 6"!

3.

Χ-+6

ημ6χ

-ν� 7χ - π -

� Π

-

6

Λύ ση

μορφή = �� .J7χ - π - �( ( % )) 6

. = lιm

π Χ -> -

[

�]

�----"""π

-

7χ - π - 6 'ΠΙ: 6χ. li": � + �Γπ� = Χ-> Χ-> 6 7 χ-6 6 6

6

ημ6χ -/Ίχ - π +

-

Γ

(θέτω χ -

(�2 + α )] =

[ ] συν α lim -2u · σφ = lim -2u

U->0

·

πu

πu

πu

α t τότε t

ημ6χ

]

[

πu

-2 · α 1 = ιιm . u->Ο π ημ

�6

1.

=

ι

J

u � 6χ = π + 6u και u ---t Ο αφού

x ---t -π ) 6 Γπ = = ..!_ lim ημ(π + 6u) · 7π - π + U 7 u->O 6 �� = _ ..!_ lim ημ6u · 2 · � = �� 7 u->0 U = _ ..!_ lim 6 ημ6u · 2 � = 7 u->O 6u �� (θέτω 6u = t άρα t ---t Ο αφού u ---t Ο)

ΕΥΚΛΕΙΔΗΣ Β ' λζ' τ.2/71

[�

]


Μαθηματικά Γ' Λυκείου

Λύση

Έστω f 2 (x) - 2κf(χ) = g (x) οπότε θα είναι και lim g (x) = -κ2 άρα

. f(χ) + λχ Αν lιm = α (1) και χ-1 . (λ2 - λ)f(χ) + 4 Iιm = β (2 ), οπου α, β Ε R f(x) + λχ

�.

Χ --> 1

.

>--> 1

τότε να βρείτε . �3 + f(x) - 1 . .. το lιm χ-1

ί. το λ Ε R .

ιι.

. .... 1

Λύση

ί. Από την ( I ) αφού το ιim(x - 1) = Ο και α Ε R Χ -> 1

πρέπει και +λ=Ο + λχ] = Ο {::} ιimf(x) ιim[f(x) Χ -> } Χ -> 1 {::} limf(x) = -λ (α). Χ -> 1

f 2 (χ) - 2κf(χ) + κ2 = g (χ) + κ2 ή [f (x) - κ]2 = g (x) + κ2 ή lim [ f(x) - κ]2 = Χlim [g (x) + κ2 ] ή Χ ----+ Χ0 ----+ Χ0 2 ιim (f (χ) - κ) = Χιim g (χ) + κ2 ή Χ --+ Χ 0 --+ Χ 0

[ ] [ ιim (f(x) - κ)]2 = -κ2 + κ2 = Ο ή Χ --+ Χ 0

ιim (f(x) - κ) = Ο ή ιim f(x) = κ . 3χ 2 συνχ - χημ7 Να βρεθεί το όριο lim 2 Χ -1

7.

Χ-->-1«>

Από την (2) αφού ιim ( f(x) + λχ) = Ο (από (α)) Χ -> 1 Λ ύση : και β Ε R πρέπει και Το πεδίο ορισμού της συνάρτησης είναι lim[(λ2 - λ)f(χ) + 4] = Ο {::} Χ ->1 A r = (- oo , - 1) U (- 1, 1) U (l, + oo ) . (λ2 - λ) lim f(x) + 4 = Ο {::} Οπότε για κάθε χ Ε (1, +οο) χ -->1 3χ 3χ 2συνχ - χημ 7 2συνχ - χημ 7 λ :;: Ο, λ :;: 1 ) (β) ιim f(x) = --4 ( < Χ-->1 l f < x) ι = λ2 - λ = 2 -1 χ χ2 - 1 Από (α) και (β) πρέπει 4 -λ = - -{::} λ3 - λ2 - 4 = Ο {::} (Homer) l2συνχ Ι + χημ λ2 - λ < = {::} (λ - 2)(λ2 + λ + 2) = ο χ2 - 1 1 Ι {::} λ = 2, (λ2 + λ + 2 = Ο αδύνατη το R). <αφού Ι αβ l � l αl + Ι β l ) ίί.Αφού λ=2 έχουμε ιimf(x) = -2. Επομένως 2lσυνχl + χ η μ 3x 3+f(x)+1 3+ -1 f(x) � � )( ) { . �3 +f(x) -1 = � . 7 < 2.1+χ·1 � - χ2 - 1 Χ-1 χ2 - 1 ( x-1) ( �3+f(x) +1) f(x)+2-2x+2 · --��==�� f(x)+2 (αφού ι συνχl � 1 και η μ 3χ � 1 ) = \+ 2 ιιm = ιι. m 7 χ -1 ι ( x-1) (�3+f(x) +1 ) ..... ι ( x-1) (�3+f(x) +1) 2 2 2 + + χ χ :::; f(x) :::; χ + άρα lf(x)l :::; δηλ. f(x)+2x -2(χ-2) χ2 - 1 χ2 - 1 χ2 - 1 = ιim = lim ι 2 χ + = ιι· m -χ = ιι· m -1 = ο δηλ. ( x-1) (�3+f(x) +1 ) ( x-1) (�3+f(x) +1) ομως ιι· m -χ - +οο Χ χ-> +οο Χ 2 χ -> +οο Χ 2 - 1 α α-2 =--1 = (από το κριτήριο παρεμβολής είναι και 2 2

ι

I

l l

χ --> 1

.....

..... ι

I

---':-----:---'-

Η)

Hl

-τ Ι

I

ι

.....

ι ι

·

--

lim

Χ ---+ + 00

5.

Αν lim [f 2 (x) - 2κf(x)) = -κ \ Χ � ΧΟ

βρείτε το lim f(x) . Χ � Χο

KER.

Να

8.

f(x) = O ·

Δίνεται η συνάρτηση f( χ) -

ΕΥΚΛΕΙΔΗΣ Β' λζ' τ.2/72

αχ 2 + βχ + γ , χ * ι και α, β, γ Ε R για χ-1


Μ αθηματικά Γ Λυκείου

Ο συντελεστής διεύθυνσης της ευθείας ΑΒ είναι α - β = 2β + 1 - β = ι . Άρα η ΑΒ σχηματίζει το σημείο Μ(β,γ) ανήκει στην ευθεία λ = β+ι β+1 ε : χ + 2y + 3 = Ο με τον άξονα χ'χ γωνία ω με εφω = ι δηλ. Λύση Η ΑΒ έχει εξίσωση ω = π/ 4 . Είναι αχ 2 + βχ + γ = (χ - 1) · f(x), χ � 1 . Άρα y - β = ι( χ + ι) =? y = χ + β + ι και δεν διέρχεται 2 ιim(αχ + βχ + γ) = ιim[(χ - 1)f(χ)] ή από την αρχή των αξόνων αφού β � - ι . Άρα η χ --> 1 χ --> 1 ΑΒ σχηματίζει με τους άξονες ορθογώνιο και ισο­ α + β + γ = Ο ή γ = -α - β (ι). Οπότε είναι σκελές τρίγωνο. αχ 2 + βχ - α - β = f(x) = χ-1 10. Να βρείτε το όριο του πηλίκου ( 2 ) ) ( χ - 1} [α( χ + 1) + β ] _- α χ - 1 + β ( χ - 1 -_ -_ χ-1 χ-1 ι· ημ ( ημ(ημ(ημχ)) ) = αχ + α + β. . .•!!: ημ ( ημ(ημχ) ) Από υπόθεση είναι ιim f(x) = 3 {::} 2α + β = 3 ή χ _. ] Λύση β = 3 - 2α (2). Η ( 1) από (2) γίνεται γ = α-3 (3). Αν ψ = ημχ, z = ημψ και ω = ημz τότε επειδή Το σημείο Μ(β , γ) ανήκει στην ε, αν και μόνο αν οι χ ---t Ο το z Ο και ω Ο, ψ ---t Ο . συντεταγμένες του ικανοποιούν την εξίσωση Διαδοχικά έχουμε: χ + 2y + 3 = Ο . Έχουμε ημ ημ(ημ(ημχ)) ) ημ ημ(ημψ) ) από (2)( 3) ιim ( = ιim ( = 3 - 2α + 2( α - 3) + 3 = Ο . Άρα β + 2γ + 3 ψ-->0 χ -->0 η μ ( ημ(ημχ) ) ημ(ημψ) το Μ ανήκει στην ε. ιim ημ(ημz) = lim ημω = 1 (Εξ ορισμού). Ζ -->0 ημz ω-->0 ω 9. Δίνονται τα σημεία Α(-l,β) και Β(β,α), την οποία ισχύει lim f(x) = 3 . Αποδείξτε ότι Χ --+ 1

---t

α, β ε R* , β :;t: 1 ημ(αχ) + χ Αν ισχύει lim = 2 , να δείξετε ότι χ--+ 0 ημ(βχ) + χ -

.

η ευθεία που διέρχεται από τα σημεία Α και Β, σχηματίζει με τους άξονες ισοσκελές τρίγωνο.

Α σ κή ση ς για λύση

1.

)

(

χ ->+οο

ημαχ + χ αχ · --=�. ημ(αχ) + χ = ιιm αχ ιιm η μ χ - ο ημ(βχ) + χ χ-ο βχ _β_χ + χ βχ χ α ημαχ + ι . αχ = ι� = χ ημβχ χ β -- + 1 βχ

) )

Έστω f συνάρτηση για την οποία ισχύει ιim f(x) + 2χ = 2 . Να δείξετε ότι : χ -->+οο 3χ - ι α) ιim f(x) = 4

Λύση

Έχουμε

( [

---t

2.

Χ

Δίνεται η συνάρτηση f : R ισχύει ιim f(x) = +οο .

---t

R για την οποία

χ -> +οο

α + 1 αρα , -α + 1 = 2 {:} α = 2β + 1 (1). = -β+1 β+1

Να βρεθούν τα όρια . 2f 2 (x) + 3f(x) - 2 α) ιιm χ ->+ οο f 2 (χ) + ι β) ιϊm �f 2 (x) + 2f(x) - f(x) . χ -> +οο

ΕΥΚΛΕΙΔΗΣ Β' λζ' τ.2Π3

[

]


I Δι, α φοpιιι6ς Λογι�μός Απόστολος Μπουρνής Στο υπέροχο βιβλίο «Μεγάλες στιγμές των Μαθηματικών» Howard EVES, μπορεί κανείς να διαβάσει τα εξής : Η επινόηση νέων μαθηματικών μεθόδων στηρίζεται σχεδόν αποκλειστικά στην ύπαρξη προβλημάτων

που οι λύσεις τους δεν έχουν επιτευχθεί. Το πρόβλημα της ύπαρξης και σχεδίασης εφαπτομένων σε καμπύ­ λες καθώς και το πρόβλημα των μεγίστων και ελαχίστων οδήγησε τους μαθηματικούς στην δημιουργία του διαφορικού λογισμού. Πολλές αξιόλογες προσπάθειες έγιναν από Torrίcellί, Descartes , Kepler, Fermat, Vίet, Βαrrοw αλλά η πιο σημαντική ήταν του Leίbnίtz. Ο συμβολισμός και πολλοί γενικού κανόνες διαφόρισης τους οποίους χρησιμοποιούμε σήμερα δόθηκαν από τον Leίbnίtz.

Α. Υπολογι σμός παραγώγου με την βοήθεια του

2.

Έστω f συνάρτηση f : R � R τέτοια ώστε λ (χ - l)κ + α :s:; f(x) :s:; (x - l) + α (1)

. f(x 0 Iιm

Χ-+ 0

α ε � κ, λ ε Νκ,λ :<:: 2 . Να βρεθεί f '(l) . Η ( 1 ) για χ =

Λύση

Ι γίνεται: α � f(l) � α . Άρα f( I ) = α.

Έχουμε: (l){=> (x - Ι) κ � f(x) - α � (χ - Ι) λ :::;. =? (χ - Ι) κ � f(x) - f(I ) � (χ - Ι)λ (2) Διακρίνουμε τις περιπτώσεις: α.

Χο

ε IR

·

Να δείξετε ότι:

ορι σμού .

1.

Έστω f παραγωγίσιμη στο

Αν χ> Ι τότε

Χ ->)+

Χ ->)+

. Άρα ιιm f(x) - f(l) = 0 Χ->)+ Χ-Ι

β. Αν χ< Ι τότε

(3) .

�(Ι) �

(χ - Ι) κ- 1 � f(x) (χ - l) λ- 1 . χ ο · f(x) - f(l) = ο (4) . ποτε ιιm '

Χ ->)+

Χ

-1

Από τις (3) και (4) :::;. ιim f(x) - f( I ) = O � f ' (l) = O Χ ->)+ Χ-Ι

α ε !R*

Λύση

Είναι : f(x 0 + αh) - f(x 0 - αh) h f(x 0 + αh) - f(x 0 ) - [f(x 0 - αh) - f(x 0 )] h f(x + αh) - f(x 0 ) f(x αh) f(x ) = 0 · α + 0 - - 0 · α ( Ι). (-αh) α·h _ -

(2) *? (χ - Ι) κ- 1 � f(x) (Ι) � (χ - Ι) λ-1 . χΙ Είναι Iim (χ - Ι) κ- = Iim (χ - Ι/- 1 = Ο .

+ αh) - f(x0 - αh ) _ 2 - αf ' ( χ0 ) h

_

-

' + Ειναι : κιι· m f(x o αh) - f(x o ) = -ο αh li m f(x o + κ) - f(x o ) = f ' (x 0 ) (Θέσαμε ah = κ) Κ Κ --> 0 f(x - αh) - f(x 0 ) και κιι. m 0 = -ο -αh f(x o + κ) - f(χ ο ) ' (χ = ιi m = f . 0 ) (Θέσαμε -αh=κ) 0 --> Κ Κ Οπότε παίρνοντας τα όρια στην σχέση ( 1 ) θα έ­ χουμε: ιim f(x 0 + αh) - f(x 0 - αh) = Κ--> 0 h = αf ' (χ 0 ) + αf ' (χ 0 ) = 2αf ' (χ 0 ) =

ΕΥΚΛΕΙΔΗΣ Β ' λζ' τ.2Π4


Μαθηματικά Γ Λυκείου

Β. Ε ύ ρε ση 1.

Παραγώ γ ων διαφόρων τά ξεων

Έστω f δύο φορές παραγωγίσιμη στο R ώστε f(x3 ) = χ 13 ( l ).Να βρεθούν (i) f' (α) ii) f"(α) α>Ο Λύση

Παραγωγίζοντας και τα δύο μέλη της (1). Βρί­ σκουμε: 1 3 · x 1 0 (2). f'(x . 3 ) · (x 3 )' = 1 3 · x 1 2 =? f1(x 3 ) = 3 Στην (2) θέτω όπου χ το � οπότε

Στο Ο. Για χ>Ο lim f(x) - f( O) = χ-.ο+ χ =

[ ]

lim x; -1 =

χ-.ο+

j�

:: �::

+ οο αν μ < ν Για χ<Ο lim f(x) - f(O) χ-.οχ μ �- 1 � (-χ) m = li -- = - lim (-χ) ν χ-.οχ-.οχ =

1

0 w μ>ν 1 αν μ = ν -οο αν μ < ν Οπότε ο f'(O) υπάρχει όταν μ � ν . =

3. Έστω f : R + � R με f(x) = x 2 Rnx . f'(α) = � <[σ!Ο = � α3 � 3 3 i) Να βρεθούν f ' ,f",f"' την βρίσκουμε Παραγωγίζοντας (2) ii) Να δείξετε ότι για κάθε ν ε Ν ν � 3 1 30 130 f " (x 3 ) · 3x2 = · χ 9 =? f " (x 3 ) = χ 7 (3). ισχύει: 9 3 •- • ! Στην (3) θέτω όπου χ το � οπότε f (x)<•> = 2(-t) . ν - 3 ) (l). χ 130 1 30 f " (α) = . ρ- = · α2 � . 9 9 Λύση

��

2.

Γενικό θέμα. Να βρεθεί η παράγωγος · f(x) = � ν,μ ε Ν ν � 2 . Λύση

(i) Περίπτωση Αν μ περιττός τότε Dr = [Ο, +οο) .

[]

I

Για κάθε χ>Ο είναι f'(x) = χ � = χ � �

xv Στο , ιm f(x) - f(O) = 1.ιm - 0 = χ--.0 χ χ ο Ι. χ--.0

[ ]

= !� χ � - ' =

��

:: �::

ι

(i) Είναι f'(x) = 2xRnx + χ = x(2Rnx + 1) 2 f " (x) = 2Rnx + 1 + χ · - = 2Rnx + 3 χ f'" (x) = � χ (ii) Για ν 3 ο τύπος (1) δίνει 1 2(- 1) 3- · (3 - 3) ! = -2 αλη θης ' f "'( χ ) = χ χ 3-2 Υποθέτουμε ότι είναι αληθής για ν κ δηλαδή κ-1 f(χ) (κ ) = 2( - l )Χκ (κ 3) ! -2 Θα δείξουμε ότι είναι αληθής για ν κ+ 1 δηλαδή κ f(χ) (κ+1) == 2(- l )Χκ(κ1 - 2) ! Είναι: ' f(χ) <κ+ 1) = ( f(χ) κ ) ' = 2(- Ι) κ- 1 (κ - 3) ! κι z = 2(- ι γ- 1 · (κ - 3) ! (- l )Χκ· (κ1 - 2) = κ 2(- l) · (κ - 2) ! Χ κ-1 αφού (κ - 3) !· (κ - 2) = (κ - 2) ! =

=

+οο αν μ < ν (ii) Περίπτωση. Αν μ άρτιος τότε Dr = R οπότε η συνάρτηση f θα γράφεται ; αν χ -> Ο v r-::μ = l x l�μ = x Οπότε f(χ) = �χ � (-χ) ν αν χ < Ο � -μ χ ν - 1 χ>Ο ν f'(x) = � -μ (-x) V -1 · (- 1) χ < Ο ν 4. Έστω f, ν φορές παραγωγίσιμη να βρεθεί η ν-στή παράγωγος της g( χ ) = χ · f ( χ )

j

ι )

ΕΥΚΛΕΙΔΗΣ Β' λζ' τ.2Π5


Μαθηματικά Γ Λυκείου

= ν (1 + x2)f1(x) - xf(x) + νχ f(x) = �1 + χ2 �1 + χ2 = ν(l + x z ) . f Ι (χ) = ν(1 + x z ) . f(x) = f(x) �1 + χ2 �1 + χ2 ν�1 + χ2

Λύση

Για ν = 1 . g1(x) = xf1(x) + f(x) . Για ν = 2 g 11 (x) = xf"(x) + 2f1(x) και για ν = 3 g 111 (x) = xf111(x) + 2f11(x) . Πιθανή μορφή της g(x)(v) είναι

g(x)(v) = χ · f(x)(v) + ν · f(x)(v-l) (1) . Είναι φανερό ότι η ( 1) αληθής για ν = 1 ,2,3 . Υπ<r

Γ. Στα πο λυ ώνυμα

1.

Ρ (χ)=α ν χν +α ν_ ι Χ ν-t + .... +α ι χ+α ο

θέτουμε ότι είναι αληθής ν = κ δηλαδή g(χ)(κ) = χf(χ)(κ) + κf(χ)(κ- Ι ) . Θα δείξουμε ότι είναι αληθής για ν κ+ 1 δηλαδή

και ρ ι ,ρ2 , ... ,ρν ν διακεκριμένες ρίζες του να δείξετε ότι: 1- _ 11_ (i) Ρ ' (χ) = + + ... + Χ - ρν Ρ(χ) Χ - ρl Χ - ρ2

g(x/κ+l) = χf(χ)(κ+Ι) + (κ + 1)f(χ)(κ) .

Είναι

g(χ)(κ+ Ι ) = [ g(χ)(κ) ]1 = [ χf(χ)(κ) + κf(χ)(κ- 1 ) ] 1 = = xf(x)(κ+l) + f(χ)(κ) + κf(χ)(κ) = = χf(χ)<κ+ Ι ) + (κ + 1)f(χ)(κ)

άρα αληθής για κάθε ν Ε Ν* . 5.

'Εστω f(x) = �x + .Jx 2 + 1 να δείξετε ότι ν 2 (1 + x 2 )f"(x) + ν 2 χf ' (χ) = f(x) . Λύση

(

Είναι f1(x) = χ + �χ2 + 1

�(

)

� x + Jx' + l ��

(

ι

I

Υ

Ι =

Άρα

Λ ύ ση

(i) Είναι γνωστό ότι

Ρ( χ) = αν (χ - ρι (χ - ρ z ) .... (χ - ρν ) (1) . Παραγωγίζω την ( 1) οπότε: Ρ1( Χ ) = αν (χ - ρ 2 ) ... (χ - ρν ) + + αv (χ - ρι ) · (χ - ρ 3 ) ... (χ - ρ v ) + ... + αν (χ - ρι )( Χ - ρ z ) ... (χ - ρν_ ι ) = = Ρ1(χ) = -1- + _1_ + ... + -1- (2) Ρ(χ) χ - ρ 1 χ - ρ z χ - ρν (ii) Παραγωγίζουμε και τα δύο μέλη της (2) οπότε: Ρ1(χ) 1 + 1 + ... + 1 < {::} Ρ(χ) (χ-ρ1 ) 2 (χ-ρ 2 ) 2 (χ-ρν ) 2 ο Ρ"(χ)Ρ(χ) - Ρ1(χ)Ρ1(χ) < Ο {::} {::} p2 (x) {::} Ρ"(χ)Ρ(χ) < [ Ρ1(χ)]2 I

--

)

+χ 1 = f(x) = ! χ + �χ2 + 1 � - . νΥ+1 ν ν�1 + χ2 �χ2 + 1 f1(x)�1 + x2 - f (x) · x JI+;! 1 1 +'-χ2-και f" (χ) = - · -------::---'--ν 1 + χ2 (χ) -:= xf(x) __;____;_i:::=:-'--'= = - (1 + x2)f1 ν (1 + χ 2 )�1 + χ 2 .

(ii) Ρ"(χ)Ρ(χ) < [ Ρ 1 ( χ)] 2

[ ]

+ + i+J] =

) (

Έστω i,ίΧ) το πολυώνυμο

_;__

2.

Θεωρούμε Χ

το

Χ2

Χ

ν

πολυώνυμο

Ρ(χ) = 1- + - + .... + - Να δείξετε ότι το 1! 2! ν! Ρ(χ) μπορεί να έχει μόνο απλές ρίζες.

_

ν2 (1 + x2)f"(x) + ν2χf1(χ) = f(x)'= ::Ξ:::: == ν 2 (1 +χ 2) (1 + x2)f1(x). - xf(x) + ν 2 · Χ -;::: ν�1 + χ2 ν(1 + χ2)�1 + χ2

=-

Λύση

Έστω ότι το Ρ(χ) έχει τον αριθμό ρ τουλάχιστον διπλή τότε προφανώς ρ Ο και το ρ είναι ρίζα του �

Pl(x) . Είναι

ΕΥΚΛΕΙΔΗΣ Β ' λζ' τ.2/76


Μαθηματικά Γ Λυκείου

--

Χ2 Χ ν-1 Ρ ' (χ) = 1 + χ + - + ... + 2! (ν - 1) ! χν χ 2 ... χ ν- 1 χ ν =1+χ++-- = + + 2! (ν - 1) ! ν ! ν ! Χν = Ρ(χ) - ­ ν! Οπότε ν ν Ρ ' (ρ) = Ρ(ρ) _ Ε._ ::::} ο = o - L {::} ρ ν = ο {::} ρ = ο ν ν. άτοπο.

Λύση

α) Είναι f ' (x) = 2χ · ημχ + (χ 2 - 4)συνχ και f ' (Ο) = -4 Ο . β) Για την f ισχύει το Θ.Μ.Τ. άρα υπάρχει ξ ε (-2, 2) ώστε f(2) - f(-2) f ' ( ξ) = {::} 2+2 0 0 {::} {::} 2ξημξ + ( ξ 2 - 4)συνξ = +=

{::} 2 ξημξ = (4 - ξ2 )συνξ {::} 2ξ = συνξ σφξ = 2ξ {::} {::} 2 ημξ (2 - ξ)(2 + ξ ) 4-ξ 1 1 - --ξ {::} σφξ = 2-ξ 2+ γιατί: ξ + 2 + ξ -:- 2 2ξ (2 - ξ)(2 + ξ) (2 - ξ)(2 + ξ) ξ+2 1 1 2-ξ ξ) 2 ξ 2+ξ (2 - ξ)(2 + ξ) (2 - ξ)(2 +

--

Δ) Θ.Μ.Τ.

1)

Δίνεται η συνάρτηση f(x) = (n(συνχ), χ ε

( - i ;) ,

__....ο._ ..._ = -=------=--

α) Αποδείξτε ότι f ' (x) = -εφχ για κάθε χε

(

-�

'

2 2

)

β) Για κάθε α, β ε

(- ; , ;) με α<β υπάρχει

3)

τουλάχιστον ένας ξ ε (α, β) για το οποίο συνβ < α -β )εφξ =e . ισχύει συνα

---

Να αποδείξετε ότι δεν υπάρχει συνάρτηση f συνεχής στο [0,2] παραγωγίσιμη (0,2) με 1 :ς f(Ο) :ς 2, 4 :ς f(2) :ς 9 και s :ς f ' (x) :ς S , για κάθε χ ε (0, 2) . Λύ ση

Λύ ση

Έστω ότι υπάρχει τέτοια συνάρτηση f. Από το 1 ' ' f α) (χ) = [fη(συνχ)] = · (συνχ) ' = Θ.Μ.τ. έχουμε ότι υπάρχει τουλάχιστον ένας συνχ ξ ε (Ο, 2) ώστε f ' (ξ) = f(2) - f(O) ( 1 ). Ο μως = - ημχ = -εφχ . 2-0 συνχ 2 ή -2 ::; -f(O) ::; - 1 9 και 1 4 f(O) f(2) ::; ::; ::; ::; β) Για την f(x) ισχύει τ? Θ.Μ.Τ. για χ ε [α, β] ' άρα υπάρχει ένας τουλάχιστον ξ ε (α, β) ώστε και προσθέτοντας κατά μέλη έχω f 2 -< f(2) - f(O) -< 8 {::} � -< f(2) - (O) -< �� f ' (ξ ) = fn(συνβ) - fn(συνα) {::} 2 2 2 β-α ' {::} 1 ::; f (ξ) ::; 4 συνβ συνβ e<α- β ) εφξ . . = {::} {::} -εφξ (β - α) = fη άτοπο αφού 5 ::; f ' (ξ) ::; 8 . Άρα δεν υπάρχει τέτοια συνα. συνα συνάρτηση f. 2

--

2)

--

( )

Δίνεται η συνάρτηση f(x) = (χ - 4)ημχ . Να αποδείξετε ότι: α) η εξίσωση f '(x) = Ο δεν έχει ως ρίζα τον αριθμό ξ = Ο. β) με τη βοήθεια της f, υπάρχει ξ ε (-2, 2) 1 1 , σφ ξ = -- - -- . ωστε: 2-ξ 2+ξ

4)

Αποδείξτε ότι: α) l ημχ + συνχ l :ς J2 , για κάθε χ ε IR . β) Αν f είναι μια συνάρτηση παραγωγίσιμη στο R, με f ' (χ) = ημχ + συνχ , για κάθε χ ε IR , για όλα τα α, β ε IR ισχύει ι r (α) - f(β) Ι :ς J2 · Ι α - β ι .

ΕΥΚΛΕΙΔΗΣ Β ' λζ' τ.2/77


Μαθηματικά Γ Λυκείου

{::} xe x f'(x) = 1 - xRnx {::}

Λύση

α) Έστω l ημχ + συνχ l :::; J2 {::} -1 - Rnx 1 xRnx {::} f'( χ ) = {::} f'( χ ) = χ x {::} {::} Ο :::; (ημχ + συνχ)2 :::; 2 {::} xe x e {::} ημ 2 χ + συν2χ + 2ημχσυνχ :::; 2 {::} 1 e - - e · Rnx {::} ο :::; 1 + ημ2χ :::; 2 {::} - 1 :::; ημ2χ :::; 1 ' {:} f1(x) = χ x {::} (e )2 {::} l η μ2χ l :::; 1, ισχύει. β) Η f παραγωγίσιμη στο R, άρα και συνεχής, Rnx Rnx ' + c, c E JR {:} f1(x) = {:} f(x) = x οπότε ισχύει το Θ.Μ.Τ. για την f στο [α,β] δηλ. e ex Αλλά το σημείο υπάρχει ξ Ε (α, β) ώστε f'(ξ) = f(β) - f(α) β-α (1, 0) Ε Cf {::} f(1) = 0 {::} .... {::} C = 0 . lf - f α>Ι {::} Ι ημξ + συνξ l = < > :::; J2 {::} Άρα f(χ) = Rnx , χ>Ο. ex lβ-α {::} l f <β > - f(α>Ι :::; J2 · Ι β - αΙ . 3) Να βρεθεί συνάρτηση f, για την οποία 2 Αν α β προφανής. ισχύει: f"(x) χ > 3 και η --'-'-----

χ

χ

( )

r �

=

=

Δ ) Συ νέπειε ς του Θ.Μ.Τ.

Ι)

Να βρεθεί συνάρτηση f παραγωγίσιμη στο R για την οποία ισχύει: f'(x) ημχ + συνχ , για κάθε χ ε JR και ex

--= r( ;) = ι + eπ' 2 •

(χ - 3)

2,

=

εφαπτομένη της Cr στο σημείο της Α(4,ι6) είναι παράλληλη της ευθείας ε 1 : y 6χ - 5 . Λύση

f σαν δύο φορές παραγωγίσιμη στο (3, +οο) εί­ ναι παραγωγίσιμη στο (3, +οο) άρα και συνεχής στο διάστημα αυτό. ' 2 Λ ύ ση Αφού f " (x) = {:} f 11 (x) = 2 -1- άχ-3 (χ - 3) 2 = ημχ + συνχ {::} Για κάθε χ Ε JR έχουμε f'(x) 2 + ex ρα f (χ) = -c , c Ε R, χ > 3 . Η εφαπτοχ-3 1 1 {::} f'(x) = e χημχ + eχ συνχ Cr στο σημείο με τετμημένη 4 έχει κλίση {::} f'(x) = (eχημχ)' άρα f(x) = e χ ημχ + c, c Ε JR μένη λε = f' ( 4) = 2 + c 1 δηλ. Όμως ε/Ιε ι αλλά = λε λε {::} 2 + c] = 6 {::} c] = 4 . π1 f = 1 + eπ1 2 {::} eπ1 2ημ + c = 1 + e 2 {::} c = 1 f'(x) = -2- + 4 = 2 (x - 3)' + (4χ)' = Άρα 0χ-3 χ-3 άρα f(x) = e χημχ + 1, χ Ε JR . = [ 2Rn(x - 3) + 4χ]' 2) Έστω συνάρτηση f ορισμένη και πότε παραγωγίσιμη στο (Ο, +οο) για την οποία f(x) = 2Rn(x - 3) + 4x + c , c E lR, χ > 3 . Αλλά 2 2 το M(4, 1 6) E Cr {:} f(4) = 1 6 {:} c = 0 . Άρα ισχύει επιπλέον ότι: 2 f(x) = 2 ln(x - 3) + 4x, χ > 3 . α) x(e x f'(x) + Rnx) για κάθε χ>Ο. Η

(

Ι

)

( Ξ)

Ξ

I

=ι,

β) Η γραφική παράσταση της τέμνει τον άξονα χ'χ στο σημείο με τετμημένη ι. Να βρεθεί ο τύπος της. Λ ύ ση

Για χ>Ο έχουμε x(e x f'(x) + Rnx) = 1 {::} xe x f'(x) + xRnx = 1 {::}

4) Να βρεθούν οι παραγωγίσιμες στο (Ο, +οο) συναρτήσεις f, για τις οποίες ισχύει: _!_ _!_4 , χ > Ο ( ι). f'(x 3 ) = 4 +

ΕΥΚΛΕΙΔΗΣ Β' λζ' τ.2Π8

_

χ3

χ


Μαθη ματικά Γ Λυκείου

g1(ξ) = ο {::} 3f1(ξ) = 2 {::} f1(ξ) = � ο 3 Δηλαδή υπάρχει τουλάχιστον ένα ξ Ε (1, 4) ώστε ο συντελεστής διεύθυνσης της ' με 2 . ' ισος ' εφαπτομενης της cr στο ξ να ειναι J Αλλά λε · f1 (ξ) = - 1 δηλ .....

Λύ ση

1 1 f Ι (x) = 4 + -3 - -4 {:} f (χ 3 ) · 3χ 2 = χ χ 1 = 4 + -3 14 3χ 2 {:} χ χ 3 3 {::} fΙ(χ 3 ) · (χ 3 )Ι = 12χ 2 + - - -2 {::} χ χ 1 {::} [f(x 3 )]1 = 4χ 3 + 3fnx +

(

_

_

I

)

(

�)

άρα f(x 3 ) = 4χ 3 + 3fnx + 1. + c, c Ε IR . χ 3 Για χ = t > Ο είναι χ = :ift

�+ f(x) = 4χ + 3fn� + � +

άρα f(t) = 4t + 3.en:ift +

c

2) Έστω η άρτια και παραγωγίσιμη στο [2,2]συνάρτηση f. Να αποδείξετε ότι: α) η συνάρτηση g(x) = e1 - 2 '2 f(x) είναι επίσης •

άρτια και παραγωγίσιμη στο [-2,2]. β) υπάρχει τουλάχιστον ένας ξ ε [-2, 2] για τον οποίο ισχύει f '(ξ) = 4ξ f(ξ).

δηλ.

c, c Ε IR,

·

Λύ ση

χ> Ο.

α) Είναι g( -χ) = e 1 -2 ( - x )' · f( -χ) = = e 1 - 2 x' · f(x ) = g(x), x E [� 2, 2] άρα g(x) άρτια για χ Ε [ -2, 2] και παραγωγίσιμη αφού Ε) Θ. Rolle g1(- x ) = 4e 1 - 2 x' · f(-x) + e1 - 2 x' · f1(x) και f(χ) 1) Δίνεται συνάρτηση f συνεχής στο [1,4] παραγωγίσιμη στο [ -2,2]. παραγωγίσιμη στο (1,4) και f(4) = 2 + f(l) . β) Η g(x) είναι συνεχής στο [ -2,2] και α) Βρείτε την τιμή του λ ε IR ώστε να παραγωγίσιμη στο (-2,2) και g(2) = g( -2) (g ισχύουν οι προϋποθέσεις του Θ. Rolle άρτια) άρα ισχύει το θ. Rolle οπότε υπάρχει [1,4]για στο συνάρτηση τη ένας τουλάχιστον ξ Ε (-2, 2) ώστε g1(ξ) = Ο g(x) = 3f(x) + λχ . δηλ. -4ξe 1 - 2 ξ2 f(ξ) + e 1 -2 ξ' . f1 (ξ) = Ο {::} β) Για την τιμή του λ που βρέθηκε, {::} e1 - 2 ξ' (-4ξf(ξ) + f1 (ξ)) = Ο {::} aποδείξτε ότι η γραφική παράσταση της {::} f1(ξ) = 4ξ · f(ξ) , (e 1 -2 ξ' Ο) . f έχει μια τουλάχιστον εφαπτομένη που •

:;z:

είναι κάθετη στην ευθεία ε με εξίσωση 3χ - λy + 12 = 0 . Λύ ση

α) Η g(x) είναι συνεχής στο [1 ,4]σαν άθροισμα συνεχών συναρτήσεων και παραγωγίσιμη στο (1 ,4). Για να ισχύει το Θ. Rolle για την g(x) στο [ 1 ,4]πρέπει g(l) = g(4) {::} 3f(l) + λ = 3f(4) + 4λ {::} {::} -3λ = 3(f(4) - f(1)) {::} -3λ = 3 · 2 {::} ο {::} λ = -2 β) Η ευθεία ε έχει συντελεστή διεύθυνσης λε = l = 1_ Από (α) για λ = -2 ισχύει το θ. 2 λ Rolle για την g(x) άρα υπάρχει τουλάχιστον ένα ξ Ε (1, 4) ώστε _

_

3)

Δίνεταί η συνάρτηση f(x) = e(x - 2) .en(x + e - 1) - e•- ι . (χ - 1) . Να αποδείξετε ότι υπάρχει χ0 ε (1, 2) τέτοιο ώστε η εφαπτομένη της Cr στο σημείο με τετμημένη χ0, να είναι παράλληλη στον άξονα χ'χ. ·

Λύση

Είναι Ar = (1 - e, +οο) . Η f είναι παραγωγίσιμη στο [ 1 ,2] άρα και συνεχής στο [ 1 ,2]. Επίσης είναι f(l) = e(- 1)fne = -e και f(2) = -e · 1 = -e άρα από το θ. Rolle υπάρχει ένα τουλάχιστον Χ 0 Ε (1, 2) ώστε f1(x 0 ) = Ο . Δηλ. η εφαπτομένη της Cr στΌ σημείο με τετμημένη χ0 είναι παράλλη­ λη του άξονα χ ' χ.

ΕΥΚΛΕΙΔΗΣ Β' λζ' τ.2Π9


Μ αθη μ ατικά Γ Λυ κείου

4)

Δίνεται η εξίσωση

χ4 + 2 χ - 5 = Ο

(1).

β) Έστω

Αποδείξτε ότι: α) έχει δύο τουλάχιστον ρίζες στο R, μία τουλάχιστον στο (-2 ,-1) και μία τουλάχιστον στο (1,2 ). β) Η (1) έχει δύο μόνο ρίζες στο R. α) Εφαρ μόζουμε το

ότι έχει τρεις ρίζες ρ1

f(ρ1 ) = f( ρ2 ) = f( ρ 3 ) = Ο .

< ρ2 < ρ 3

Η f(x) είναι συνεχή ς

στα [ ρ 1 , ρ2 ] και [ ρ2 , ρ 3 ] κ αι παρ αγωγίσιμη στα (ρ 1 , ρ2 ) και (ρ2 , ρ 3 ) και f(ρ 1 ) = f( ρ2 ) , f(ρ2 ) = f( ρ 3 )

Rolle υπάρχει

άρα από το

θ.

ξ1 Ε (ρ1 , ρ2 ) ώστε f' (ξ1 ) = Ο και

ξ2 Ε (ρ2 , ρ 3 ) ώστε f'(ξ2 ) = Ο . Από το

θ. Bolzano για την

άρα

θ. Rolle

στο (ξ ι , ξ2) για την f'( x) υπάρχει ξ 3 Ε (ξ1 , ξ2 ) 3 ώστε f"(ξ 3 ) = Ο άτοπο , γιατί f'( x) = 4χ + 2

f(x) = x4 + 2x - 5 στα δ ιαστήματα (-2,- 1 ) και ( 1 ,2).

κ αι f "(x) = 1 2χ 2 > Ο . Ά ρα έχει μόνο δύο ρίζες στο

R η (1).

Αvο ικι ό Γ ρ ά μ μ α Είναι εξαιρετικά ση μαντική η προσφορά εργασιών πολλών συναδέλφων με & κλεκτές εργασίες για τη στήλη «Ευκλείδης προτείνει . . Ευκλείδη .

και

Διόφαντο»

Είμαστε περήφανοι για το επίπεδο της συνεργασίας. Ωστόσο το περιοδικό έχει μεγάλη ανάγκη απ ' τη βοήθεια των συναδέλφων κα τους άλλους τομείς ας πούμε για τις τάξεις . Θα παρακαλούσαμε οι συνάδελφοι να παρακολουθούν την εξέλιξη τη ς ύλη; στα σχολεία και να παρεμβαίνουν μέσω του περιοδικού στα διδακτικά δρώ μενα. Θα μας βοη θήσει πολύ ! Παρόμοια υπάρχει ανάγκη για συ μμετοχή σε γενικά άρθρα που μπορούν προκαλέσουν το ενδιαφέρον των αναγνωστών του περιοδικού .

Από τη Σ.Ε. Μ ε συναδελφικούς χαιρετισμούς

ΕΥΚΛΕΙΔΗΣ Β' λζ' τ.2/80

w


Εκδόσει� τη� ΕΛΛΗΝΙΚΗΣ ΜΑθΗΜΑΤΙΚΗΣ ΕΤΑΙΡΕΙΑΣ \Γ\'f!(ι

��Μ[�nς��

·

PWtAHΣ y'

nι ι.ι ι ' '"

� \ \ 1 1 \Ιk.ΙI:'. \1 \HII\1 \ 1 1 1\ t ll ..

�.....� ........

ιυcιιοεs

l 1 � 11'\ Hl.

τ·

-·-

LOUIS BRAND ΜΑθΗΜΑηΚΗ ΑΝΑΛVΣΗ

ΔJ�§J}J�I1 ι\l;)g}Jι1ι�\'ΓJ:ω UΛ'/ι\lfl�\ι θ1 . 1959 - 2000

ΙΙΣΑΙοπι Σ1ΙΣ - - E!WUIIΣ

Ιii!lllq ,ιΒΙJιrιaq ΙΙΙΙρΕΙΙ

ι \ H"IJ\11

\1\ΗΙ Ι '\ 1 \ Ι Ι Κ Ι Ι HIH \HH 1' \ !JI \

\:Ξ:III \01 11�11 1 ..11\1 \l \1 1 .. 0'

Σ t (Ι1χt-:ιιt\ΗΣ ΓιΩΜΕΙΨΙ,\ \lιΟ ΑΝ1ΗΊΩ'11 ΣΚΟΙΗΑ

I I"I IJ'

"1 1 1� 1 1 \I \1 \�H O.•l lll!"('""'-"

ll ι:.· ι. ι. ε,, ι f' Ε ι

κ ο ι·,.· ι '

\I ' ·ι ιι ι '' ι ι Ι ( .\· , • ιι J , ,. O H H ι r ι r .� ' 9 .J �......� '""'οι, •�••� "· '" . ........ . .. . .. ...... .......,..

J••M-.M ......M•fΊ>;.M ... fHih" ........... . ...... �...........

Τεύχος 2 ευρώ ΕυκλείδηςΑ' : Συνδρομή (4 τεύχη) 10 ευρώ (4 τεύχη + 2 ευρώ ταχυδρομικό) Σχολεία: 8 ευρώ δρχ. ' ΕυκλείδηςΒ': Τεύν, Λος 2,5 ευρω Συνδρομή (4 τεύχη) 12 ευρώ (4 τεύχη + 2 ευρώ ταχυδρομικό) Σχολεία: 10 ευρώ ΕυκλείδηςΓ: Τεύχος 5 ευρώ Συνδρομη' (2 τεύχη) 10 ευρώ Μαθημ. Επιθεώρηση: Τεύχος 5 ευρώ Συνδρομή (2 τεύχη) 10 ευρώ Αατρολόβος: Τεύχος 5 ευρώ Συνδρομή (2 τεύχη) 10 ευρώ Δελτίο (Bulletln): Τεύχος 7,30 ευρώ

Διεθνής Μαθηματικές Ολυμπιόδες 20 ευρώ 1959 • 1999: Βαλκανικές Μαθημαπκές Ολυμmόδες 1984-2001: 15 ευρώ Θέματα εξετόσεων στα A.E.I 1976 • 1989: 6 ευρώ Πρακτικό: 1ou ΓΙcινελλη.iου Συνεδρίου 6 ευρώ

2ou Πανrλληvίου Σιινεδρίοιι ...n... ι,. ' .... 3αJ nn. "�"• •.....w,:,. ' , ....... ι•ouu """"'"""

8 ευρώ 8 εuρω' ""........ ,..... .. """",""" 8 εuρω' 4αXSou l'h"'n ι,. ,. .... -;, ..Mn�.v 6ou Πανrλληνίου Συνεδρίου 8 ευρώ 7ou Πανrλληνίου Συνεδρίοu θ ευρώ 8ou Πανrλληνίου ΣιΝΕδρίου 8 ευρώ 9ou Πανrλληνίου Συνεδρίοu 8 ευρώ 10ou Πανrλληvίου Σιινεδρίοιι 17 ευρώ •

1 1ou Παν&λληvίου ΣιΝΕδρίου 17 ευρώ 14ou Πανrλληνίου Σuvεδρίου 1 7 ευρώ 15ou Πανrλληνίου ΣιΝεδρίου 1 7 ευρώ 1 6ou Πανrλληvίου Συνεδρίου 1 7 ευρώ 17ou Πανrλληvίου Συνεδρίου 25 ευρώ 18ou Γlcιvελλry.ιίου Συνεδρίοu 25 ευρώ ...n -;, •..w,:,. ,""" 2S εuρω ' ..... ,•.;,. 19ou nn. ' ""... ..,.,, �·"""" Συνέδριο Hermis '92 (Αγγλικό)

Συνέδριο Hermis '94 2 τόμοι ο τόμος Γραμμική Άλγεβρα

25 ευρώ (Αγγλικό) 15 ευρώ

(Gr. Muncres) 6 ευρώ ΗΡΩΝΟΣ ΑΛΕΞΑΝΔΡΕΩΣ (Ονόματα Γεωμετρικών όρων 20 ευρώ ΓΕΩΜΕΤΡΙΚΑ) (Μετρικό · Διόmρα) 20 ευρώ

5 ευρώ

Διαλtξεις: Ο τόμος Μαθηματική Ανόλυση

(Lου ι5 Βran d) , Δια φορικtς Εξισωσεις (Steρhenson)

' 25 ευρω 1 0 ευρώ

ιστορία Μαθηματικών ιoria

(4 Ίόμοι) Α, Β, Γι\ο ΓΒ ο Ίόμος 8 ευρώ 70 Χρόνια Ε.Μ.Ε. 4 ευρώ

Ελληνική Μαθηματική ' ' Β ι βλιοyρα φ ια 4 ευρω • • Στοιχειωδης Γεωμετρια από Ανώτερη Σκοπιό Θεωρία Αριθμών 10 Χρόνια 'ΈΥΚΛΕΙΔΗΣ"

ΣΕΙΡΑ ιι:

(1990-1999) CD

θ ευρώ

1 7 ευρώ 25 ευρώ

Τα παλαιότερα τεύχη όλ ω ν των εκδόσεων πωλούνται με τις τρέχουσες τιμές του 2003



Issuu converts static files into: digital portfolios, online yearbooks, online catalogs, digital photo albums and more. Sign up and create your flipbook.